You are on page 1of 89

12.

Differential Equation
A. Order and Degree of Differential Ans. (b) : Given, differential equation –
2
Equation d 2 y 3  dy 
= 1+  
1. The order of the differential equation dx 2  dx 
3
 d 2 y    dy  
2
d2 y dy
2x 2 2 − 3 +y = 0 =
 2     
 1 +
dx dx  dx    dx  
(a) 2 (b) 1 3 2
(c) 0 (d) not defined  d 2 y   dy 
SRM JEEE-2018  2  –   –1 = 0
 dx   dx 
Ans. (a) : Given that, differential equation Hence order = 2
d2 y dy Degree = 3
2x 2 2 – 3 + y = 0 5. The order of differential equation of all circles
dx dx
It is clear that order is 2. of given radius 'a' is
2. The order of the differential equation obtained (a) 2 (b) 3
by eliminating arbitrary constants in the family (c) 4 (d) 1
Karnataka CET-2015
of curves C1y = (C2 + C3) e x + c4 is
Ans. (a) : Given that, a circle having radius ‘a’ the
(a) 2 (b) 3 general equation of circles (r – h)2 + (y – k)2 = a2
(c) 4 (d) 1 It has two arbitrary constant h, and k, therefore the order
Karnataka CET-2020 of the given differential equation is 2.
Ans. (d) : Given that family of curve 6. The order and degree of the differential
C1 y = ( C2 + C3 ) e x + c4 dy 2
equation y = x + is
 C + C3  x C 4 dx dy
y= 2  e .e dx
 C1  (a) 1, 2 (b) 1, 3
y = C ⋅ ex (c) 2, 1 (d) 1, 1
Hence order is one. Karnataka CET-2014
3. The order of the differential equation dy 2
Ans. (a) : Given, y = x +
y = C1eC2 + x + C3eC4 + x is dx dy
(a) 1 (b) 3 dx
(c) 2 (d) 4 dy  dy 
2

Karnataka CET-2019 y = x  + 2
Ans. (a) : Given, differential equation
dx  dx 
2
y = C1eC2 + x + C3eC4 + x  dy  dy
x  – y + 2 = 0
y = C1e .e + C3e .e
C2 x C4 x  dx dx
Hence order → 1 and degree 2
y = ( C1e + C3e ) e
C2 C4 x
7. If 'm' and 'n' are the order and degree of the
y = ke x
(Where, k = C1e + C3e )
C2 C4 differential equation
( )
3
dy y ''
( y '') + 4 ''' + y ''' = sin x, then
x 5
= ke
dx y
dy (a) m = 3, n = 5 (b) m = 3, n = 1
=y (c) m = 3, n = 3 (d) m = 3, n = 2
dx
dy Karnataka CET-2013
–y=0 Ans. (d) : Given differential equation
dx
( y '' )
3
Hence, order is one.
4. The degree and order of the differential ( y ) + 4 y''' + y''' = sin x
'' 5

2
d2 y 3  dy   d2y 
3
equation = 1 +   respectively are
dx 2
 dx  5  2
 d2 y  dx d3 y
(a) 2 and 3 (b) 3 and 2 ⇒  2  + 4  3  + 3 = sin x
 dx   d y  dx
(c) 2 and 2 (d) 3 and 3  3
Karnataka CET-2018  dx 
Differential Equation 1109 YCT
2 3 5 Squaring both side, we get –
 d3 y   d 2 y   d 2 y   d3 y   d3 y 
 3  + 4  2  +  2   3  = sin x  3  1  d2y 
3

 dx   dx   dx   dx   dx  1+ 2
=  2
Hence, order (m) = 3  dy   dx 
Degree (n) = 2  
 dx 
8. If m and n are degree and order of 2 3 2
 dy   d 2 y   dy 
+ + 1 =  2  
( ) m n  
2 / 3
1 + y12 = y 2 , then the value of is  dx   dx   dx 
m−n
(a) 3 (b) 4 Order (highest derivation of equation) is 2 and degree
(power of highest derivative) is 3.
(c) 5 (d) 2
Karnataka CET-2011 11. The order of the differential equation of all
circles which is lie in the first quadrant and
Ans. (c) : Given, touch both the axis is
( 1)
2 / 3
1 + y 2
= y 2
(a) four (b) one
(c) three (d) two
2
  dy  2   d 2 y 
3
MHT CET-2019
1 +    =  2  Ans. (b) : The differential equation of all circle
  dx    dx  (x – h)2 + (y – k)2 = a2
2 2 Since circle lie in the first quadrant and touches both the
 d 2 y    dy  
3

 +  = axis.
 2     – 1 0
 dx    dx   Then, center (h, k) = (a, a)
So, m and n are degree and order are 3 and 2 Equation of circle,
respectively. (x – a)2 + (y – a)2 = a2
m + n 3+ 2 x2 + y2 – 2ax – 2ay + a2 = 0
∴ = =5 Here, number of arbitrary constant is one
m– n 3– 2 Hence, order is 1
9. The order and the degree of the differential 12. The order of the differential equation of all
7
circles whose radius is 4, is
  dy  3  3  d2 y  (a) 4 (b) 3
equation 1 +    = 7  2  are
  dx    dx  (c) 2 (d) 1
(a) 2, 3 (b) 2, 2 MHT CET-2019
(c) 3, 2 (d) 3, 3 Ans. (c) : The differential equation of all the circle
MHT CET-2016, 2019, 2020 whose 2radius is 42 2
COMEDK-2012 / Karnataka CET-2007 (x – h) + (y – k) = 4
Ans. (a) : Given that, differential equation It has two arbitrary constant then order is 2
7 / 3 13. The order of the differential equation of all
  dy 3   d2 y  parabolas, whose latus rectum is 4a and axis
1 +    = 7  2  parallel to the x-axis, is
  dx    dx  (a) one (b) four
3 7
  dy     d y   2
3 (c) three (d) two
1 +    = 7  2   MHT CET-2018
  dx     dx   Ans. (d) : Let the equation of all parabola.
3 7 (y – k)2 = 4a (x – h)
 d 2 y    dy  
3

343  2  – 1 +    = 0 Differentiating w.r.t. x, we get–


 dx      dx
2(y – k)
dy
= 4a
Hence, order is 2 and degree is 3 dx
10. The order and degree of the differential Again differentiating w.r.t. x, we get –
2
3 d 2 y  dy 
1  d y 2
2 (y – k) +   =0
equation 1 + =   are respectively dx 2  dx 
 dy 
2
 dx 2  3
  d 2 y  dy 
 dx  2a +   =0
dx 2  dx 
(a) 2, 3 (b) 3, 2
(c) 3, 3 (d) 2, 2 Hence order is 2.
MHT CET-2020, 2009, 2008, 2007 14. If m and n are order and degree respectively of
Karnataka CET-2004 the differential equation.
Ans. (a) : Given differential equation 3
 d2 y 
d y
3/ 2
d y
5  
 dx 2  +  d y  = x 2 – 1, then
2 2 3
1
1+ 2
= 2    + 4  
 dy   dx   dx 2   d 3 y   dx 3 
   3
 dx   dx 
Differential Equation 1110 YCT
(a) m = 3, n = 3 (b) m = 3, n = 2 Ans. (a) : The equation of a family of circle of radius r
(c) m = 3, n = 5 (d) m = 3, n = 1 passing through the origin and having center on y – axis
JCECE-2015 is (x – 0)2 + (y – r)2 = r2
MHT CET-2011 or x2 + y2 – 2ry = 0
Ans. (b) : Let m and n are order and degree of So, this is one parameter family of circle, so its
differential equation. differential equation is order one.
3
18. The differential equation of all circles of radius
 d2y  ‘a’ is of order
5  2 (a) 2 (b) 3
 d2 y   dx  +  d y  = x 2 – 1
3

 2 + 4  3 (c) 4 (d) 5
 dx   d y   dx 
3
SRM JEEE-2011
 3
 dx  Ans. (a) : Let, (h, k) be the centre of the circle. Then,
2 3 5 equation of circle having radius ‘a’ be
 d3 y   d 2 y   d 2 y   d3 y   d3 y 
 3  + 4  2  +  2   3  = (x – 1)  3 
2
(x – h)2 + (y – k)2 = a2 …(i)
 dx   dx   dx   dx   dx  Differentiating both the sides, we get–
Hence, order (m) is 3 and degree (n) is 2. dy
15. The order of the differential equation 2(x – h) + 2(y – k) =0
dx
d2 y dy  x−h
2x 2 2 − 3 +y = 0 dy
dx dx = −  ...(ii)
(a) 2 (b) 1
dx  y−k 
(c) 0 (d) not defined Again differentiating both the sides, we get–
2
SRM JEEE-2018 d 2 y  dy 
(y − k) 2 +   = −1
Ans. (a) : The differential equation dx  dx 
2
2 d y dy 2
2x –3 +y=0  dy 
dx 2 dx −1 −  
Order is 2 and degree is 1 (y – k) =  dx 
16. The order and degree of the differential d2 y
2
equation of all tangent lines to the parabola x dx 2
= 4y is From equation (i), we have
(a) 1, 2 (b) 2, 2 (x − h) 2 a2
(c) 3, 1 (d) 4, 1 +1 =
SRM JEEE-2007 (y − k) 2
(y − k)2
Ans. (a) : Given, parabola x2 = 4y 2  dy  
2

Equation of tangent ( y − k )    + 1 = a
2

  dx  
a
x = my + 2 2
m   dy  
Where, m is an arbitrary constant  −1 −    
 dx    dy  + 1
2

Differentiating w.r.t. x, we get – a2 =   


 d2 y   dx 
dy    
1= m dx 2
dx  
2 2
1 
2 d y
2
   dy 
2
  dy 2 
m= a  2  =    + 1   + 1
 dy 
   dx    dx  
  dx  
 dx Hence, order is 2.
Putting the value of m
19. The degree and order of the differential
b dy
x= + dy
dy dx equation y = px + a 2p 2 + b 2 ,where p =
dx
dx respectively is
Required equation
2
(a) (2, 1) (b) (2, 2)
 dy  dy (c) (1, 2) (d) (1, 1)
  −x +y=0
 dx  dx BCECE-2010
Hence, order is 1 and degree is 2 SRM JEEE-2012
17. The order of the differential equation of all Ans. (a) : Given,
circles of radius ‘r’, having center on y-axis y = px + a 2 p 2 + b 2
and passing through the origin is
2
(a) 1 (b) 2 dy 2  dy 
(c) 3 (d) 4 y = x⋅ + a   + b2
dx  
dx
SRM JEEE-2010
Differential Equation 1111 YCT
Squaring both side, we get– 1
2 y = 1+
ex
dy   2  dy  
2 2
 2 
 y − x  = a   + b Differentiating w.r.t. x, we get –
 dx    dx   dy
  = 0 + (–e – x )
 dy 
2
dy  dy 
2 dx
y 2 + x 2   − 2xy = a 2   + b2 Again differentiating w.r.t. x, we get –
 dx  dx  dx  d2 y
∴ Order = 1, Degree = 2. = –1(–1)e – x = e – x
20. The order and degree of the differential dx 2
equation d2 y 1
=
 dy 
2/3
d3 y dx 2 e x
1 + 3  =4 are equal to 23. The product of the degree and order of the
 dx  dx 3 2 3
 2  d 2 y   dy 
(a) 1,  (b) (3, 1) D.E.  2  –   = y 3 is
 3  dx   dx 
(c) (3, 3) (d) (1, 2) (a) 4 (b) 6
CG PET-2018 / SRM JEEE-2016 (c) 2 (d) 3
JCECE-2010 / Jamia Millia Islamia-2005 COMEDK-2015
AIEEE-2002 Ans. (a) : Given differential equation is
2
Ans. (c) :Given, differential equation.  d 2 y   dy 
3

 2   =y
3
2/3 3 –
 dy  d y  dx   dx 
1 + 3  = 4 3
 dx  dx So, order = 2 and degree =2
Cubic on both sides, ∴ Product of the degree and order is,
2
d y
3 3 =2×2=4
 dy 
1 + 3  = 64  3  24. Order and degree of the differential equation
 dx   dx  d4y  d3y 
Hence, order and degree of the differential equation are 4
+ sin  3  = 0
3 and 3 respectively. dx  dx 
dy (a) order = 4, degree = 1
21. The general solution of x 2 = 2 is- (b) order = 3, degree = 1
dx (c) order = 4, degree = not defined
2 2
(a) y = c + (b) y = c − (d) order = 4, degree = 2
x x JCECE-2018
3 COMEDK-2016
(c) y = 2cx (d) y = c − 3
x d4y  d3 y 
BITSAT-2007 Ans. (c) : Given, 4
+ sin  3  = 0
Ans. (b) : Given, dx  dx 
dy 2 The highest order derivative which occurs in the given
= 2 4
d y
dx x differential equation is , therefore its order is 4.
2 dx 4
dy = 2 dx As the given differential equation is not a polynomial
x
Integrate both side, we get– dy
equation is , therefore its degree is not defined.
2 dx
y=− +c 25. The order and degree of the differential
x
x
equation whose solution is
e +1 y = cx + c2 – 3c3/2 + 2,
22. The second order derivative of x
is
e where c is a parameter, is
x 1 (a) order = 4, degree = 4
(a) e (b) x
e (b) order = 4, degree = 1
ex − 1 1 (c) order = 1, degree = 4
(c) x
(d) e x + x (d) None of these
e e
MHT CET-2004 VITEEE-2018
Ans. (c) : We have,
ex + 1 y = cx + c 2 − 3c3/ 2 + 2
Ans. (b) : Given y = x ....(i)
e Differentiating both side with respect to x, we have
ex 1 dy
y= x + x =c
e e dx
Differential Equation 1112 YCT
Putting this value of c in equation (i), we have 28. The degree of the differential equation of all
dy  dy 
2 3/ 2 curves having normal of constant length c is
 dy 
y=x +   − 3  +2 (a) 1 (b) 3
dx  dx   dx  (c) 4 (d) None of these
2 3/ 2
dy  dy   dy  VITEEE-2010
y−x −   − 2 = −3   Ans. (d) : Length of normal = c
dx  dx   dx 
2
Squaring both side, we have–  dy 
2 y 1+   = c
 dy  dy 
2
  dy 
3
 dx 
y − x −   − 2 = 9   On squaring both sides,
 dx  dx    dx 
So, order is 1 and degree is 4   dy  
2

y 2 1 +    = c 2
26. The order and degree of differential equations,   dx  
2 3/2 Clearly, this is the differential equation of degree 2.
  dy  
1 +    29. The order and power of differential equation
  dx  
p=  are, respectively d2 y dy
dx ∫
d2y 2
+7 + ydx = sin x is
2
d x
dx (a) 1, 3 (b) 3, 1
(a) 2, 2 (b) 2, 3 (c) 1, 2 (d) 2, 1
(c) 2, 1 (d) None of these UPSEE-2010
JCECE-2016 Ans. (b) : Given differential equation
VITEEE-2012
d2 y dy
Ans. (a) : Given, differential equation–
2
+ 7 + ∫ ydx = sin x
3/2 dx dx
  dy 2  Differentiating with respect to x, we get–
1+   
  dx   d3 y d2y dy
p=  3
+ 7 2
+y = cos x
2
d y dx dx dx
Clearly order is 3 and degree is 1
dx 2
3/ 2
 d 2 y    dy  
2 dy 1 − y2
p  2  = 1 +    30. The solution of the equation = is
 dx    dx  
dx 1 − x2
On squaring both sides, we get (a) sin −1 y − sin− 1x = c

 d 2 y    dy  
2 2 3
(b) sin −1 y + sin− 1 x = c
p  2  = 1 +   
2
(c) sin −1 ( xy ) = 2
 dx    dx  
Clearly, it is a second order differential equation of (d) None of the above
degree 2. UPSEE-2009
Note: The higher order derivative is in the dy 1 – y2
transcendential, then we do not determined the degree Ans. (a) : Given =
of the equation. dx 1 – x2
27. The order and degree of the differential dy dx
2/3
=
2
 dy  d2y 1– y 1 – x2
equation  1 + 4  = 4 2 are respectively
 dx  dx Integrating both sides, we get –
sin–1y = sin–1x + c
2
(a) 1, (b) 3, 2 sin–1y – sin–1x = c
3 31. The length of the normal to the curve
2 π
(c) 2, 3 (d) 2,
3 x = a ( θ + sin θ ) , y = a ( 1 − cos θ ) at θ = is
2
VITEEE-2011
a
Ans. (c) : Given differential equation is (a) 2a (b)
2 2
 dy  3 d2 y a
1 + 4  = 4 2 (c) (d) 2a
 dx  dx 2
Cubic on both sides, UPSEE-2009
2 3 Ans. (d) : Given, x = a (θ + sinθ), and y = a(1 – cosθ)
3d y
2
 dy 
1 + 4  = 4  2  Differentiating we get –
 dx   dx  dx dy
Hence, order is 2 and degree is 3. = a(1 + cos θ) and = a(0 + sin θ)
dθ dθ
Differential Equation 1113 YCT
dy d3y  d 2y 
dy dθ a sin θ 34. + 2 1 + 2  = 1 has degree and order as:
= = dx3  dx 
dx dx a(1 + cos θ) (a) 3, 1 (b) 3, 2
dθ (c) 1, 3 (d) 2, 3
dy sin θ
= JCECE-2004
dx 1 + cos θ Ans. (c) : Given, the differential equation
π d3 y  d2y 
sin +
 dy  2 = 1 =1 2 1 + 2  = 1
  π= π 1+ 0
dx 3  dx 
 dx θ= 1 + cos Hence, order is 3 and degree is 1
2
2
y = a(1 – cosθ) 35. Order and degree of the differential equation
1
π
d 2 y   dy   4
2
at, θ=
2 =     are
y +
dx 2   dx  
 π
y = a  1 − cos  = a(1 – 0) (a) 4 and 2 (b) 1 and 2
 2
(c) 1 and 4 (d) 2 and 4
y=a
BCECE-2017
2
 dy  Ans. (d) : Given,
∴ Length of normal = y 1 +   = a 1 + 1 = 2a
 dx  1

d 2 y   dy   4
2
32. Find the order and degree of the differential = +
   
y
equation dx 2   dx  
3/5 Taking to the power of 4 in both sides,
 d4 y  d3 y d2 y dy
 4  – 5 3 + 6 2 – 8 +5 = 0
 d2 y 
4 2

 dx  dx dx dx  dy 
Now,  2  = y +  
(a) 4, 3 (b) 3, 4  dx   dx 
(c) 4, 5 (d) 5, 4 4 2
 d 2 y   dy 
JCECE-2012  2  –  –y =0
Ans. (a) : Given differential equation –  dx   dx 
3/ 5 Hence, order is 2 and degree is 4
 d4 y  d 3y d2 y dy
 4 −5 3
+6 2 −8 +5 = 0 36. The order and degree of the differential
 dx  dx dx dx
2
3/ 5 dy  dy 
 d4 y  d3 y d2y dy equation y = x + a 2   + b 2 is
 4  = 5 3 – 6 2 +8 –5 dx  dx 
 dx  dx dx dx
(a) 3,1 (b) 1,3
Now, taking to the power of 5 in both sides, (c) 2,1 (d) 1,2
3 5
 d 4 y   d3 y d2 y dy  BCECE-2016
 4  =  5 3 – 6 2 + 8 – 5 Ans.(d): Given, differential equation
 dx   dx dx dx 
2
Hence, order is 4 and degree is 3 dy 2  dy 
33. The differential equation of all conics whose y = x + a   + b2
dx  dx 
center is lie at the origin is of order
2
(a) 2 (b) 3 dy 2  dy 
(c) 4 (d) None of these y – x = a   +b
2

JCECE-2011
dx  dx 
Ans. (b) : Given, the center of the conic lies at the Squaring on both sides, we get–
2 2
origin (0, 0)  dy  2  dy 
The equation of all conic  y – x  = a   +b
2

 dx   dx 
ax2 + by2 + 2hxy + 2gx + 2fy + C = 0 Hence order is 1and degree is 2
Center lies of origin (–g, –f) = (0, 0) the equation
37. The differential equation representing the
ax2 + 2hxy + by2 + C = 0 family of curves y2 = 2c(x + c2/3), where c is a
Dividing by a in the above equation, we get– positive parameter, is of
2h b c (a) order 3, degree 3 (b) order 2, degree 4
x2 + xy + y 2 + = 0
a a a (c) order 1, degree 5 (d) order 5, degree 1
Since, the equation has three parameters AMU-2009
2h b c Ans. (c) : The given differential equation is,
, ,
a a a y2 = 2c(x + c2/3) …(i)
Hence, order of equation is 3 On differentiating both side, we get-
Differential Equation 1114 YCT
dy dy 39. Family y = Ax + A3 of curve is represented by
2y = 2c (1 + 0 ) , c=y the differential equation of degree
dx dx
Putting the value of c in equation (i), we get- (a) 3 (b) 2
(c) 1 (d) None of these
dy   dy  
2/3

y 2 = 2y  x +  y   CG PET- 2005
dx   dx   Ans. (a) : We have family of curve,
2
+1 y = Ax + A3 ….(i)
dy  dy  3 Now differentiating w.r.t x, we get-
y − 2xy
2
= 2 y 
dx  dx  dy
5/3 =A
dy  dy  dx
y 2 − 2xy = 2 y 
dx  dx  Putting the value of A in equation (i), we get-
3
Take cube on both side, we get- dy  dy 
3 5 y=x + 
 2 dy  3 dy  dx  dx 
 y − 2xy  = 2  y 
 dx   dx  Hence, degree of differential equation is 3.
So, Order = 1, Degree = 5 40. The degree and order of the differential
38. The order and the degree of the differential equation of the family of all parabolas whose
equation of the family of all circles of radius c, axis is X- axis, are respectively
where c is fixed, are respectively (a) 2, 1 (b) 1, 2
(a) 2, 1 (b) 2, 3 (c) 3, 2 (d) 2, 3
(c) 2, 2 (d) 3, 3 Manipal UGET-2011
AMU-2019 CG PET- 2005 / AIEEE-2003
Ans. (c) : : Let equation of the family of circle whose Ans. (b) : The equation of family of parabola whose
centre (h, k) and radius is c then axis is X axis i.e focus lies on x-axis.
(x – h)2 + (y – k) 2 = c2 ….(i) Then, y2 = 4a (x – h) ….(i)
Differentiating w.r.t x, we get-
Differentiating w.r.t x, we get-
dy
2(x – h) + 2 (y – k) =0 dy
dx 2y = 4a
dx
dy
(x – h) + (y – k) =0 dy
dx y = 2a
dx
dy
(x – h) = –(y – k) ….(ii) Again differentiating w.r.t x, we get-
dx 2
Now again differentiating w.r.t x we get d 2 y  dy 
y +   =0
d 2 y dy dy dx 2  dx 
1 + (y – k) + . =0 Hence required differential equation has,
dx 2 dx dx
2
Degree = 1
d 2 y  dy  Order = 2
1 + (y – k) +  = 0
dx 2  dx  41. The order and degree of the differential
Putting the value of (x – h) in equation (i), we get- equation
2 1/3
 dy   d 2 y   dy 
( ) ( )
2
 − y − k + y − k = c 2
  + 1/4
  + x = 0 are respectively
 dx  2
 dx   dx 
  dy 2  (a) 2,3 (b) 3,3
(y – k)2 1 +    = c 2 (c) 2,6 (d) 2,4
  dx   CG PET- 2006
Put the value of (y – k) in equation (ii), we get– Ans. (a) : We have given differential equation,
  dy  2  1
1 +    
2 1
2
 d y  dy  3

  dx    1 +  dy   = c 2 2
+   + x4 = 0
dx  dx 
 d 2 y    dx  
    1
2 1
 dx   dy  3 d2 y
  + x 4
= −
  dy 2 
3 2
 dx  dx 2
2d y
2
1 +
    = c  2 Taking cube on both side, we get-
  dx    dx  3
 1
1 3
  + x 4  =  − d y 
2
This is the required differential equation. dy 3

Hence, Order = 2  dx    dx 2 


Degree = 2  
Differential Equation 1115 YCT
3 45. The order and degree of the differential
 1
 1
1 
1
1 3
 dy   + x 4 + 3  dy  x 4  dy  + x 4  = −  d y  equation ( y') = 1 – y''' are respectively.
3 2 4
3 3 3

 dx      dx   2
 dx   dx  (a) 3,4 (b) 1,2
   
2 1 (c) 3,2 (d) 3,1
3
dy 3
 dy  3
1
 dy  3
1
 d2 y  CG PET- 2017
+ x 4 + 3  x 4 + 3  x 2 = −  2 
Ans. (d) : We have, ( y ' ) = 1 − y '''
4
dx  dx   dx   dx 
Hence, Order = 2 4
 dy  d3 y
Degree = 3   = 1 −
42. What is the degree of the differential equation  dx  dx 3
2/ 3 On squaring both sides, we get –
 d3y  d2y dy
 3  + 4 − 3 dx 2 + 5 dx = 0?
8
 dy  d3 y
 dx    = 1 −
(a) 1 (b) 2  dx  dx 3
(c) 3 (d) 4 d 3 y  dy 
8

SCRA-2012, 2014 +  =1
BITSAT-2011 dx 3  dx 
Ans. (b) : Given, differential equation are– Hence, order =3 and degree=1
2 / 3 46. What is the degree of the differential equation
 d3 y  d2 y dy 2 3
 3  +4–3 2 +5 =0  dy  x y  dy  x y  dy 
2 2 3 3

 dx  dx dx x = 1 + xy   +   +   + ..?
 dx  2  dx  6  dx 
2/3
 d3 y  d2 y dy (a) 1 (b) 2
 3  = 3 2 −5 −4 (c) 3 (d) can not be determined
 dx  dx dx SCRA-2010
Taking cube on both sides, we get – Ans. (a) : The given differential equation,
2 3
 d3 y   d 2 y dy  2 2 2
 dy  x y  dy  x y  dy 
3 3 3

 3  = 3 2 – 5 – 4 x = 1 + xy   +   +   + ...
 dx   dx dx   dx  2  dx  6  dx 
Hence, degree = 2  dy 
43. The order and degree of the differential Let, t =  xy dx 
2
 
4
 d3 y  d2y  dy  t t 2
t3
equation  3  − 3 2 + 2   = y are 4
x = 1 + + + + ......
 dx  dx  dx  1! 2! 3!
(a) 1, 4 (b) 3, 4  x x x2 x3 
(c) 2, 4 (d) 3, 2 Q e = 1 + + + + ....∞ 
CG PET- 2012  1! 2! 3! 
Ans. (d) : We have the following differential equation, So,
2 4 x = et …..(i)
 d3 y  d2 y  dy  Taking log both sides, we get-
 3 − 3 + 2   = y 4

 dx  dx 2  dx  log x = log e e t
Hence, Order = 3 log x = t log e e
Degree = 2
44. The order and degree of the differential log x = t
2 1/4
On putting the value of t, we get-
d 2 y   dy   dy
equation = y +    are given by log x = xy
dx 2   dx   dx
(a) 4 and 2 (b) 1 and 2 Hence, Order = 1
(c) 1 and 4 (d) 2 and 4 Degree = 1
CG PET- 2013 47. What is the degree of the differential equation
Ans. (d) : Given differential equation, corresponding to the family of curves
1 y = a(x+a)2 where 'a' is an arbitrary constant?
d y   dy  
2 2 4 (a) 3 (b) 2
=  y +    (c) 1 (d) Not defined
dx 2   dx   SCRA-2010
It can be written as, Ans. (a) : The given family of curve,
y = a(x + a)
4
 d 2 y    dy  
2 2
….(i)
 2  = y +   
 dx    dx   On differentiating w.r.t. x, we get-
Hence, Order → 2 dy
= a × 2 (x + a )
Degree → 4 dx
Differential Equation 1116 YCT
dy On putting the value of a in equation (i), we get-
= 2ax + 2a 2
dx dy  y dy 
y2 = 2y  x + 
Again, differentiating w.r.t. x, we get – dx  2 dx 
d2 y dy  y dy 
= 2a + 0 y = 2 x + 
dx 2 dx  2 dx 
1 d2 y dy  dy 
2
a= y = 2x + y 
2 dx 2
On putting the value of a in equation(i), we get-
dx  dx 
2
2  dy   dy 
1 d2y  1 d2y  y   + 2x   − y = 0
y= 2 
x+   dx   dx 
2 dx  2 dx 2  Which required differential equation.
1 d2 y  2 1  d2 y  d2 y  Hence, Order = 1
2

y= 2
 x +  2  +x 2 50. The degree and order respectively of the
2 dx  4  dx  dx 
  differential equation of the family of the curves
3 2
x 2 d2 y 1  d2 y  x  d2 y  represented by y = c(x + c) are (Here, c is a
y= +   +   parameter)
2 dx 2 8  dx 2  2  dx 2 
(a) 1, 3 (b) 2, 3
So, (c) 3, 1 (d) 2, 2
Order = 2 AP EAMCET-22.04.2019, Shift-II
Degree = 3
Ans. (c) : The family of curve,
48. What is the degree of the differential equation
obtained from y = cx2+c-1 where c is an
arbitrary constant?
y= (
c x+ c )
(a) 4 (b) 3 On squaring both side we get –
(c) 2 (d) 1 y2 = c (x + c ) ….(i)
SCRA-2010 On differentiating w.r.t x, we get-
Ans. (c) : The given differential equation, dy
2y =c
1 dx
y = cx 2 + ….(i) On putting the value of 'c' in equation (i),we get-
c
On differentiating equation (i) w.r.t. x, we get- dy  dy 
y ' = 2cx + 0 y2 = 2y  x + 2y 
dx  dx 
y'
c= dy
2x Put, y′ =
On putting this value in equation (i), we get- dx
y
y' 2 1 xy ' 2x x ( y ' ) + 4x
2
∴ = x + 2yy′
y= x + = + = 2y′
2x y '/ 2x 2 y' 2y ' y
2y ' y = x ( y ' ) + 4x
2 − x = 2yy′
2y′
2yy '− x ( y ') = 4x On squaring both side, we get-
2
2
 y 
( )
2
So, Degree = 2
Order = 1  ′ − x  = 2yy′
 2y 
49. The order of the differential equation 2
y xy
corresponding to the family of parabolas whose + x2 − = 2yy′
4 ( y′ ) y′
2
axes are along the X-axis and whose foci are at
the origin, is y2 + 4x2(y′)2 – 4xyy′ = 8y(y′)3
(a) 4 (b) 3 2 3
(c) 2 (d) 1  dy   dy   dy 
y2 + 4x2   – 4xy   = 8y  
AP EAMCET-20.04.2019, Shift-II  dx   dx   dx 
Ans. (d) : Equation of the family of parabolas whose Hence, degree = 3 and order = 1
focus at (0, 0) and axis is x-axis,
51. The degree of the differential equation
y2 = 4a (x + a) ….(i) 2 3
On differentiating w.r.t. x, we get-  dy  1  dy  1  dy 
x = 1 +   +   +   +……
dy  dx  2!  dx  3!  dx 
2y = 4a
dx (a) 3 (b) 2
y dy (c) 1 (d) Not defined
a= Manipal UGET-2016
2 dx BCECE-2015 / WB JEE-2010
Differential Equation 1117 YCT
Ans. (c) : Given that equation, (y – h)2 = 4a (x –k)
2 3 Where h, k and a are arbitrary constant since, there are
 dy  1  dy  1  dy 
x =1 +   +   +   + .... three arbitrary constant in the equation.
 dx  2!  dx  3!  dx  So, order of differential equation is 3.
We know that, 55. The degree of the differential equation
x2 x3 x4 1 – x 2 + 1 – y 2 = a ( x – y ) is
ex = 1 + x + + + + .... satisfying
2! 3! 4! (a) 1 (b) 3
1 1
Then, x = 1 + y′ + + ( y ') + ( y ' ) + .... (c) 2 (d) none of these
2 3

2! 3! AMU-2012 / CG PET-2016
x = e y′ Jamia Millia Islamia-2009
Taking log both side, we get- Ans. (a) : The differential equation is,
y′ = log x 1 – x 2 + 1 – y2 = a ( x – y )
dy
= log x Put x = sinα and y = sinβ
dx
Hence, degree of differential equation is 1. Q 1 – sin 2 α + 1 – sin 2β = a ( sinα – sinβ )
52. If 1, ω, ω2 are cube roots of unity, cosα + cosβ = a (sinα – sinβ)
1 ω n
ω 2n  α +β  α –β  α +β α –β
2cos   cos   = a  2cos sin 
then ω 2n 1 ωn has value  2   2   2 2 
ωn ω 2n 1 α –β α –β
cot   = a sin  
(a) 0 (b) ω  2   2 
(c) ω2 (d) ω + ω2 α–β
= cot –1a
WB JEE-2008 2
Ans. (a) : Given, 1, ω, ω2 are cube roots of unity. α – β = 2 cot –1a
We have, sin–1 x – sin–1y = 2cot –1a
 1 ωn ω2n  1

1 dy
=0
 2n  2 2 dx
ω 1 ωn  1– x 1– y
 ωn ω2n 1  Thus, degree of differential equation is 1.

56. The differential equation whose solution is Ax2
∆ = 1(1 – ω ) – ωn(ω2n – ω2n) + ω2n(ω4n – ωn)
3n
+ By2 = 1, where A and B are arbitrary
=(1 – 1) – ωn (0) + ω2n (ωn – ωn) constant, is of
=0–0+0=0 (a) first order and second degree
53. If p and q are the order and degree of the (b) first order and first degree
differential equation (c) second order and first degree
3 (d) second order and second degree
dy  d2y  AIEEE-2006
y + x 3  2  + xy = cos x, then
dx  dx  Ans. (c) : Given the solution,
(a) p < q (b) p = q ax 2 + by 2 = 1 .....(i)
(c) p > q (d) none of these
AMU-2016 Now differentiating equation (i) w.r.t. x, we get–
Ans. (a) : Given, p and q are the order and degree of the dy y  dy  a
2ax + 2by = 0 or   = −
differential equation, dx x  dx  b
dy  d2 y 
3 Again differentiating w.r.t. x, we get
y + x 3  2  + xy = cos x
 dy   dy  
2
dx  d2 y 
 dx  x   + y  2  − y   
It shows that order is 2 and degree is 3.  
dx  dx   dx  
=0
Hence p < q 2
x
54. The differential equation of all parabolas 2
whose axis of symmetry is parallel to x-axis is  d2 y   dy   dy 
or, xy  2  + x   − y   = 0
of order  dx   
dx  dx 
(a) 2 (b) 3 This is the differential equation of order 2 and degree 1.
(c) 1 (d) 4
AMU-2016 57. The differential equation representing the family
Ans. (b) : Since, the parabola is symmetric about a line ( )
of curves y2 = 2c x + c , where c > 0, is a
parallel to x-axis. parameter, is of order and degree as follows
So, parabola may be leftward or rightward. Let us (a) order 2, degree 2 (b) order 1, degree 3
consider the parabola is rightward. The family of (c) order 1, degree 1 (d) order1, degree 2
parabola is rightward whose axis of symmetry is Jamia Millia Cslamia-2012
parabola to x-axis is represented by the equation. J&K CET-2009 / AIEEE-2005
Differential Equation 1118 YCT
Ans. (b) : y2 = 2c (x + c) 59. The degree of the differential equation
3 y 3/2 1/2
2 – y 1 – 4 = 0 is
y 2 = 2cx + 2c 2
……(i) (a) 6 (b) 3
On differentiating with respect to x, we get– (c) 2 (d) 4
dy Jamia Millia Islamia-2013
2y = 2c
dx Ans. (a) : y23/ 2 = y11/ 2 + 4
 dy  Squaring on both sides we get
c = y  …..(ii)
 dx  y32 = y1 + 16 + 8y1/1 2
On substitution equation (ii) in (i) we get–
(y − y1 − 16 ) = 64y1
3 2
3 2
 dy   dy  2
y 2 = 2xy   + 2  y  y 62 − 32y 32 − 2y32 .y1 + y12 + 32y1 + 256 = 0
 dx   dx 
2 3
Hence, the degree of the given equation is '6'.
 2  dy   2 3  dy 
 y − 2xy    = 2 y   60. Find the degree of the differential equation
  dx    dx  y 2/3 + 2 + 3y + y1 = 0
3 2
2 3
 2  dy   3  dy  (a) 4 (b) 2
 y − 2xy    = 4y  
  dx    dx  (c) 3 (d) 1
order = highest differential equation AP EAMCET-21.09.2020, Shift-I
Degree = highest degree of order Ans. (b) : Given, differential equation,
∴ order = 1 and degree = 3 y 32 / 3 + 2 + 3y3 + y1 = 0
58. The difference between degree and order of a Then, it is also written as-
differential equation that represents the family 2/ 3
  d3 y   d 2 y  dy
a  3 + 2 + 3 2  + =0
of curves given by y2 = a  x +  , a > 0 is .....
 2   dx   dx  dx
 2/ 3
JEE Main 26.02.2021, Shift-I  d3 y   d 2 y  dy 
 3 = −  2  × 3 + + 2
Ans. (2) : The given family of curves,  dx   dx  dx 
 a 2 3
y2 = a  x +  , a > 0 ....(i)  d3 y   d 2 y dy 
 2  3  = − 3 × 2 + + 2
   dx   dx dx 
Now, differentia tinned (i) with rasped to x, we get
So, the degree of differential equation is 2 and order is 3.
dy
2y =a .....(ii) 61. Find the sum of the order and degree of the
dx 3 2
Now substituting the value of form equation (ii) in the  dy  d y
differential equation y = x   + 2
equation (i) i.e.  dx  dx
dy (a) 2 (b) 3
Putting a = 2y
dx (c) 4 (d) 5
We obtain AP EAMCET-21.09.2020, Shift-I
 a Ans. (b) : Given, the differential equation,
y2 = a  x + 
 2
2
 dy  d y
3
  y = x  + 2
dy  1 dy   dx  dx
y 2 = 2y  x + 2y  1
dx  dx 
3
 dy   d y 
2
2 y = x  +  2  ⇒
 dx   dx 
dy   dy  
3/ 2

y = 2x +  2 y   We know that,
dx   dx   Order = higher derivative,
3/ 2 2 and, degree = degree of higher derivative,
dy    dy  
2

 y − 2x  =  2  y   Then, order = 2 and degree = 1
 dx    dx   So, order + degree = 2 + 1 = 3
2 3
dy  dy   dy  62. The order and degree of the differential
y 2 = 4xy + 4x 2   = 2y  
dx  dx   dx  dy dy
equation –4 – 7x = 0 are
From above steps we can see that the degree of the dx dx
differential equation is 3 and the order is 1. 1
So, the deference is 3 – 1=2 (a) , 1 (b) 2, 1
2
Differential Equation 1119 YCT
(c) 1, 1 (d) 1, 2 (a) 2 and 6
CG PET-2021 (b) 3 and 2
Ans. (d) : Given, (c) 2 and 3
The differential equation, (d) 2 and degree is undefined
GUJCET-2017
dy dy
− 4 − 7x = 0 Ans. (c) : Given, the differential equation –
dx dx 3
 d2y  dy
dy dy
= 4 + 7x  2  + 3 = x; x > 0
dx dx  dx  dx
On squaring both side we get- Then
2 Order = Highest derivative = 2
 dy   dy 
2
Degree = Power of highest derivative = 3
  =  4 + 7x 
 dx   dx  Hence order = 2, Degree = 3
Find the degree of ( y 2 ) − y1 = y3
2 2
dy  dy  dy 66.
= 16   + ( 7x ) + 56x
2

dx  dx  dx (a) 3 (b) 4
dy  dy 
2
dy (c) 1 (d) 2
= 16   + 4gx 2 + 56x 2
dx  
dx dx GUJCET-2007
Ans. (b): Given, (y 2 ) − y1 = y
2 2 3

[1 − 56x ] = 16   + 4gx 2
dy dy
dx  dx  d2 y dy
So, order = 1 and degree = 2 y2 = , y1 =
dx 2 dx
63. The degree of the differential equation Degree = Power of highest order derivative
5
  dy 2  3 (y 2 ) 2 = y3 + y1
5 d y 
2

5 +    = x  2  is Squaring both the side, we have–


  dx    dx 
(y 2 ) 4 = y6 + 2y1 2 y3 + y1
1

(a) 4 (b) 3
(c) 5 (d) 10 So, degree = 4
Manipal UGET-2017  dy 
67. The degree of the e x + sin   = 3 is ...........
Ans. (b) : Given,  dx 
2 5/ 3
  dy   (a) 0 (b) 1
5 d y
2
5 +
    = x  2 (c) 2 (d) degree is not defied
  dx    dx  GUJCET-2009
5 3 Ans. (d): Given,
  dy  2  15  d y 
2

5 +    = x  2   dy 
  dx    dx  e x + sin   = 3 …. (i)
 dx 
∴ degree = 3 Order = Highest order derivative
64. Order and degree of the differential equation Degree = Degree of highest order derivative
d2 y
 dy 
e = x are ______ respectively.
dx 2
Degree is not defined because   is in sine function.
(a) 2 and not defined (b) 1 and 2  dx 
(c) 2 and 1 (d) 1 and not defined 68. The order and degree of the differential
GUJCET-2021  d3y 
5
 d2y 
4

Ans. (a) : Given, equation  3  =  2  is :


4 3

d2 y  dx   dx 
e dx = x
2
(a) 2 and 16 (b) 3 and 15
Order = Highest order derivative (c) 3 and 16 (d) 2 and 12
Order = 2 and GUJCET-2023
d2 y Ans. (b) : Given, the differential equation,
2
Degree = not defined because it has a term e dx 5 4
 d 3y   d2 y 
65. The order and degree of the differential 4  3 = 3
 2
equation  dr   dx 
3 5/ 4 4/3
 d2y  dy  d 3y   d2 y 
 2  +3 = x;x > 0 are____respectively. ⇒  3 = 2
 dx  dx  dx   dx 
Differential Equation 1120 YCT
5
×12
4
×12 2 dy
 d3 y  4  d2 y  3 Put, a = y in equation (i), we get-
 3 = 2  5 dx
 dx   dx  3/ 2
2 dy  2 dy 
 d3 y 
15
 d2y 
16 y2 − 5 × y x − 5  y  =0
 3  = 2 
5 dx  5 dx 
 dx   dx  dy  2 dy 
3/ 2

So, order = 3 and degree = 15 y 2 − 2xy = 5 y 


dx  5 dx 
69. The differential equation of the family of
On squaring both side, we get-
1
curves y = ax + , where a (≠ 0) is an arbitrary  2 dy 
2
 2 dy 
3
a  y − 2xy  = 25  y 
constant, has the degree  dx   5 dx 
2 3
(a) 4 (b) 3  2 dy  8  dy 
(c) 1 (d) 2  y − 2xy  =  y 
 dx  5  dx 
TS EAMCET-2015 Then, Order (m) = 1
Ans. (d) : Given, the family of curves- degree(n) = 3
1 So, m–n=1–3=–2
y = ax + ..... (i)
a 71. If m and n are the order and degree of the
On differentiating both side, we get- differential equation of the family of parabolas
dy with focus at the origin and X-axis as its axis,
= a ⋅1 + 0 then mn – m + n =
dx
(a) 1 (b) 2
dy
a= (c) 3 (d) 4
dx TS EAMCET-04.05.2019, Shift-II
dy Ans. (c) : From question,
Put, a = in equation (i), we get-
dx Equation of family of parabolas with focus
dy 1 At, (0, 0) and x –axis is-
y= .x+
dx  dy  y2 = 4k(x + k) …..(i)
  On differentiating both side w.r.t.x, we get-
 dx 
2 dy
dy  dy  2y = 4k
y⋅ = x   +1 dx
dx  dx 
y dy
2 k=
 dy   dy  2 dx
x   − y  +1 = 0
 dx   dx  y dy
Put, k = in equation (i), we get-
So, the differential equation of the family of curves y 2 dx
1 dy  y dy 
= ax + has the degree 2. y2 = 2y  x +
a 
dx  2 dx 
70. If m and n are respectively the order and the 2
dy  dy 
degree of the differential equation representing y = 2x + y 
3 dx  dx 
the family of curves y 2 – 5ax – 5a 2 = 0 (a > 0 is Then, order (m) = 1, degree (n) = 2
a parameter), then the value of m – n is So, mn – m + n = 1 × 2 – 1 + 2 = 3
(a) 1 (b) –1 72. For the differential equation
(c) 2 (d) –2 2

TS EAMCET-19.07.2022, Shift-II d y 3  dy
2
 dy  
2
= y + x sin   
Ans. (d) : Given, the family of curves- dx  dx  dx  
y − 5ax − 5a = 0
2 3/ 2
…..(i) (a > 0) (a) Order is two degree is 3
(b) Order is 3 and degree is 3
On differentiating both side, we get-
(c) Order is 3 and degree is 2
dy
2y − 5a = 0 (d) Order is 2 and degree is not defined
dx TS EAMCET-18.07.2022, Shift-II
dy Ans. (d) : Given, the differential equation,
2y = 5a
dx 2
d2 y  dy  dy   n 2
a= y
2 dy = 3  y + x sin    ∑ X i
5 dx dx 2  dx  dx   i =1

Differential Equation 1121 YCT


1/ 2
 d2 y    dy  dy   
2/3 Ans. (d) : Given differential equation,
 2 =   y + x sin     3/ 2
 dx    d2 y  dy d3 y 
 dx    dx 2
+ y+ − 3  =0
1/ 2 2/3
dx  dx dx 
 d2 y   dy  dy  
 2 =  y + x sin     dy d 3 y 
3/ 2
 d2 y 
 dx   dx  dx    − 3 = − 2 + y 
3 4  dx dx   dx 
 d 2 y   dy  dy  
 2  =  y + x sin   
3 2
 dy d 3 y   d 2 y 
 dx   dx  dx    − 3  =  2 + y
 dx dx   dx 
So, order = 2
Clearly, order and degree is 3 and 3 respectively.
 dy 
and, degree is not defined because   is sine function. 75. If the order and degree of the differential
 dx  equation corresponding to the family of curves
73. If α and β are respectively the order and ( x – 2 ) + ( y – a ) = b2 , (where a and b are
2 2

degree of the differential equation for which


parameters) are m and n respectively, then
ax 2 – by 2 = 1 is the general solution, then the
m2 + n =
eccentricity of the ellipse ax + by = 1 is
2 2
(a) 7 (b) 5
1 1 (c) 4 (d) 3
(a) (b) TS EAMCET-14.09.2020, Shift-I
2 2
Ans. (b) : Given, ( x − 2) + ( y − a ) = b 2
2 2
1 1
(c) (d)
2 2 2 +1 On differentiating w.r.t. 'x' ...(i)
TS EAMCET-11.09.2020, Shift-II 2 ( x − 2 ) + 2 ( y − a ) y' = 0
Ans. (a) : Given the solution, Again differentiating w.r.t. 'x'
ax 2 + by 2 = 1 .....(i) 1 + ( y − a ) y"+ y '2 = 0 ...(ii)
Now differentiating equation (i) w.r.t. x, we get– From equation (i) and (ii), we get–
2ax + 2by
dy y  dy 
= 0 or   = −
a − ( x − 2 ) y"
1+ + y'2 = 0 ⇒ ( x − 2 ) y" = y'3 + y'
dx x  dx  b y'
Again differentiating w.r.t. x, we get Q Order = 2, degree = 1
 dy  2  d2 y   
dy  ∴ m2 + n = ( 2) + 1 = 5
2
x   + y  2  − y   
 dx   dx   dx   76. The degree of the differential equation
2
=0
x dy
2 2 5y +
 d2 y   dy   dy  d y dx is equal to
or, xy  2  + x   − y   = 0 2
=
 dx   dx   dx  dx d 2
y
This is the differential equation of order 2 and degree 1 dx 2
obtained from the solution (1). (a) 2 (b) 3
∴ Comparing with ax 2 + by 2 = 1 (c) 4 (d) 5/2
J&K CET-2011
a = 2, b = 1 ⇒ 2x 2 + y 2 = 1
Ans. (b) : Given, the differential equation-
x2 y2 1 2 dy
or, + =1 ⇒ a = , b =1
2
5y +
1 1 2 d2 y dx
  =
2 dx 2 d2y
Eccentricity, dx 2
a2 1/ 2 1/ 2 1 1 d2 y d2 y dy
e = 1− 2 = 1− = 1− = 1− ⇒ e = ⋅ = 5y +
b 1 1 2 2 dx 2
dx 2
dx
74. The order and degree of the differential On squaring both side, we get-
3/2 2 2
d2 y  dy d 3 y   d2 y   d2 y   dy 
equation + y +  – 3 = 0, are  2   2  =  5y + 
dx 2  dx dx   dx   dx   dx 
respectively. 3 2
 d2 y   dy 
(a) 3, 4 (b) 2, 2  2  =  5y + 
(c) 3, 2 (d) 3, 3  dx   dx 
TS EAMCET 14.09.2020, Shift-II So, degree = 3, order = 1.
Differential Equation 1122 YCT
77. For the differential equation  dy 
2
dy
5 or 16 ⋅   + (56x − 1) + 49 x 2 = 0
  dy   2 3
d2y  dx  dx
1 –    = 8 2 has the order and degree So, the order is 1 and degree is 2.
  dx   dx
80. If m is order and n is degree of the differential
______ respectively. equation
(a) 2 and 3 (b) 2 and 6 3
(c) 2 and 1 (d) 2 and 2  d2 y 
5  
 d2 y  dx 2  d3 y 
+ 4  3  +  3  = x 2 – 1, then
MHT CET-2022
 
Ans. (a) : Given that, the differential equation–  dx 
2
 d y   dx 
5  3
  dy  2  3 d2 y  dx 
1 −    = 8 2 (a) m = 3, n = 5 (b) m = 3, n = 3
  dx   dx (c) m = 3, n = 2 (d) m = 3, n = 1
  dy   2 5 3 MHT CET-2021
 d2 y 
Here, 1 −    = 83  2  Ans. (c) : The given differential equation can be written
  dx    dx  as–
∴ Order = 2 and degree = 3
5 3 2
78. The order and degree of the differential  d 3 y   d 2 y   d 2 y   d3 y   d3 y 
2  3  2⋅ + 4 ⋅  2   3  = (x − 1) ⋅  3 
+ 2

equation 3 2 =
dy dy
are respectively  dx   dx   dx   dx   dx 
dx dx ∴ The order of the differential equation is 3 and degree
(a) 3, 3 (b) 1, 3 is 2.
(c) 2, 3 (d) 2, 2 Hence, m = 3, n = 2
MHT CET-2021 81. The order and degree of the differential
Ans. (d) : Given, d2y  dy 
3/2
2
equation + x +   = 0 are
d2y dy dx 2  dx 
3 =
dx 2 dx respectively
2 3 (a) 2, 4 (b) 2, 3
 d 2 y   dy 
 2   = (c) 2, 2 (d) 3, 4
 dx   dx  (e) 4, 3
2 3 Kerala CEE-2020
 d 2 y   dy 
or,  2  −  = 0 Ans. (c) : Given that,
 dx   dx  3/ 2
Clearly, order is 2 and degree of the differential d2 y  dy 
+ x 2
+   =0
equation is 2. dx 2  dx 
79. The order and degree of the differential 3
 dy  2 d2 y
equation
dy
–4
dy
– 7x = 0 are respectively x 2
+   = −
dx dx  dx  dx 2
(a) 2 and 2 (b) 2 and 1 On squaring on both side–
(c) 1 and 1 (d) 1 and 2 3/ 2
 d2 y 
2
 dy 
MHT CET-2021 x 2 +   = − 2 
 dx   dx 
Ans. (d) : Given that,
3 2
 dy  2  d y 
2
dy dy
− 4 ⋅ − 7x = 0 x +  = 2
2

dx dx  dx   dx 
It can be written as, ∴ Order = 2 and degree = 2
dy dy 82. The degree of the differential equation
= 4⋅ + 7⋅x
dx dx 3
  dy 2  d2y2
On squaring both the sides, we get– 1 +    = l 2 is
dy  dy 
2
dy   dx   dx
= 16 ⋅   + 49 x 2 + 56 x ⋅
dx  dx  dx (a) 1 (b) 2
2 (c) 3 (d) 4
 dy   dy  dy
or 16 ⋅   −   + 56 x ⋅ + 49x 2 = 0 (e) 5
 dx   dx  dx Kerala CEE-2019
Differential Equation 1123 YCT
Ans. (b) : Given that, d 2 y dy dy
y. + . =0
3
dx 2 dx dx
  dy 2  2 d2 y
1 +    = 1 2 Hence, order and degree of the given equation is 2 and 1
  dx   dx respectively.
On squaring both sides, we get– 86. The order and degree of the differential
3 2 equation
  dy 2   d2 y 
1 +    =  2   d 2 y  dy  2 
3/2
 d3y 
  dx    dx  2 2 +    =  3  are respectively.
So, the degree of differential equation is 2.  dx  dx    dx 
83. The order and degree of the differential (a) 2 and 2 (b) 2 and 1
equation (y″′) + (y″) – (y′) + y = 0 is
2 3 4 5 (c) 3 and 2 (d) 3 and 3
(a) 3 and 2 (b) 1 and 2 (e) 2 and 4
(c) 2 and 3 (d) 1 and 4 Kerala CEE-2015
(e) 3 and 5 Ans. (c) : Given that the differential equation–
Kerala CEE-2018 3

Ans. (a) : Given equation  d 2


y  
dy 
2 2
d3 y
 2 2
+    =
(y''')2 + (y'')3 – (y')4 + y5 = 0  dx  dx   dx 3
Clearly, its order is 3 and degree is 2 . On squaring both sides, we get-
84. The order of the differential equation 3
 d 2 y  dy  2   d3 y 
2

2 2 +    =  3 
2 2 5
 d y   d y   dy 
3 2

 3  +  2  +   = 0 is  dx  dx    dx 

 dx   dx   dx 
Hence, order is 3 and degree is 2.
(a) 3 (b) 4
(c) 1 (d) 5 87. The order and degree of the differential
(e) 6 equation of the family of circles of fixed radius
r with centres on the y-axis are respectively
Kerala CEE-2017
(a) 2, 2 (b) 2, 3
Ans. (a) : Given,
2 2
(c) 1, 1 (d) 3, 1
5
 d 3 y   d 2 y   dy  (e) 1, 2
 3 + 2 +  =0
 dx   dx   dx  Kerala CEE-2013
3
d y Ans. (e) : The equation of family of circles of fixed
Since, the highest order derivative is 3 radius 'r' with centres on the y-axis is
dx
(x –0)2 + (y–a)2 = r2 …(i)
∴ Order of the given differential equation is 3.
On differentiating with respect to x, we have
85. The differential equation representing the
2 dy
family of curves y = a (ax + b), where a and b 2x + 2 ( y − a ) =0
are arbitrary constants, is of dx
(a) order 1, degree 1 dy x
=−
(b) order 1, degree 3 dx y−a
(c) order 2, degree 3
−x
(d) order 1, degree 4 (y − a) =
(e) order 2, degree 1  dy 
 
Kerala CEE-2016  dx 
Ans. (e) : Given that, On putting value in equation (i), we have-
2
y = a(ax + b) x2
2 2 x2 + 2
= r2
y = a x + ab  dy 
On differentiating with respect to x, we get–  
 dx 
dy
2y =a 2
  dy  
2
 dy 
2
dx x 2 1 +    = r 2  
dy a 2   dx    dx 
y = Hence, order = 1 and degree = 2
dx 2
Differential Equation 1124 YCT
88. The order and degree of the differential 90. The differential equation representing the
1 family of curve y2 = 2c(x + c3), where c is a
d y
3 2
dy 3
positive parameters, is of
equation  3  = 2 2 + 3 cos 2 x are
 dx  dx (a) order 1, degree 1
respectively (b) order 1, degree 2
(a) 3 and 1 (b) 3 and 3 (c) order 1, degree 3
(d) order 1, degree 4
(c) 1 and 3 (d) 3 and 2
(e) order 2, degree 1
(e) 2 and 2
Kerala CEE-2008
Kerala CEE-2012
Ans. (d) : We have,
Ans. (a) : Given that,
1
y 2 = 2c ( x + c3 )
 d3 y  3 d2y 3 On differentiating with respect to x, we have
 3  = 2 2 + cos x
2

 dx  dx dy
2y = 2c
On cubing both sides, we get dx
3
 d3 y   d2 y 3  dy
c=y
 3  =  2 2 + cos x 
2

dx dx dx
   
4
Hence, highest order derivation = 3 and degree = 1 dy  ydy 
∴ +2
y2 = 2yx

89. The degree and order of the differential dx  dx 
Hence, order is 1 and degree is 4
equation y = px +x 3 a 2p 2 + b 2 , where
91. The order and degree of the differential
p=
dy
,are respectively equation sin x ( dx + dy ) = cos x ( dx – dy ) is
dx
(a) (1, 2) (b) (2, 2)
(a) 3, 1 (b) 1, 3
(c) (1, 1) (d) (2, 2)
(c) 1, 1 (d) 3, 3 (e) (0, 1)
(e) 3, 2 Kerala CEE-2006
Kerala CEE-2010 Ans. (c) : Given differential equation,
Ans. (b) : Given that, sin x ( dx + dy ) = cos x ( dx − dy )
y = px + x a p + b
3 2 2 2
 dy   dy 
sin x 1 +  = cos x 1 − 
dy  dx   dx 
Where, p =
dx
1
dy
dx
( )
sin x − cos x = cos x − sin x
dy   dy   2 3
Now, y = x + x a 2   + b2  dy cos x − sin x
=
dx   dx   dx sin x − cos x
1
  dy  2 3 ∴ Order = 1, degree = 1
dy
y− x = x a 2   + b 2  92. Order and degree of a differential equation
dx   dx   1/4
d 2 y   dy  
2

 dy 
3
 2  dy   2 =     are
y +
dx 2   dx  
 y − dx x  = x a  dx  + b 
3 2

      (a) 4 abd 2 (b) 1 and 2


dy  dy 
2
 dy 
3
(c) 1 and 4 (d) 2 and 4
y3 − 3y 2 + 3y   − x 3  
dx  dx   dx  Manipal UGET-2015
  dy  2  Ans. (d) : Here the given differential equation can be
= x 3 a 2   + b2  written as
  dx   4
 d 2 y    dy  
2

∴  2  = y +   
3 3 2 2 3
(a – b) = a – 3a b + 3ab – b
Hence, highest order of derivative = 1  dx    dx  
The power of the highest order derivative = 3 Clearly, its order and degree are 2 and 4 respectively.

Differential Equation 1125 YCT


B. Integrating Factor, Solution and dy
It is form of + Py = Q
Formation of differential dx
–2
Equation (General, particular P= and Q = x 2 cos x
x
and separation of variable)
I.F = e ∫
Pdx

dy 2 ( 2 – 1)
x–y y –2
∫ dx 1
93. If + x
= 0, x, y > 0 y ( 1) =1 , then = e x = e –2log x = 2
dx 2 –1 x
y(2) is equal to : Now general solution –
(a) 2 + log2 3 (b) 2 + log2 2 y.I.F = ∫ Q.I.F + c
(c) 2 − log2 2 (d) 2 − log2 3
JEE Main-27.06.2022, Shift-I  1  1
y ⋅  2  = ∫ (x 2 cos x) 2 dx + c
Ans. (d) : Given, x  x
x–y
dy 2 (2 – 1) y y
+ = 0 , x, y > 0, y(1) = 1 = sin x + c
dx 2x – 1 x2 2
y = x sinx + cx2
dy –2x (2y – 1)
= 95. The solution for the differential equation
dx 2 y (2 x – 1) dy dx
2y 2x + = 0 is
∫ 2y – 1 dy = – ∫ 2x – 1 dx y x
1 1
y (a) + =c (b) log x.log y = c
1 2 ln 2 1 2x ln 2 y x
ln 2 ∫ 2 – 1 ln 2 ∫ 2 – 1
y
dy = – x
dx
(c) xy = c (d) x + y = c
1 1 Karnataka CET-2016, 2002
ln 2y – 1 = – ln 2 x – 1 + C Ans. (c) : Given that, differential equation
ln 2 ln 2
At x = 1, y = 1 dy dx
+ =0
Put given value, we get c = 0 y x
ln 2 y – 1 + ln 2x – 1 = 0 Now, integrating both sides, we get –
log y + log x = log c
1 log xy = log c
2y – 1 = x
2 +1 xy = c
At x=1 96. The general solution of the differential
1 dy y
2y = + 1 equation + = 3x is
3 dx x
4 c c
2y = (a) y = x − (b) y = x +
3 x x
So, c c
4 (c) y = x 2 − (d) y = x 2 +
y = log 2 x x
3 Karnataka CET-2014
y = log 2 4 – log 2 3 Ans. (d) : Given,
y = 2 – log23 dy y
+ = 3x
94. The general solution of the differential dx x
equation x2dy – 2xydx = x4 cosx dx is dy
(a) y = x2 sin x + c (b) y = sinx + cx2 The above equation is in the form of + Py = Q .
dx
(c) y = cosx + cx2 (d) y = x2 sinx + cx2
1
Karnataka CET-2020 P = and Q = 3x
Ans. (d) : Given differential equation – x
1
x2dy – 2xy dx = x4cos x dx ∫ dx
2 4 I.F = e x = e log x = x
x dy = 2xy dx + x cos x dx For general solution
x2 dy = (2xy + x4 cos x) dx
y.x = ∫ x.3x dx + c
dy x 4 cos x + 2xy
= 3x 3
dx x2 xy = +c
dy 2y 3
= x 2 cos x + xy = x + c
3
dx x
dy 2y c
– = x 2 cos x y = x2 +
dx x x
Differential Equation 1126 YCT
97. The general solution of the differential
( etan y ) −1 2
tan −1 y
equation 1 − x y .dx = y.dx + x.dy is
2 2 (a) x ⋅ e = +c
2
(a) sin ( xy ) = x + c ( xy ) + x = c
−1
= ( e tan )
(b) sin −1 −1 2
(b) e tan y y
+c
(c) sin ( x + c ) = xy (d) sin ( xy ) + x = c
(e )
2
tan −1 x
tan −1 y
Karnataka CET-2013 (c) x ⋅ e = +c
Ans. (c) : We have differential equation, 2
= ( e tan )
−1 −1 2

1 – x 2 y 2 dx = ydx + xdy (d) e tan y x


+c
MHT CET-2020
1 – (xy) 2 dx = d(xy) VITEEE-2016, 2013 / AIEEE-2003
d(xy) Ans. (a) : We have differential equation–
dx =
1 – (xy) 2 (1 + y2) + x − e tan y
−1

(
dy
dx
=0 )
Let, xy = t –1 dy
d(xy) = dt (1 + y2) = –(x – e tan y )
Integrating on both sides, dx
dx –1
dt (1 + y 2 ) + x = e tan y
∫ dx = 1 – t 2 dy
–1

x = sin–1t + c dx x e tan y
+ =
x = sin–1(xy) + c dy 1 + y 1 + y 2
2

(x + c) = sin–1(xy) dx
sin (x + c) = xy Comparing + Px = Q , we get –
dy
98. The general solution of –1
1 e tan y
dy
= 1 − x 2 − y 2 + x 2 y 2 is P= and Q =
dx 1+ y 2
1 + y2
I.F = e ∫
Pdy
(a) 2sin −1 y = x 1 − x 2 + sin −1 x + c
1
(b) cos −1 y = x cos −1 x + c ∫ 1+ y 2 dy −1  1 
=e = e tan y
Q ∫ 2
dy = tan −1 y 
1
(c) sin −1 y = sin −1 x + c  1+ y 
2 For general solution,
−1
(d) 2sin y = x 1 − y + c 2
x ⋅ (I.F) = ∫ Q ⋅ (I.F)dy + C
Karnataka CET-2011 e tan y
–1

Ans. (a) : We have, x.e tan –1 y


= ∫e tan –1 y
. dy
1 + y2
dy
= 1 – x 2 – y2 + x 2 y2 −1  Put, tan −1 y = t 
dx −1 e 2 tan y  
dy xe tan y
=∫ dy  1 dy = dt 
= (1 – x 2 ) – y 2 (1 – x 2 ) (1 + y 2 )  (1 + y 2 ) 
dx  
−1
dy xe tan y
= ∫ e 2 t dt
= (1 – x 2 )(1 – y 2 )
dx
dy −1 e 2t
= (1 – x 2 . 1 – y 2 xe tan y
= +C
dx 2
−1
Taking integration both sides, we get– −1 e 2 tan y
xe tan y
= +C
dy 2
∫ 1 – y2 = ∫ 1 – x dx
2

(e )
2
tan −1 y
tan –1 y
We know that , x.e +C=
2
x 2 a2 –1 x
∫ a – x dx = 2 a – x + 2 sin a + C
2 2 2
100. The differential equation of the circles having
their centres on the line y = 8 and touching the
x 1 C
sin −1 y = 1 − x 2 + sin −1 x + X-axis is
2 2 2
2   dy  
2

2sin –1 y = x 1 – x 2 + sin –1 x + C (a) ( y − 8 ) 1 −    = 64


  dx  
99. The general solution of the differential
  dy 2 
( y − 8)
2
equation (1 + y 2 ) + ( x – etan y ) 1 +    = 64
–1 dy (b)
= 0 is   dx  
dx
Differential Equation 1127 YCT
 dy  kt 2
(c) y 2 1 +  = 64 (c) 8πr 2 = kt + c (d) 4πr 2 = +c
 dx  2
  dy 2  MHT CET-2019
(d) ( y − 8 ) 1 +    = 64 Ans. (b) : We know that, surface area of balloon with
  dx   radius r is 4πr2.
MHT CET-2020 According to question,
Ans. (b) : Given that, the circle having their centre lie
on y = 8 and touching the x-axis.
d
dt
( 4 πr 2 ) = k
Thus, equation of circle is, dr
(x – h)2 + (y–8)2 = 82 ……(i) 4π ( 2r ) = k
Differentiating w.r.t x, we get– dt
8πr dr = kdt
dy Now integrating both side –
2(x – h) + 2(y – 8) =0
dx 8π∫ rdr = k ∫ dt
dy
(x – h) = – (y – 8) r2
dx 8π = kt + c
Squaring both side, we get– 2
2 4πr2 = kt + c
2 2  dy 
(x–h) = (y – 8)   103. The solution of the differential equation
 dx  dx 1 + x
Putting value in equation (i), we get– + = 0 is
2 dy 1– y
 dy 
(y – 8) 2   + (y – 8)2 = 64 1+ x
 dx  (a) ( x − 1) ( y + 1) = c (b) =c
1− y
  dy   2

(y – 8)2 1 +    = 64 1+ x
  dx   (c) (1 + x ) (1 − y ) = c (d) =c
  1+ y
101. The general solution of the differential MHT CET-2019
dy Ans. (b) : Given differential equation,
equation e dx = x is dx 1 + x
(a) y = x ( log x − 1) + c (b) x = y ( log x − 1) + c + =0
dy 1 – y
(c) x = y ( log x + 1) + c (d) y = x ( log x + 1) + c dx 1+ x
=–
MHT CET-2019 dy 1– y
Ans. (a) :We have differential equation– dx dy
dy + =0
e dx = x 1 + x 1 –y
Taking log on both sides, we get– Integrating both sides we get–
log (1 + x) – log (1 – y) = logc
dy
log e = log x 1+ x
dx =c
1− y
dy
= log x 104. The differential equation of all lines having
dx slope m, passing through origin is
dy = logx⋅dx
dy d2 y
Integrating on both sides, we get– (a) =m (b) =0
dx dx 2
∫ ∫
dy = log x ⋅ 1dx
dy dy
Apply integration by parts, (c) x =y (d) y = +c
dx dx


{ }
y = log x ∫ 1 ⋅ dx − ∫ ( log x )′ ∫ 1dx  dx

MHT CET-2019
Ans. (a) : Given, differential equation of all lines
1  having slope, m
y = x log x − ∫  ⋅ x  dx y = mx + c
x  Passing through (0, 0) origin
y = x log x – x + c 0=m×0+c
y = x(log x – 1) + c c=0
102. If r is the radius of spherical balloon at time t Then, y = mx + 0
and the surface area of balloon changes at a y = mx
constant rate k, then Differentiating on both sides w.r.t x, we get–
kt 2 dy
(a) πr 2 = +c (b) 4πr 2 = kt + c =m
2 dx
Differential Equation 1128 YCT
105. The differential equation of all lines making x2 + y2 + 2gx + 2fy +c = 0
intercept 3 on the X-axis is Having centre (–1, 2) ⇒ (– g, – f)
dy dy x2 + y2 + 2 × 1 × x + 2 × (–2) × y + c = 0
(a) x = 3y (b) ( x − 3) =y Equate of circle,
dx dx
x2 + y2 + 2x – 4y + c = 0
dy dy
(c) = x −3 (d) = x +3 108. The particular solution of the differential
dx dx equation x dy + 2y dx = 0, when x = 2, y = 1 is
MHT CET-2019
(a) xy = 4 (b) x 2 y = 4
Ans. (b) : Given, differential equation of straight line.
y = mx + c …..(i) (c) xy 2 = 4 (d) x 2 y 2 = 4
Making intercept 3 on the x axis then y = 0 MHT CET-2017
0 = 3m + c Ans. (b) : Given the differential equation,
c = –3m x dy + 2y dx = 0
∴ y = mx – 3m x dy = – 2y dx
Differentiating both side w.r.t. x, we get– dy –2dx
dy =
=m y x
dx Integrating both sides we get,
Putting the value of c and m in equation (i), we get - log y = – 2 log x + log c
y = mx + (– 3m) log y + 2log x = log c
y = m (x – 3) x2y = c
dy When, x = 2 and y = 1
y= (x – 3)
dx (2)2 × 1 = c
106. The particular solution of the differential c=4
 dy  Then, x2 y = 4
equation log   = x, when x = 0, y = 1, is 109. Differential equation of all the circles whose
 dx 
centers lie on X-axis is
(a) y = −e x + 2 (b) y = e x + 2 2
d 2 y  dy 
(c) y = e x (d) y = −e x (a) y 2 +   + 1 = 0
dx  dx 
MHT CET-2019, 2008 2
Ans. (c) : Given, differential equation,  dy 
(b) y   − 2y + 1 = 0
 dy   dx 
log   = x d 2 y  dy 
 dx  (c) y 2 +   = 0
Taking antilog on both sides, we get– dx  dx 
2
dy d 2 y  dy 
= ex (d) y 2 −   − 1 = 0
dx dx  dx 
dy = exdx MHT CET-2021
Integrating on both sides, we get– MHT CET-2011, 2010 / UPSEE-2009
∫ dy =x ∫ e dx
x
Ans. (a) : Given, differential equation where centre lie
y=e +c …(i) on x axis–
When, x = 0 and y = 1 x2 + y2 – 2ax = 0
1=e +c0 On differentiating both sides w.r.t. x, we get–
1=1+c dy
2x + 2y – 2a = 0
c=0 dx
Putting the value of c in equation (i) Again differentiating with respect to x, we get–
Then, y = ex 2
d 2 y  dy 
107. The general solution of the differential y +   = –1
equation of all circles having centre at dx 2  dx 
A ( –1, 2 ) is
2
d 2 y  dy 
y +   +1 = 0
(a) x 2 + y 2 − x + 2y + c = 0 dx 2  dx 
(b) x 2 + y 2 + x − 2y + c = 0 110. y = a sin ( logx ) + b cos ( logx ) , then the
(c) x + y − 2x + 4y + c = 0
2 2 differential equation without the parameter ‘a’
& ‘b’ is
(d) x 2 + y 2 + 2x − 4y + c = 0
d2 y dy
MHT CET-2019 (a) 2
+x + x2y = 0
dx dx
Ans. (d) : We have differential equation of all circle
x2 + y2 + 2gx + 2fy + c = 0 d2 y dy
(b) x 2 2 − x +y=0
Having centre (–1, 2) ⇒ (–g, –f) dx dx
Differential Equation 1129 YCT
d2 y dy Ans. (b) : Given that,
(c) x 2 2
+x +y=0 y = (tan–1x)2
dx dx
Differentiating w.r.t x, we get -
d2 y dy
(d) x 2 2 − x −y=0 dy 1
dx dx = 2 tan –1 x.
dx 1+ x2
MHT CET-2008 dy
Ans. (c) : We have– (1 + x 2 ) = 2 tan –1 x
dx
y = a sin (logx) + b cos (logx) Again differentiating w.r.t x we get–
Differentiating on both sides w.r.t x we get,
d2 y
+ (1 + x 2 ) 2 =
dy 2
dy 1 1 2x
= a cos(log x) + b(– sin(log x)). dx dx 1+ x2
dx x x
dy dy d2y
x = a cos(log x) – b sin(log x) 2x(1 + x 2 ) + (1 + x 2 ) 2 2 = 2
dx dx dx
2

2 (
1 + x 2 ) + 2x(1 + x 2 )
Again differentiating w.r.t x, we get– d y 2 dy
=2
dy 2
d y 1 1 dx dx
+ x 2 = –a sin(log x) – b cos ( log x ) 113. The solution of the differential equation
dx dx x x
d2 y dy dy
x2 2 + x = – ( a sin(log x) + b cos(log x) ) = 3x+y at x = y = 0 is
dx dx dx
2 (a) 3x + 3− y − 2 = 0 (b) 3x − 3− y − 2 = 0
2 d y dy
x +x +y=0 (c) 3x + 3− y + c = 0 (d) 3x + 3− y − c = 0
dx 2 dx
MHT CET-2005, 2004
111. Solution of the differential equation
Ans. (a) : We have –
(1 + y 2 ) tan-1 x dx + (1 + x2 ) 2y dy = 0 is dy
(a) 1 + x 2 1 + e 2y = c = 3x + y
dx
(b) ( tan −1 x ) + 2 log 1 + y 2 = c
2
dy
= 3x.3y
dx
(c) tan −1 x + log 1 + y 2 = c
dy
= 3x dx
(d) 1/2 ( tan –1 x ) + 2 log 1 + y 2 = c
2
3y
MHT CET-2006 Integrating on both side we get–
∫ 3 dy = ∫ 3 dx
–y x
Ans. (b) : We have differential equation–
2 –1 2
(1 + y )tan x dx + (1+ x ) 2y dy = 0
–3– y 3x
Separating the variable, we have – = +c
–1 log 3 log 3
tan x 2y
dx + dy = 0 3x + 3–y = c
1+ x2 1 + y2 When, x = y = 0
Integrating both side, we get– 30 + 3–0 = c
–1
2y tan x c=2
∫ 1 + y2 dy = – ∫ 1 + x 2 dx Then, 3x + 3–y – 2 = 0
114. The differential equation of all parabolas
log |1+y2| = – ( tan –1 x ) + c
1 2
whose axes are parallel to the axis of y, is
2
d3 y d3 y
= = −1
1
( tan x ) + log |1 + y |= c (a) 1 (b)
–1 2 2

2 dx 3 dx 3
(tan–1x)2 + 2 log |1+ y2| = c d3 y
(c) =0 (d) none of these
d2 y ( dx 3
112. If y = ( tan x ) , then 1 + x 2 ) + 2x ( x 2 + 1)
–1 2 2
SRM JEEE-2008
dx 2
Ans. (c) : The equation of a member of the family of
dy
= parabolas having axis parallel to y-axis is
dx y = Ax2 + Bx + c
(a) 3 (b) 2 Where, A, B and C are arbitrary constant.
 1+ x2   y − x2  Differentiating both side w.r.t. x, we get–
(c) log   (d) log   dy
 x   tan x  = 2Ax + B
MHT CET-2006 dx
Differential Equation 1130 YCT
d2 y (c) x(y – 1) (x + 1) + 2 = 0
= 2A (d) y(x + 1) – x = 0
dx 2 SRM JEEE-2012
d3 y Ans. (a) : Given,
=0
dx 3  dy  y −1
slope =   = 2
dy  
dx x +x
115. If = e-2y and y = 0 when x = 5, then the
dx dy dx
=
value of x for y = 3 is y −1 x2 + x
(a) e5 (b) e6 + 1 Integrating on both sides we get,
e6 + 9 dy dx
(c) (d) none of these
2 ∫ y −1 = ∫ x2 + x
SRM JEEE-2010
dy dx
Ans. (c) : Given differential equation,
dy
∫ y − 1 = ∫ x(x + 1)
= e −2 y
dx dy 1 1 
e2y dy = dx ∫ y − 1 = ∫  x − x + 1  dx
Integrating both sides, log (y – 1) = log x – log (x + 1) + log c
∫ e dy = ∫ dx
2y
 cx 
log (y – 1) = log  
e2 y  x +1
=x+c ....(i)
2 cx
When, x = 5, y = 0 y–1= ….(i)
x +1
e0 Since, curve passes through point (1, 0)
= 5+c 1.c
2 0–1=
1 = 10 + 2c 1+1
2c = –9 c=–2
9 Put the above value of 'c' in equation (i) we get -
c= − −2x
2 y–1=
Putting the value of 'c' in equation (i), we get - x +1
e2 y 9 (y – 1) (x + 1) = –2x
=x− or (y – 1) (x + 1) + 2x = 0
2 2
e6 e6 + 9 d2 y
When, y = 3 , we get
9
=x− ⇒ x= 118. The solution of the equation = e−2x is
2 2 2 dx 2
116. The general solution of a differential equation e−2x e−2x
(a) (b)+ cx + d
dy x 2 4 4
= is
dx y 2 1 −2 x e−4x
(c) e + cx 2 + d (d) + cx + d
(a) x – y3 = C
3
(b) x3 + y3 = C 4 4
2 2
(c) x + y = C (d) x2 – y2 = C SRM JEEE-2016
SRM JEEE-2009 AIEEE-2002
Ans. (a) : Given differential equation, Ans. (b) :Given, differential equation,
dy x 2 d2 y
= = e −2x
dx y 2 dx 2
y2dy = x2dx Integrating both sides we get,
Integrating both side we get, dy e −2x
= +C
y3 x 3 dx –2
= + C1 Again integrating both sides we get,
3 3
3 3
y – x = 3C1 e −2x
x3 – y3 = – 3C1 C = –3C1 y= + cx + d
(−2)(−2)
or x3 – y3 = C
117. The equation of the curve through the point e−2x
y= + cx + d
y –1 4
(1, 0) and whose slope is 2 is
x +x 119. The differential equation of the family of
(a) (y – 1) (x + 1) + 2x = 0 curves x2 + y2 – 2ay = 0, where a is an arbitrary
(b) 2x(y – 1) + x + 1 = 0 constant is
Differential Equation 1131 YCT
(a) 2(x2 – y2)y’ = xy (b) 2(x2 + y2) y’ = xy (c) y = log{k(y + 1)(e x + 1)}
(c) (x2 – y2)y’ = 2xy (d) (x2 + y2)y’ = 2xy
SRM JEEE-2015  ex + 1
(d) y = log  +k
Ans. (c) : Given,  y +1 
x2 + y2 – 2ay = 0 …(i)
Differentiating w.r.t.x we get, BITSAT-2019
dy dy Ans. (c) : Given that, (e x + 1)ydy = (y + 1)e x dx
2x + 2y − 2a =0
dx dx dx y y
= +
dy dy 1 + y (1 + y)e x
x + (y – a) =0
dx dx  y   1
 =  1 + 
x 2 + y 2  dy  x 2 + y2  dy  1 + y   e x 
x + y −  =0 Q a = 
 2y  dx  2y 
dx  y   e x + 1 
 y − x  dy
2 2 =  
x+  =0 dy  1 + y  e x 

 2y  dx  y   ex 
  dy =  x  dx
( x 2 − y2 ) dx
dy
= 2xy  1+ y   e +1
After integrating on both sides, we get -
dy
Q = y′ y ex
dx ∫ 1 + y dy = ∫ 1 + e x
dx
∴ ( x 2 − y2 ) y ' = 2xy
120. The equation of the curve passing through the y − log (1 + y) = log (1 + e x ) + log k
 1 Hence, y = log[k(1 + y)(1 + e x )]
point  a, –  and satisfying the differential
 a  122. The solutions of (x + y + 1)dy = dx are
dy  dy  (a) x + y + 2 = Ce y
equation y – x = a  y2 +  is
dx  dx 
(b) x + y + 4 = C log y
(a) (x + a)(1 + ay) = −4a 2 y
(c) log(x + y + 2) = Cy
(b) (x + a)(1 − ay) = 4a 2 y
(c) (x + a)(1 − ay) = −4a 2 y (d) log(x + y + 2) = C − y
(d) None of these BITSAT-2020
BITSAT-2019 Ans. (a) : We have differential equation –
dy  dy  (x + y + 1) dy = dx
Ans. (c) : We have, y − x = a  y2 + 
dx  dx  dx
= x + y +1
ydx – xdy = ay 2dx + ady dy
y(1 − ay)dx = (x + a)dy dx
– x = y +1 …..(i)
dx dy dy
− =0
x + a y(1 − ay) Comparing the equation we get –
On integrating we get, P = –1 and Q = y + 1
log(x + a) − log y + log(1 − ay) = log C
I.F = e ∫
Pdy

(a + x)(1 − ay)
= log C
I.F = e ∫
or log –1dy
y = e–y
(x + a)(1 − ay) = Cy Now general solution we get –
 1 x. I. F = ∫ Q.(I.F)dy + C
Since, the curve passes through  a, − 
 a x. e–y = ∫ e – y .(y + 1)dy + C
C
∴ 2a × (1 + 1) = − ⇒ C = −4a 2
 d 
a x. e–y = (y + 1) ∫ e – y dy – ∫  (y + 1).∫ e – y dy  dy
So, (x + a)(1 − ay) = −4a 2 y  dy 
121. The general solution of differential equation = – (y + 1) . e–y – ∫ (1)(–e – y )dy + C
(e x + 1)ydy = (y + 1)e xdx is
x e–y = – (y + 1)e–y – e–y + C
(a) (y + 1) = k(e x + 1) x e–y = C – e–y (y + 2)
(b) y + 1 = e x + 1 + k or x + y + 2 = Cey

Differential Equation 1132 YCT


123. The solution of differential equation 125. The solution to the differential equation
dy dy yf'(x) – y 2
– y tan x = –y 2sec x is = where f(x) is a given function
dx dx f(x)
(a) y −1 sec x = cot x + c (b) y −1 cos x = tan x + c is
−1
(a) f(x) = y(x + c) (b) f(x) = cxy
(c) y sec x = tan x + c (d) None of these (c) f(x) = c(x + y) (d) yf(x) = cx
BITSAT-2006 BITSAT-2013
Ans. (c) : We have differential equation, VITEEE-2013
Ans. (a) : We have,
dy
– y tan x = – y 2 sec x dy f '(x) y2
dx = y−
1 dy 1 dx f (x) f (x)
− tan x = − sec x, dy f '(x) y2
y 2 dx y − y=−
1 −1 dy dv dx f (x) f (x)
Let, = v; 2 = Dividing by y2, we get –
y y dx dx dy f '(x) 1
−dv y −2 − y −1 =−
∴ − v tan x = − sec x dx f (x) f (x)
dx dy f ′(x) 1
dv − y −2 +y =
+ v tan x = sec x dx f (x) f (x)
dx Put, y −1 = z
Here, P = tan x, Q = sec x dy dz
− y −2 =
Q v⋅IF = ∫ Q ⋅ IFdx + c dx dx
dz f ′(x) 1
− − (z) = −
I.F. = e ∫
tan xdx
= elog(sec x ) = sec x dx f (x) f (x)
v sec x = ∫ sec 2 xdx + c dz f ′(x) 1
+ z=
−1 dx f (x) f (x)
Hence, the solution is y sec x = tan x + c f '( x )
∫ dx
dy I. F. = e f ( x ) = elog f ( x ) = f (x)
124. What is the solution of + 2y = 1 satisfying ∴ The solution is
dx
1
y(0) = 0 ? z ( f (x) ) = ∫ ( f (x) ) dx + c
1− e −2x
1+ e −2x f (x)
(a) y = (b) y = y −1 ( f (x) ) = x + c
2 2
1 + ex f (x) = y(x + c)
(c) y = 1 + e x (d) y = 126. y = aemx + be–mx satisfies which of the following
2
differential equations :
BITSAT-2010
dy dy
Ans. (a) : Given, (a) + my = 0 (b) – my = 0
dx dx
dy
+ 2y = 1 d2 y d2 y
dx (c) 2
– m2 y = 0 (d) + m2 y = 0
dx dx 2
dy Karnataka CET-2002
= 1 − 2y
dx Ans. (c) : We have,
dy 1 y = aemx + be–mx
∫ 1 − 2y ∫ = dx −
2
log 1 − 2y = x + C Differentiating with respect to x, we get–
dy
At, x = 0, y = 0 = ame mx – bme – mx
dx
1 Again differentiating with respect to x, we get –
− log1 = 0 + C
2 d2 y
= am 2 e mx + bm 2e – mx
C=0 dx 2
−1 d2 y
log (1 − 2y ) = x = m 2 ( aemx + be – mx )
2 dx 2
log (1 – 2y) = – 2x d2 y
−2x = m2 y
1 − 2y = e dx 2
1 − e−2x d2 y
y= 2
– m2 y = 0
2 dx
Differential Equation 1133 YCT
127. The general solution of the differential
∫e (dy – dx) = ∫ dx + c
y–x
Integrating both sides,
dy 1 + cos2y
equation + = 0 is given by : Put y – x = t
dx 1 – cos2x
(a) tany + cosx = c (b) tany – cotx = c d(y – x) = dt
(c) tanx – coty = c (d) tanx + cotx = c ∴ ∫ e t dt = x + c
Karnataka CET-2004
Ans. (b) : We have differential equation– e y– x = x + c
dy 1 + cos 2y 130. The solution of
+ =0
dx 1 – cos 2x  dy   dy 
(x + y)2  x + y  = xy  1 +  is
We know that,  dx   dx 
cos2θ = 2 cos2θ – 1
1
cos 2θ = 1 – 2sin2θ (a) log(xy) = – +C
x+y
dy 2 cos 2 y
Then, + =0 x 1
dx 2 sin 2 x (b) log   = – +C
dy cos 2 y  y x+y
=− 2
dx sin x 1
(c) log(xy) = +C
dy dx x+y
+ =0
cos 2 y sin 2 x (d) None of these
Integrating on both side, we get– COMEDK-2014
dy dx Ans. (a) : Given that the differential equation is
∫ cos2 y + ∫ sin 2 x = 0  dy   dy 
(x + y)2  x dx + y  = xy  1 + dx 
∫ sec y dy + ∫ cosec 2 x = 0
2

tan y – cot x = c  dy  –2  dy 
( xy ) + y  = ( x + y ) 1 + 
–1
x
128. Solution of differential equation  dx   dx 
xdy - ydx = 0 represents Integrating on both sides we get –
(a) a rectangular hyperbola
–1  dy  –2  dy 
(b) parabola whose vertex is at origin ∫ (xy)  x dx + y  = ∫ (x + y) 1 + dx 
(c) straight line passing through origin
(d) a circle whose centre is origin We know that integral
Karnataka CET-2021 / AMU-2002 f '(x)
Ans. (c) :We have differential equation– ∫ f (x) dx = log(f (x)) + C and
x dy – y dx = 0
 dy 
x dy = y dx Put, (x + y) = t ⇒  1 +  = dt
dy dx  dx 
=
y x log(xy) = ∫ t −2 dt
Integrating both side, we get–
t −1
dy dx log(xy) = +c
∫ y =∫ x −1
log y = log x + log c (x + y) –1
log(xy) = +C
y = cx –1
Which is a straight line passing through the origin. –1
log(xy) = +C
dy x+y
129. The solution of – 1 = e x– y is
dx 131. The differential equation of y = aebx +c is
(a) e x + y + x = c (b) e –( x + y) + x = c
–( x + y) (a) y 2 = y1 + y (b) y 22 = y y1
(c) e = x+c (d) e – x + y = x + c
COMEDK-2011 (c) y12 = y y 2 (d) y 2 = y1 y 2
Ans. (d) : Given that the differential equation – COMEDK-2017
dy Ans. (c) : Given that the differential equation
– 1 = ex – y
dx y = ae bx + c …(i)
dy ex Differentiating equation (i) w.r.t. x, we get –
–1 = y
dx e y1 = a(ebx +c )(b) = b(ae bx + c ) = by
e y dy – e y dx = e x dx y1
b= …(ii)
e y–x
(dy – dx) = dx y

Differential Equation 1134 YCT


Again differentiating equation (ii) w.r.t. x, we get – Ans. (c) : Given that that differential equation –
y.y 2 – y1.y1 ydx + (x – y 2 )dy = 0
0=
y2 dx
y. y2 – y1 y1 = 0 y + x – y2 = 0
dy
y12 = y.y 2
dx x
dy + =y
132. Solve = e x–y + x 2e –y . dy y
dx dx
x3 This is of the form + Px = Q
(a) e = e + + c
y x
(b) e = e + 3
y x dy
3 1
(c) e y = e – x + e x (d) e y = 3x 3 + e x Where, P = ,Q = y
UPSEE-2006 / COMEDK-2018 y
dy
Ans. (a) : We have differential equation – ∫
I.F. = e ∫ = e y = elog y = y
Pdy

dy
= e x – y + x 2e – y ∴ The solution is given by
dx
dy xy = ∫ y.ydy + c
= e x e – y + x 2e– y
dx y3
xy = +c
= e (e + x )
dy –y x 2
3
dx 3
3xy – y = 3c
Integrating on both sides, we get–
135. The solution of the differential equation
∫ e dy = ∫ (e + x )dx
y x 2
dy 2yx 1
+ = is
(1 + x 2 )
3 2 2
x dx 1 + x
ey = e x + + c
3
133. Solution of the differential equation (a) y (1 + x 2 ) = C + tan −1 x
dy y
xy 3 = 1 – x 2 + y 2 – x 2 y 2 is (b) = C + tan −1 x
dx 1 + x2
(a) x 2 + y 2 + log x 2 (1 + y 2 ) = c (c) y log (1 + x2) = C + tan–1x
(d) y (1 + x2) = C + sin–1x
(b) x 2 + y 2 – log x 2 (1 + y 2 ) = c VITEEE-2012
(c) x 2 – y 2 + log x 2 (1 + y 2 ) = c Ans. (a) : Given differential equation,
(d) x 2 – y 2 – log x 2 (1 + y 2 ) = c dy 2yx 1
+ =
dx 1 + x 2 (1 + x 2 )2
COMEDK-2019
Ans. (b) : Given that the differential equation – 2x 1
Here, P = , Q=
3 dy + (1 + x2 )
2 2
xy =1– x + y – x y
2 2 2 2 1 x
dx
2x
∫ 2 dx log(1+ x 2 )
3 dy
= (1 − x 2 )(1 + y 2 ) Now, I.F. = e ∫ = e 1+ x = e
P dx
xy
dx
I.F. = 1 + x2
y3 1 − x2 Solution of differential equation is
dy = dx
1 + y2 x
y ⋅ IF = ∫ Q ⋅ IF dx + C
Integrating both sides, we get –
1– x 
y (1 + x 2 ) = ∫ . (1 + x 2 ) dx + C
3 2
y 1
∫ 1 + y2 dy = ∫  x  dx ( )
1 + x 2 2

 y  1 
y (1 + x 2 ) = ∫
1
∫  y – y  dy = ∫  – x  dx dx + C
+1 (1 + x 2 )
2
 x 
y2 1 x2
− log y + 1 = log x − + c1
2
y (1 + x 2 ) = tan −1 x + C
2 2 2
x + y – log x (1 + y ) = c
2 2 2 2 xdy
136. The integrating factor of − y = x4 − 3x is
134. Solution of the differential equation dx
(a) x (b) log x
y dx + (x – y2) dy = 0 is
(a) xy+y3=3c (b) xy–y3=3c 1
(c) (d) –x
(c) 3xy–y3=3c (d) None of these x
COMEDK-2020 VITEEE-2019
Differential Equation 1135 YCT
Ans. (c) : Given that the differential equation – Ans. (a) : Given that,
dy dv k
x − y = x 4 − 3x + v = −g
dx dt m
Dividing by x on both side, we get – Multiply by m/k on both side we get–
dy y m  dv k  m
− = x3 − 3
dx x  + v  = −g
k  dt m  k
It is form of
m dv mg
dy
+ Py = Q +v=−
dx k dt k
1 m dv mg
Here, P = and Q = x3 – 3 =− −v
x k dt k
dv k
Hence, I ⋅ F = e ∫ ∫ mg = −∫ m dt
Pdx

1 +v
= e ∫ x = e − log x
− dx
k
1  mg  kt
I⋅F = log  + v  = − + log c
x  k  m
137. The solution of the differential equation 1
Q ∫ dx = log x
{ } { 2 2 2
}
1 + x ( x + y ) dx + ( x + y ) − 1 y dy = 0is
2 x
 mg  kt
y2 1 2 log  + v  − log c = −
(a) x +2
+ (x + y ) = C
2 3/ 2  k  m
2 3 We know that,
y2 1
(b) x 2 − + ( x 2 + y 2 ) = C
1/ 2
a
log = log a − log b
3 2 b
y2 1 2
(c) x − + ( x + y ) = C
2 2 3/ 2  mg 
2 3  k +v kt
(d) None of these So, log e  =−
VITEEE-2019  c  m
 
Ans. (c) : Given that,
mg
{ } { }
1 + x ( x 2 + y 2 ) dx + ( x 2 + y 2 ) − 1 ydy = 0 k
+v
=e m

kt
(Q logba = x ⇒ a = bx)
Rearranging the equation, we have c
kt
mg
dx − y dy + ( x 2 + y 2 ) ( xdx + y dy ) = 0

+ v = ce m
k
1
( x + y ) ( 2xdx + 2ydy ) = 0
kt
dx − y dy + 2 2
v = ce

m

mg
2 k
2 2
Let x +y =t
2x dx + 2y dy = dt 139. If the I.F. of the differential equation
dy
Integrating both side, we get – + 5y = cos x is ∫ e Adx .then A =
2
y 1 dx
x − + ∫ t dt = C (a) 0 (b) 1
2 2
(c) 3 (d) 5
y 2 1 t 3/ 2
x− + × =C VITEEE-2018
2 2 3/ 2 Ans. (d) : Given that,
Putting the value of t = x2+y2 dy
y2 1 2 + 5y = cos x
x − + (x + y ) = C 2 3/ 2 dx
2 3 The integrating factor of the differential equation –
dv k dy
138. The solution of + v = − g is + Py = Q
dt m dx
−k

I.F. = e ∫
k
t
mg mg − m t Pdx
(a) v = ce − (b) v = c −
m
e
k k Here, P = 5
k k
− t mg t mg
(c) ve = c − (d) ve = c − ∴ I.F. = e ∫
m m 5dx
k k
VITEEE-2018 Hence, A = 5
Differential Equation 1136 YCT
140. The solution of the differential equation f ' ( xy )
dy y d ( xy ) = x dx
log x + = sin 2x is f ( xy )
dx x Integrating on both sides we get–
1
(a) y log | x |= C − cos x f ' ( xy )
2 ∫ d ( xy ) = ∫ x dx
1 f ( xy )
(b) y log | x |= C + cos 2x
2 x2
log  f ( xy )  = +C
1 2
(c) y log | x |= C − cos 2x
2
2
x x2
+C
1 f ( )xy = e 2
= e 2
⋅ eC
(d) xy log | x |= C − cos 2x x2
2
VITEEE-2017 f(xy) = k ⋅ e 2
(∴ ec = k)
Ans. (c) : Given that the differential equation – 142. The solution of the differential equation
dy
dy y = ( 4x + y + 1) , is
2
log x + = sin 2x
dx x dx
Dividing by log x in both the side, we get – (a) ( 4x + y + 1) = tan ( 2x + C )
dy y sin 2x (b) ( 4x + y + 1) = 2 tan ( 2x + C )
2
+ =
dx x log x log x
(c) ( 4x + y + 1) = 3 tan ( 2x + C )
3
dy
+ Py = Q (d) ( 4x + y + 1) = 2 tan ( 2x + C )
dx
Solution of the differential equation is VITEEE-2011
y.e ∫ Pd x ∫
= ∫ Q.e dx + C
Pd x Ans. (d) : We have differential equation,
dy
= ( 4x + y + 1)
2
1 sin 2x ...(i)
P= , Q= dx
x log x log x Put 4x + y + 1 = v
1 Differentiating both sides w.r.t. x, we get–

I.F = e ∫ = e x log x
Pd x dx
dy dv
log ( log x ) 4+ =
I.F = e dx dx
I.F = log | x | (Q elog x = x ) dy dv
dx dx
= −4
So, the solution of given differential equation, dv
sin 2x − 4 = v2 (QFrom eq.(i) )
y ⋅ log | x | = ∫ ⋅ log x dx + C dx
log x dv
cos 2x = v2 + 4
= ∫ sin 2x + C = − +C dx
2 dv
cos 2x = dx
∴ y ⋅ log | x |= C − v 2
+4
2 dv
1
y ⋅ log | x |= C − cos 2x
∫ v + 22 ∫
2
= dx
2 1 v
tan −1   = x + c
dy f ( xy ) 2 2
141. If x. + y = x. , then f(xy) is equal to
dx f' ( xy )  + y +1 
4x
tan −1   = 2x + c
x2
2 y2 / 2
 2 
(a) k.e (b) k.e 4x + y + 1 = 2 tan ( 2x + C )
xy
2
(c) k.e x (d) k.e 2 143. The general solution of the differential
VITEEE-2014 d2 y dy
equation 2
+2 + y = 2e 3x is given by
Ans. (a) : We have dx dx
dy f ( xy ) e3x
x⋅ + y = x⋅ (a) y = (c1 + c 2 x)e x +
dx f ' ( xy ) 8
e −3x
The equation can be written as, (b) y = (c1 + c 2 x)e − x +
f ( xy ) 8
d
i. e ( xy ) = x e3x
f ' ( xy )
−x
dx (c) y = (c1 + c 2 x)e +
8
Differential Equation 1137 YCT
e −3x On putting the value of (y – b) in equation (ii), we get
(d) y = (c1 + c 2 x)e x +
8   dy  2  dy
VITEEE-2007 1 +   
  dx   dx
d2 y dy x −a =  .........(v)
Ans. (c) : Given, + 2 + y = 2e3x d2 y
dx 2 dx
The auxiliary equation is, dx 2
Substituting the values of (x – a) & (y – b) in (i), we
D2 + 2D + 1 = 0 get
or m2 + 2m + 1 = 0
2 2
(m + 1) (m + 1) = 0   dy  2   dy  2   dy  2 
m = – 1, – 1 1 +      1 +   
i.e, repeated roots   dx    dx  +   dx   = r2
∴ Complementary function = (c1 + c2x)e–x d y
2 2
d y
2 2

Now Partial Integral (P.I.),  dx 2   2


1    dx 
= 2 .2e 3x   dy 2 
3 2
D + 2D + 1  d2 y 
⇒ 1 +    = r 2  2 
Putting the value of D = 3, we get–
  dx    dx 
1 2e3x e3x
P.I. = 2 .2e3x = = 145. The solution of (D2 + 16) y = cos4x is
3 + 2.3 + 1 16 8 x
y = C.F. + P.I. (a) Acos4x + Bsin 4x + sin 4x
e3x 8
y = ( c1 + c 2 x ) e − x + x
8 (b) Acos4x + Bsin 4x – sin 4x
144. The differential equation of the system of all 8
circles of radius r in the XY plane is x
(c) Acos4x + Bsin 4x + sin 4x
3 2 4
  dy  
2
 d2 y 
(a) 1 +    = r 2  2  x
  dx    dx  (d) Acos4x + Bsin 4x – sin 4x
4
2
  dy 3  d y
2 3 VITEEE-2006
(b) 1 +    = r 2  2  Ans. (a) : Given, differential equation ,
  dx    dx  (D2 + 16)y = cos 4x
3 Now the auxiliary equation is,
  dy  2 
2
 d2 y  m2 + 16 = 0 ⇒ m = ± 4
(c) 1 +    = r 2  2 
  dx    dx  ∴ Complementary function:–
2 3
= (A cos 4x + B sin 4x)
  dy  
3
2d y
2
(d) 1 +    = r  2  1
And, particular integral (P.I.) = 2 .cos 4x
  dx    dx  D + 16
VITEEE-2007 Differentiating we get–
Ans. (c) : The equation of the family of circles of 1 x
radius r is- But, cos ax = sin ax
2 2 2 D2 + a 2 2a
(x – a) + (y – b) = r .........(i)
x
Where a & b are arbitrary constants. ∴ P.I. = sin 4x
Since equation (i) contains two arbitrary constants, we 2 × 4
differentiate it two times & the differential equation x
will be second order. P.I. = sin 4x
8
Differentiating (i) w.r.t.x, we get ∴ y = Complementary function + Particular Integral
dy x
2(x − a) + 2(y − b) =0 y = A cos 4x + Bsin 4x + sin 4x
dx 8
dy
⇒ (x − a) + (y − b) =0 .........(ii) d2 y
dx 146. The particular integral of + 2y = x 2 is
Differentiating (ii) w.r.t.x, we get dx 2
2 (a) x 2 − 1 (b) x 2 + 1
d 2 y  dy 
1 + (y − b) 2 +   = 0 ..........(iii)
(c) ( x 2 − 1) (d) ( x 2 + 1)
1 1
dx  dx 
2
2 2
 dy  VITEEE-2006
1+  
 dx  Ans. (c) :Given,
⇒ (y − b) = − ..........(iv)
d2 y d2 y
+ 2y = x 2
dx 2 dx 2
Differential Equation 1138 YCT
 d  Ans. (c) : Given that,
(D 2
+ 2) y = x2
 D = dx  dy
(x + 2y3 ) =y
1 dx
Now, particular integral (P.I.) = 2 ⋅ x2
D +2 2y3 + x dx
=
−1 y dy
1 1  D2 
= ⋅x 2
= ⋅ 1 +  ⋅x
2
2y2 + x/y = dx/dy
 D2  2  2 
2 1 +  dx x
 2  – = 2y 2
dy y
We know that,
dx
D It is form of + Px = Q
Let, D1 = dy
2
P = –1/ y and Q = 2y 2
(1 + D1 )
−1
∴ = 1 − D1 + D12 − D13 + .......
1 1
∫ – dy log  
Q I. F = e ∫ = e y = e  y 
Pdy
1 D  D  
2 2 2
P.I. = 1 −  +  + ...... ⋅ x
2
1
2  2   2   I.F =
 y
1  2 D2 2  ∴ We know that,
P.I. =  x − ( x )
2 2  x.I.F = ∫ (Q × I.F)dy + C
1 2
P.I. =  x − 1 1  1
2 x. = ∫  2y 2 .  dy + C
147. The solution of the differential equation
y  y
2
dy x y
= 2e x–y + x 2e –y is = 2 +C
dx y 2
3
x 3 y + Cy =x
(a) e y = 2e x + + C 149. The solution of the differential equation
3
−3 dy y ( logy – logx + 1)
x = is
(b) e − y = 2e x + +C dx x
3 (a) x = ye Cy
(b) y = xeCy
3 Cx
x (c) x = ye (d) None of these
(c) e − y = 2e x + + C
3 UPSEE-2015
x 3 Ans. (d) : Given differential equation,
(d) e y = 2e− x + + C dy y(log y – log x + 1)
3 =
UPSEE-2016 dx x
Ans. (a) : We have differential equation, dy y
= ( log y / x + 1) .....(i)
dy dx x
= 2e + x e
x–y 2 –y
Let, y = vx
dx
dy dy dv
= 2e x .e – y + x 2e – y Then, = v+x
dx dx dx
dv
= ( 2e x . + x 2 ) e – y
dy v+x = v(log v + 1) [from equation (i)]
dx dx
e y dy = ( 2e x + x 2 ) dx v+x
dv
= v log v + v
dx
Integrating both side we get–
dv
= v log v
∫ e dy = ∫ (2e + x )dx
y x 2
x
dx
x3 Separating the variable and integrating,
ey = 2ex + +C dv dx
3
148. The solution of the differential equation ∫ v log v = ∫ x
( x + 2y 3 ) dy
dx
= y is
log(log v) = logx + log C
log (logv) = log Cx
4 logv = Cx
xy
(a) y3 + Cx = y (b) + xy = Cy log y/x = Cx
2 y/x = eCx
3 3
(c) y + Cy = x (d) x + 2y = y + C y = x eCx
UPSEE-2016
Differential Equation 1139 YCT
150. The solution of differential equation dy dv
= v+x
( x + y ) − 2xy dx = 0 is
2 2 dy dx dx
dy
(a) x2 + y2 = xc (b) x2 – y2 = xc Putting the value of in equation (i), we get–
(c) x2 + y2 = c (d) x2 – y2 = c dx
UPSEE-2014  dv 
x2  v + x  – vx 2 = 1 + cos v
Ans. (b) : We have differential equation,  dx 
dy dv
(x2 + y2) – 2xy =0 vx2 + x3 – vx 2 = 1 + cos v
dx dx
dy dv
2xy = x 2 + y2 x3 = 1 + cos v
dx dx
dy x 2 + y 2 dv
= 3
dx
= .....(i)
dx 2xy 1 + cos v x
Let, y = vx Integrating in both sides we get–
dv x 2 + v 2 x 2 1 1
Then,
dy
= v+x = [From (i)] 2 ∫ sec2 v / 2 = ∫ 3 dx
x
dx dx 2xvx –2
dv 1 + v 2 1 v x
v+x = tan × 2 = +C
dx 2v 2 2 –2
1 v 1
dv 1 + v 2 tan × 2 = – 2 + C
x = –v 2 2 2x
dx 2v
Put y = vx or v = y/x
dv 1 + v – 2v 2 2
x = tan y/2x = C – 2
1
dx 2v 2x
dv 1 – v 2 152. The solution of the differential equation
x =
dx 2v xdy − ydx = ( x 2 + y 2 )dx is
Separating the variable we get–
2v dx (a) y − x 2 + y 2 = Cx 2
∫1– v 2
dv = ∫x
– log (1 – v2) = log x + log c (b) y + x 2 + y 2 = Cx 2
– log (1 – v2) = log xc (c) y + x 2 + y 2 + Cx 2 = 0
 1 
log  2 
= log cx (d) None of these
1– v  VITEEE-2008
1 UPSEE-2006, 2012
= cx
1 – v2 Ans. (b) : Given,
Now, putting value v = y/x we get–
1 (
xdy – ydx = x 2 + y 2 dx)
= cx
y
1–  
2
( )
xdy = y + x 2 + y 2 dx
x
dy y + x 2 + y 2
x2 = ……(i)
= cx dx x
x 2 – y2 Now putting, y = vx
x2 – y2 = cx dy dv
= v+x
2 dy y
151. The solution of x − xy = 1 + cos is dx dx
dx x Putting in equation (i), we get–
y 1 y 1
(a) tan = C− 2 (b) tan = C + dv vx + x 2 + v 2 x 2
2x 2x x x v+x =
dx x
 y
(d) x = ( C + x ) tan
C
(c) cos   = 1 + 2 2 y/x
v+x
dv
= v + 1 + v2
x x dx
UPSEE-2013 dv
Ans. (a) : Given that, x = 1 + v2
dx
dy
x2 – xy = 1 + cos y / x …..(i) dv dx
dx =
Let, y = vx 1+ v 2 x

Differential Equation 1140 YCT


Integrating both sides, we get – Ans. (c) : We have differential equation,
dv dx dy 3(1 + x) 2 (1 + y 2 )
∫ 1 + v2 = ∫ x 4xy
dx
=
(1 + x 2 )
We know that, Now separating the variable, we get–
log | v + v 2 + 1 |= log x + log C 4y dy 3(1 + x) 2
=
log | v + v 2 + 1 |= log xC (1 + y ) x(1 + x 2 )
2

4y dy  3 6 
v + v 2 + 1 = xC = + 
1 + y2  x 1 + x 2 
Putting, v = y/x we get –
Now integrating both side, we get–
y y2 4y dy 3 6 
+ + 1 = xC
x x2 ∫ 1 + y2 = ∫  x + 1 + x 2 dx
y + x 2 + y2 = x 2C Let, 1 + y2 = t
153. The solution of the equation 2y dy = dt
 dy  dt 3 6 
sin −1   = x + y is 2∫ = ∫  +  dx
 dx  t  x 1 + x2 
(a) tan (x + y) + sec (x + y) = x + C 2 log t = 3log |x| + 6 tan–1x + c
(b) tan (x + y) – sec (x + y) = x + C 2log (1 + y2) = 3log |x| + 6tan–1x + c
(c) tan (x + y) + sec (x + y) + x + C = 0 155. If y2 = P(x) be a cubic polynomial, then
(d) None of the above d  d2y 
UPSEE-2012 2  y 3 2  is equal to
dx  dx 
Ans. (b) : We have,
(a) P''' (x) + P' (x) (b) P'' (x) P''' (x)
sin ( dy / dx ) = x + y
–1
(c) P(x)P''' (x) (d) constant
dy UPSEE-2010
= sin(x + y) ……(i)
dx Ans. (c) : Given,
Let, x+y=v y2 = P(x)
dy dv Differentiating w.r.t x, we get–
1+ =
dx dx dy
2y = P '(x)
From equation (i), we get– dx
dv dy P '(x)
– 1 = sin v =
dx dx 2y
Separating the variable, we get–
Again differentiating w.r.t x, we get–
dv
= dx  d 2 y  dy  2 
1 + sin v 2  y 2 +    = P ''(x)
(1 − sin v)  dx  dx  
dv = dx
(1 + sin v)(1 − sin v) 2
d2y  dy 
1 – sin v 2y + 2   = P"(x)
2
dv = dx dx 2  dx 
cos v
d y P"(x) – 2 ( dy / dx )
2 2
 1 sin v  =
 –  dv = dx
2
 cos v cos v 
2 dx 2 2y
2
(sec v – tanv sec v)dv = dx  P '(x) 
2

Integrating both sides, we get– P"(x) – 2  


tanv – sec v = x + C d2 y  2y 
=
Putting v = x + y we get– dx 2 2y
tan (x + y) – sec (x + y) = x + C Multiplying by y3 in both side we get–
154. The solution of differential equation 2
3d y
2
y3 P′′(x) 2y 3  P′(x) 
4xy
dy
=
3 ( 1 + x )
2
( 1 + y 2
) is y 
 dx 
2  =
2y
− 
2y  2y 

dx ( 1 + x 2
)  d 2 y  y 2 P"(x)  P '(x) 
2

(a) log (1 + y ) =log x + 2 tan x +constant y3  2  = – y2  


1  dx  2  2y 
(b) log (1 + y ) = 3log   + 6 tan x + constant
2 −1
Again differentiating with respect to x we get–
x
2 –1
d 3  d2y  2 d 2
(c) 2 log (1 + y ) = 3 log x + 6 tan x + constant
(d) None of the above 2 y 
dx  dx 2  2 dx
= {y P ''(x)} – 42 dxd {P '(x)}2
UPSEE-2011
Differential Equation 1141 YCT
d 3  d2 y  d 1 d d2 y
y  2  = [ P(x)P"(x)] – [ P '(x) ] = e−2x is
2
2 158. The solution of the equation
dx  dx  dx 2 dx dx 2
d 3  d2 y  (a) y =
1 −2x
+
cx
+d
2 y   = [P(x) P''(x) + P''(x) P'(x)] e
dx  dx 2  4 2
1
1
– 2P '(x)P ''(x) (b) y = e −2x + cx + d
2 4
d 3  d2 y  1 −2x
2 y  2  = P(x)P′′′(x) (c) y = e + cx 2 + d
dx  dx  4
156. The solution of differential equation x cos2 y dx 1 −2x
(d) y = e + cx 3 + d
= y cos2 x dy is 4
(a) x tan x – y tan y – log (sec x/ sec y) = c UPSEE-2007 / JCECE-2006
(b) y tan x – x tan x – log (sec x . sec y) = c Ans. (b) : Given,
(c) x tan x – y tan y + log (sec x . sec y) = c
d2 y
(d) None of the above = e –2x
UPSEE-2010 dx 2
Ans. (a) : The differential equation– On integrating both side, we get–
x cos2y dx = y cos2x dy dy e –2x
Separating the variable, we get– = +c
dx –2
x y Again integrating, we get–
dx = dy
cos 2 x cos 2 y e –2x
Integrating both side, we get– y= + cx + d
(–2)(–2)
∫ x sec x dx = ∫ ysec y dy
2 2
1
d y = e –2x + cx + d
x ∫ sec x dx – ∫ (x).∫ sec x dx = y ∫ sec y dy
2 2 2 4
dx 159. The differential equation of system of
d concentric circles with centre (1, 2) is:
– ∫ (y).∫ sec 2 y dy
dx dy
x tan x – log |sec x| = ytany – log|sec y| + c (a) ( x – 2 ) + ( y – 1) =0
dx
x tanx – y tan y = log (sec x/sec y) + c dy
x tan x – y tan y – log (secx/secy) = c (b) ( x – 1) + ( y – 2 ) =0
157. The solution of the differential equation dx
dy
dy
= y tan x − 2sin x is (c) ( x + 1) + ( y – 2 ) = 0
dx dx
dy
(a) y sin x = c + sin 2x (d) ( x + 2 ) + ( y – 1) = 0
1 dx
(b) y cos x = c + sin 2x UPSEE-2006
2
(c) y cos x = c – sin 2x Ans. (b) : We know that general equation of circle with
centre (h, k) and radius π
1
(d) y cos x = c + cos 2x (x – h)2 + (y – k)2 = π2
2 Then substituting we get–
UPSEE-2007
(x – 1)2 + (y – 2)2 = π2
Ans. (d) : We have differential equation– Differentiating the equation w. r. t x, we get –
dy
= y tan x – 2sin x 2(x – 1) + 2(y – 2)
dy
=0
dx dx
dy
– y tan x = –2sin x dy
dx (x – 1) + (y – 2) =0
dx
dy
Which is form of + Py = Q 160.The solution of the differential equation sec2x tan
dx y dx + sec2y tan x dy = 0 is:
P = – tanx and Q = – 2sinx
tan y
=c
I.F = e ∫ = e ∫
Pdx – tan xdx (a) tan y tan x = c (b)
= e – log sec x = cosx tan x
Now particular solution, we get–
tan 2 x
y cos x = ∫ cos x(–2sin x)dx + c (c) =c (d) None of these
tan y
y cosx = – ∫ sin 2x + c UPSEE-2004 / BCECE-2013
1 Ans. (a) : Given that the differential equation–
y cosx = cos 2x + c
2 sec2x tan y dx + sec2y tan x dy = 0

Differential Equation 1142 YCT


Separating the variable, we get– Integrating both side we get–
2
sec x sec y2 c
dx + dy = 0 2 y +2 x =c Let, a =
tan x tan y 2
differential we get– x+ y= a
Let tan x = t 163. The general solution of the differential
sec2x dx = dt equation (x + y)dx + xdy = 0 is
Integrating (a) x2 + y2 = C (b) 2x2 – y2 = C
1 2
(d) y2 + 2xy = C
∫ t dt = log t (c) x + 2xy = C
JCECE-2014
sec2 y Ans. (c) : We have differential equation,
And, ∫ tan y = log(tan y) (x + y)dx + xdy = 0
xdx + ydx + xdy = 0
sec 2 x sec 2 y
∫ tan x dx + ∫ tan y = 0 xdx + d(xy) = 0
Now integrating both side we get–
log (tanx) + log (tany) = logc
log (tanx. tany) = logc ∫2x dx + ∫ d(xy) = 0
tany ⋅ tanx = c x + 2xy = C
161. The integrating factor of the differential 164. The solution of the differential equation
equation dy
(x2 – yx2 ) + y 2 + xy 2 = 0 is
dy dx
+ 2ytanx = sinx is
dx x 1 1
(a) log   = + + C
1 2 y x y
(a) 2 tan x (b) sec x
2 2 y 1 1
(c) log sec x (d) sec x (b) log   = + + C
JCECE-2019 x x y
Ans. (d): Given, 1 1
dy (c) log ( xy ) = + + C
+ 2y tan x = sin x x y
dx 1 1
dy (d) log ( xy ) + + = C
Which is form of + Py = Q x y
dx
P = 2tan x and Q = sinx JCECE-2014 / BCECE-2015
Ans. (a) : Given,
Q I.F = e ∫ = e ∫
Pdx 2 tan x dx
dy
= e 2logsec x = elog sec x
2
(x 2 – yx 2 ) + y 2 + xy 2 = 0
dx
I.F= sec2x x2 (1 – y)dy + (y2 + xy2)dx = 0
162. Solution of the differential equation Separating the variable, we get–
 dy 
2
dy (1 – y)dy (1 + x)dx
x  + 2 xy + y = 0, is + =0
 dx  dx y2 x2
(a) x + y = a (b) x − y =a  1 1  1 1
 2 –  dy +  2 +  dx = 0
(c) x + y = a
2 2
(d) x + y = a
2
y y x x
JCECE-2005, 2016 /JCECE-2016 Integrating both side, we get–
Ans. (d) : Given, 1 1
− − log y − + log x = log C
2
 dy  dy y x
x   + 2 xy +y=0 1 1
 dx  dx log x – log y = + + C
It can be written as – x y
  dy  
2
x 1 1
dy
( y)
2
log   = + + C
 x    + 2 xy + =0
y x y
  dx   dx
2 165. Find the differential equation of curves
 dy  
 x dx  + y  = 0 y = Aex + Be–x for different values of A and B
   
d2 y d2 y
dy (a) − 2y = 0 (b) =y
x + y =0 dx 2 dx 2
dx
dy dx d2 y d2 y
= 4y + 3 +y=0
∫ y + x =0 (c)
dx 2
(d)
dx 2
JCECE-2009
Differential Equation 1143 YCT
Ans. (b) : Given, 2 2
y = Aex + Be–x (c) Ce–x+ e 2x + C (d) e − x + e 2x + C
3 3
Differentiating w.r.t x, we get– JCECE-2007
dy Ans. (a) : We have differential equation–
= Ae x – Be – x
dx dy
Again differentiating w.r.t x, we get– + y = 2e 2x
dx
d2 y dy
= Ae x – (–) B (e – x ) Which is of the form, + Py = Q
dx 2 dx
2x
d2 y P = 1, Q = 2e
= Ae x + Be – x
I.F = e ∫ = e∫ = e x
Pdx 1dx
dx 2
d2 y For general solution we get–
=y
dx 2 y(I.F) = ∫ I.F × Qdx + C
166. The general solution of
y2dx + (x2 – xy + y2)dy = 0 is ye x = ∫ e x .2e 2x dx + C
−1  x  ye x = 2 ∫ e3x dx
(a) tan  y  + log y + c = 0
 
e 3x
ye x = 2 ⋅ +C
x 3
(b) 2 tan −1   + log x + c = 0
y 2 2x 2
y= e + Ce–x = Ce − x + e 2x
( )
(c) log y + x 2 + y 2 + log y + c = 0
3 3
168. The solution of differential equation (x + y)(dx
– dy) = dx + dy is:
x
(d) sin −1   + log y + c = 0 (a) x – y = kex–y (b) x + y = kex+y
 
y (c) x + y = k(x – y) (d) x + y = kex – y
JCECE-2008 JCECE-2005 / BCECE-2017
Ans. (a) : Given, differential equation, Ans. (d) : We have differential equation–
y2 dx + (x2 – xy + y2)dy = 0 (x + y) (dx – dy) = dx + dy
dx –(x 2 – xy + y 2 ) dx + dy
= dx – dy =
dy y 2 x+y
Putting, x = vy d(x + y)
d(x – y) =
dx dv x+y
= v+ y
dy dy Integrating on both side, we get–
d(x + y)
dv −(v 2 y 2 − y 2 v + y 2 ) ∫ d(x – y) = ∫ ( x + y ) + C
v+ y = 2
dy y
2 2
x – y = log (x + y) + logC
dv y (− v + v − 1) x–y = log(x + y)⋅C
v+ y =
dy y2 ex–y = C. (x + y)
1
dv
v + y = −v2 + v − 1 x + y = ex−y
dy C
 1
dv
y = –v 2 – 1 x + y = kex–y Q k = 
dy  C
Separating the variable and integrating we get– dy
169. The differential equation x − y = x 3 has the
dv dy dx
∫ v2 + 1 = – ∫ y general solution
–1 (a) y – x3 = 2Cx (b) 2y – x3 = 2Cx
tan v = – logy + c (c) 2y + x2 = 2Cx (d) y + x2 = 2Cx
Putting, v = x/y we get– BCECE-2018
tan–1 (x/y) = –log y + c Ans. (b) : Given,
Hence, general solution is,
dy
tan–1(x/y) + log y + c = 0 x – y = x3
dx
dy dy y
167. If + y = 2e2x, then y is equal to − = x2
dx dx x
2 2 dy
(a) Ce–x + e 2x (b) (1+ x) e–x + e 2x + C Which is the form of + Py = Q
3 3 dx
Differential Equation 1144 YCT
Where, 172. The solution of x dy – ydx + x2 ex dx = 0 is :
2
P = –1/x and Q = x y x x x
1 (a) +e =c (b) +e =c
I.F = e ∫ = e ∫ x = e – log x =
Pdx – dx 1 x y
y
x (c) x + e = c (d) y +ex = c
Now general solution, we get– BCECE-2006
y ⋅ (I.F) = ∫ I.F × Qdx + C Ans. (a) : Given that, differential equation,
xdy – ydx + x2exdx = 0
1 1 2 xdy = ydx – x2 ex dx
x ∫x
y = x dx + C
dy y x 2 e x
= −
y x2 dx x x
= +C
x 23 dy y
2y = x + 2Cx − = − xe x
dx x
2y – x3 = 2Cx dy
This is the form of + Py = Q
dy  1 – y2  dx
+ 
 1 – x2 
170. The solution of = 0 is 1
dx   P = − , Q = − xe x
(a) tan–1x+cot–1 x = C (b) sin–1x+sin–1 y = C x
1
I.F = e ∫ x = e − log x
− dx
(c) sec–1x+cosec–1 x = C (d) None of the above
BCECE-2012, 2004
1
Ans. (b): Given, I. F =
x
dy  1 – y2  For general solution
+  2 
=0
dx 1– x  y ⋅ (I.F) = ∫ I.F × Qdx + c
Applying separation the variable we get–
1 1
x ∫
dy dx y⋅ = − xe x ⋅ dx + c
+ =0 x
2
1– y 1 – x2 y
Integrating both side we get– = −e x + c
sin–1y + sin–1x = C x
or, sin–1x + sin–1y = C y x
+e =c
x
171. The solution of 1 + x2 ( )
dy
dx
+ 2xy − 4x2 = 0 is 173. The differential equation of the family of
curves y = a cos (x + b) is :
(a) 3x(1 + y2) = 4y3 + c (b) 3y(1 + x2) = 4x3 + c
(c) 3x(1 – y2) = 4y3 + c (d) 3y(1 + y2) = 4x3 + c d2 y d2 y
(a) 2
−y=0 (b) +y=0
BCECE-2008 dx dx 2
Ans.(b): Given, differential equation, d2 y
(c) + 2y = 0 (d) none of these
(1 + x 2 ) dx
dy
+ 2xy − 4x 2 = 0 dx 2
BCECE-2003
dy 2xy 4x 2 Ans. (b) : We have differential equation
+ = y = a cos (x + b)
dx 1 + x 2 1 + x 2
Differentiating w.r.t x, we get–
dy
This is the form of + Py = Q dy
dx = −a sin ( x + b )
dx
2x 4x 2 Again differentiating w.r.t x, we get–
Where, P = and Q =
1+ x2 1 + x2 d2 y
2x
log (1+ x )
= −a cos ( x + b )
∫ 2 dx 2
dx 2
I.F = e 1+ x =e = 1 + x2
Now particular solution, d2 y
= −y
y ⋅ (I.F) = ∫ I.F × Qdx + c dx 2
d2 y
4x 2 +y=0
y.(1 + x 2 ) = ∫ (1 + x 2 ) dx + c dx 2
(1 + x 2 ) 174. The solution of the given differential equation
(1 + x ) y = ∫ 4x dx + c
2 2 dy
+ 2xy = y is :
dx
x3 2 2
(1 + x 2 )y = 4 +c (a) y = ce x − x (b) y = ce x − x
3 2

3y (1 + x 2 ) = 4x 3 + c (c) y = cex (d) y = ce − x


BCECE-2003
Differential Equation 1145 YCT
Ans. (a) : Given, (a) –4 <k <4 (b) k< –4, k>4
Differential equation, (c) k=0 or 4 (d) None of the above
dy SCRA-2009
+ 2xy = y Ans. (a) : The given differentiating equation is,
dx y''+ky'+4y=0
dy
+ ( 2x − 1) y = 0 d2 y dy
dx 2
+k + 4y = 0
dy dx dx
= (1 − 2x ) dx d2 d
y We that, = D2 & =D
Integrating on both sides, we get– dx 2 dx
dy So, we get:-
∫ y = ∫ (1 − 2x ) dx D 2 y + kDy + 4y = 0
D 2 + kD + 4 = 0 ......(i)
x2
logy = x –2 +c So, characteristic equation will be
2
2 b2 – 4ac
logy + (x – x) = logc = k 2 – 4.(4) .......(ii)
c Now, solution is given as:-
log = x 2 − x
y y = Aeax cos bx + Beax sin bx
2 2
c = ye x − x , y = ce x − x = eax (A cos bx + Bsin bx)
175. The differential equation of all circles through Since this is the standard solution of complex conjugate
the origin and having their centers on the x- root i.e. a ± bi
axis is And this roots come when characteristic equation is < 0
dy dy i.e. k2 – 16 < 0
(a) x2 – y2 = 2 (b) x2 – y2 = 2xy or k2 < 16
dx dx or –4 < k < 4
2 2 dy 2 2 dy So, range of k is (–4, 4)
(c) y – x = 2xy (d) y – x = 2
dx dx 177. The solution of the differential equation
AMU-2006 dy y
– = 1 is
Ans. (c) : General equation of circle is, dx x
x2 + y2 + 2gx + 2fy + c = 0 (a) x 2 log e x + y = c (b) x log e x + cx = y
Center lies on x axis and passes through origin.
Then centre of circle (–g, 0). (c) x 2 log e x − y = c (d) x log e x + y = cx
Equation of circle passing through origin and centre on CG PET- 2007
x axis is Ans. (b) : Given,
x2 + y2 + 2gx = 0 ….(i) Differential equation,
x 2 + y2 dy y
g=− − =1
2x dx x
Differentiating w.r.t x in (i), we get- dy
Which is the form of + Py = Q
dy dx
2x + 2y + 2g = 0
dx 1
P = − and Q = 1
Putting the value of g, we get- x
 ( x 2 + y2 ) 
−1
I.F = e ∫ = e ∫ x = e − log x =
dy Pdx dx 1
2x + 2y + 2 − =0 x
dx  2x 
 For general solution, we get -
dy ( + ) =0 y ⋅ I.F = ∫ I.F × Qdx + c
2 2
x y
2x + 2y −
dx x 1 1
Multiplied by 'x' both sides, y. = ∫ .dx + c
x x
dy
2x 2 + 2xy = x 2 + y2 y
= logx + c
dx x
dy y = x log x + xc
x 2 + 2xy = y2
dx x log e x + cx = y
dy
y − x = 2xy
2 2
178. The general solution of the differential
dx equation y dx + (1 + x2) tan−1 x dy = 0, is
176. The differential equation y''+ky'+4y=0 has (a) y tan−1 x = c (b) x tan−1 y = c
solution of the form y=Aeax cos bx+Beaxsin bx −1
(c) y+ tan x = c (d) x + tan−1 y = c
for all values of k, if CG PET- 2007
Differential Equation 1146 YCT
Ans. (a) : Given, (x –h)2 + (y – 0)2 = h2
Differentiating equation is, x2 + h2 – 2xh + y2 = h2
ydx + (1 + x2) tan–1xdy = 0 x2 + y2 – 2xh = 0 ….(i)
Separating the variable, we get - Which is equation of circle touching y axis at origin.
dx dy Differentiating with respect to x, we get -
+ =0 dy
(1 + x ) tan x y
2 −1
2x + 2y
dx
– 2h = 0
On integrating both side, we get -
dy
dx dy h=x+y
∫ (1 + x 2 ) tan −1 x + ∫ y = 0 dx
Putting value of (h) in equation (i), we get -
Let, tan–1x = t  dy 
1 dt x2 + y2 – 2x  x + y  = 0
=  dx 
1 + x 2 dx dy
dx
2 2 2
x + y – 2x – 2xy =0
= dt dx
1 + x2
Then, dy
y2 – x2 – 2xy =0
dt dy dx
∴ ∫ t = −∫ y dy
2xy + x2 = y2
log t = –log y + log c dx
log(tan–1x) + log y = log c  ′ dy 
log(tan–1x⋅y) = log c 2xyy′ + x2 = y2 Q y = 
y tan–1x = c  dx 
179. The differential equation whose solution 181. The solution of the differential equation
represents the family y = ae3x + bex is given by dy y
x = y – xtan   is
d2 y dy d2 y dy dx x
(a) − 4 − 3y = 0 (b) + 4 − 3y = 0
dx 2 dx dx 2 dx
d2 y
x
(c)
dy
− 4 + 3y = 0 (d) None of the above (a) x sin  +C = 0 (b) x sin y + C = 0
dx 2 dx  y
CG PET- 2011
y
Ans. (c) : Given, (c) x sin   = C (d) None of the above
Differential equation, x
y = ae3x + bex CG PET- 2013
Differentiating with respect to x, we get - Ans. (c) : Given,
dy Differential equation,
= 3a.e3x + bex
dx dy  y
x = y − x tan  
Again differentiating w.r. t x, we get- dx x
d2 y dy y y
= 9a.e3x + bex = − tan   ….(i)
dx 2 dx x x
d2 y Let,
= 12ae 3x + 4be x − 3ae 3x − 3be x y = vx
dx 2
d2 y dy dv
= 4(a.3e3x + bex) – 3(ae3x + bex) = v+x
dx 2 dx dx
Then from equation (i), we get -
d2 y dy
= 4 − 3y dv vx  vx 
dx 2 dx v+x = –tan  
dx x  x 
d2 y dy
2
−4 + 3y = 0 v+x
dv
= v − tan v
dx dx dx
180. The differential equation of the family of
dv
circles touching the y-axis at the origin is x = − tan v
(a) xy' − 2y = 0 (b) y'' − 4y' + 4y = 0 dx
(c) 2xyy' + x2 = y2 (d) 2yy' + y2 = x2 dv dx
CG PET- 2013 =−
tan v x
Ans. (c) : The equation of family of circle is, Integrating both side, we get -
(x –h)2 + (y – k)2 = r2
dx
Given, the system of circle touching y axis at the origin
then centre (h, 0) and radius be h . ∫ cot vdv = ∫ − x
Then equation of circle,
Differential Equation 1147 YCT
cos v dx  
(a) y ( log e x + Cx ) = 1
x
∫ sin v dv = − ∫ x
(b)  log + Cx  x = y
 e 
Let, sin v = t
(c) ( log Cx 2 + ex 2 ) y = x (d) None of these
dt
cos v = CG PET- 2015
dv
cos v dv = dt Ans. (d) : Given,
1 dx Differential equation is
∴ ∫ t dt = − ∫ x (y log x−1) ydx = xdy
log t = –log x + log C dy ( y log x − 1) y
=
log sin v + log x = log C dx x
log (sin v. x) =log C dy y 2 log x y
x sin v = C = −
dx x x
y
Putting y = vx or v = , we get - dy y y 2 log x
x + =
dx x x
y
x sin   = C 1 dy y −1 log x
x + = ...(i)
y 2 dx x x
182. The solution of the differential equation dy dv
dy Put, y −1 = v ⇒ − y −2 =
a+x + xy = 0 is dx dx
dx dy dv
2
( 2a − x ) x + a
2
(a − x ) x +a y −2 =−
(a) y = Ce 3
(b) y = Ce 3 dx dx
2 2
From equation (i), we have–
( 2a + x ) x + a ( 2a + x ) x + a
(c) y = Ce 3 (d) y = Ce 3 dv v log x
− + =
CG PET- 2015 dx x x
Ans. (a) : Given, dv v log x
− =− ...(ii)
Differential equation is, dx x x
dy This is linear differential equation.
a+x + xy = 0  1
dx ∫  − dx 1
Here, IF=e  x  = e − log x =
dy x
a+x = − xy So, solution is,
dx
dy −x v × IF= ∫ IF.Qdx + C
= dx
y a+x 1 1  log x 
On integrating both sides, we get. v⋅ = ∫ −  dx + C
x x x 
 −x − a a 
log y – log C= ∫  +  dx
1 log x
v ⋅ = − ∫ 2 dx + C
 a+x a+x x x
y 1   −1  1 1 
log   = − ∫ x + a dx + a ∫ dx = −  log x   + ∫ ⋅ dx  + C
 
C a+x   x  x x 
( x + a) ( a + x)
3/ 2 1/ 2
1 log x 1  1
=− +a = + +C Q v = y 
3/ 2 1/ 2 x⋅y x x  
y 2 1 = y [ log x + 1 + Cx ]
log   = − ( x + a ) x + a + 2a x + a
C 3
1 = y [ log x + log e + Cx ] [Q1 = log e]
 
y  x a 
log   = 2 x + a  − − + a  1 = y [ log e ⋅ x + Cx ]
C  3 3 
184. The differential equation representing the
2 x
= 2 x+a  a−  family of parabolas having vertex at origin and
3 3 axis along positive direction of x-axis is
y 2 dy dy
log = ( 2a − x ) x + a (a) y 2 − 2xy =0 (b) y 2 + 2xy =0
C 3 dx dx
y
2
( 2a − x ) x + a d2y d2 y
= e3 (c) y 2 − 2xy 2 = 0 (d) y 2 + 2xy 2 = 0
C dx dx
2
( 2a − x ) x + a AMU-2010
y = C ⋅ e3 Ans. (a) : Differential equation representing the family
183. The solution of differential equation (y log x − of parabola having vertex at origin is
1) ydx = xdy is y2 = 4ax ….(i)
Differential Equation 1148 YCT
Differentiating equation (i) w.r.t.x, we get - dy
y − ay 2 = ( a + x )
dy dy y2
2y = 4a ⇒ 2y = 4× Q from equation ( i ) dx
dx dx 4x dx 1
= dy
dy y 2 a + x y − ay 2
2y =
dx x 1 1
dy ∫ a+x
dx = ∫
y − ay 2
dy
2yx = y2
dx 1 1
dy ∫ a + x dx = ∫ y (1 − ay ) dy
2
y – 2yx =0
dx
1 1 a 
185. The differential equation representing the ∫ a + x dx = ∫  y + 1 − ay  dy
family of curves y = b sin ( x + a ) , where a, b are
Let, 1 – ay = t
arbitrary constants is –ady = dt
d2 y d2 y 1 1 dt
+y=0 −y=0
(a)
dx 2
(b)
dx 2 ∫ a + x dx = ∫ y dy − ∫ t
dy log(a + x) + logC = logy – log (1 – ay)
(c) +y=0 (d) none of these
dx log y = log (x + a) + log C + log(1 – ay)
AMU-2010 loy y = log[C(x + a) (1 – ay)]
Ans. (a) : The given family of curve is, y = C (x + a) (1 – ay)
y = b sin (x + a) … (i) 188. The solution of differential equation
Differentiating (i) with respect to x, we get
dy
( 2y − 1 ) dx − ( 2x + 3 ) dy = 0 will be
= b cos(x + a) …(ii) 2x − 1 2y + 1
dx (a) =C (b) =C
Differentiating (ii) with respect to x, we get 2y + 3 2x − 3
d2 y 2x + 3 2x − 1
= −b sin(x + a) (c) =C (d) =C
dx 2 2y − 1 2y − 1
d2 y CG PET- 2016
2
= –y Ans. (c) : Given, Differential equation is,
dx
d2 y
( 2y − 1) dx − ( 2x + 3) dy = 0
+ y = 0 ( 2y − 1) dx = ( 2x + 3) dy
dx 2
186. The general solution of the differential  1   1 
( )
equation y dx + x + 2y 2 dy = 0 is   dx =   dy
 2x + 3   2y − 1 
(a) xy + y 2 = c (b) 3xy + y 2 = c 2 2
(c) xy + y = c
3
(d) 3xy + 2y = c
3 ∫ 2x + 3
dx = ∫
2y −1
dy

AMU-2010 log ( 2x + 3) = log ( 2y − 1) + log C


Ans. (d) : The given differential equation is, 2x + 3
ydx + (x + 2y2)dy = 0 =C
2
ydx + xdy + 2y dy = 0 2y − 1
d (xy) + 2y2dy = 0 189. The solution of the differential equation
Integrating both sides, we get - dy xlogx 2 + x
2y 3 = will be
xy + = constant dx siny + ycosy
3
3
(a) y sin y = x2log x +C
3xy + 2y = C (b) y sin y = x2 +C
187. The solution of the differential equation (c) y sin y = x2 + log x + C
(d) y sin y = x log x +C
dy  2 dy  CG PET- 2016
y−x = ay +  is
dx  dx  Ans. (a) : Given,
(a) y = C ( x + a )(1 − ay ) (b) y = C ( x + a )(1 + ay ) dy x log x 2 + x
=
(c) y = C ( x − a )(1 − ay ) (d) None of the above dx sin y + y cos y
CG PET- 2016 (sin y + y cos y)dy = (x log x2 + x)dx
(sin y + y cos y)dy = (2x log x + x)dx
Ans. (a) : Given, d(y sin y) = d(x2 log x)
dy  dy  Integrating on both sides,
y−x = a  y2 + 
dx  dx  ysin y = x 2 log x + C

Differential Equation 1149 YCT


190. If the solution of the differential equation v + v 2 + 16 = xc
+ e ( x – 2 ) y = ( x 2 – 2x )( x 2 – 2 ) e2x satisfies
dy x 2
y
dx Put v = , we get
y(0) = 0, then the value of y(2) is ______. x
(a) –1 (b) 1 (c) 0 (d) e y y2
+ + 16 = xc
JEE Main-26.06.2022, Shift-II x x2
Ans. (c) : Given,
y + y 2 + 16x 2 = cx 2 .....(ii)
dy x ( 2
Q + e x − 2 ) y = ( x − 2x )( x − 2 ) e
2 2 2x
Given, y (1) = 3 then x = 1 they y = 3
dx From equation (ii),
∫ e x ( x 2 − 2 ) dx ( x 2 − 2x )e x
Here, I.F. = e =e 3 + 9 + 16 × 12 = c(1) 2
∴ Solution of the differential equation is 3 + 25 = c
= ∫ ( x 2 − 2x )( x 2 − 2 ) e 2x ⋅ e
( x 2 − 2x )e x ( x 2 − 2x )e x
y⋅e dx c=8
We find y(2) then, x = 2
= ∫ ( x 2 − 2x ) e x ⋅ ( x 2 − 2 ) e x ⋅ e
( x 2 − 2x )e x
dx From equation (ii),
Let, (x2 – 2x)ex = t y + y 2 + 16 × 4 = 8 × 4
∴ (x2 – 2)exdx = dt
( x 2 − 2x )e x y 2 + 64 = 32 − y
y⋅e = ∫ t ⋅ e t dt Squaring both side we get
y2 + 64 = (32 – y)2
y ⋅ e x − 2x e = ( x 2 − 2x − 1) e x − 2x e + c
( 2 ) x ( 2 ) x
y2 + 64 = 32 × 32 + y2 – 64y
∴ y(0) = 0 64 (y + 1) = 32 × 32
∴c=1 y + 1 = 16
∴ y = ( x 2 − 2x − 1) + e 2x − x e
( 2) x
y = 15
192. What is the solution of the differential equation
∴ y(2) = –1 + 1 = 0
191. Let the solution curve of the differential ( x + 2y 3 ) dydx
= y?
equation
dy (a) x = y 3 + cy (b) x 2 = y 3 + cy 2
x – y = y 2 + 16x 2 , y (1) = 3 be y = y (x).
dx (c) x 2 = y 3 + cy (d) None of the above
Then y (2) is equal to : SCRA-2010
(a) 15 (b) 11 (c) 13 (d) 17 Ans. (a) : Given,
JEE Main-29.06.2022, Shift-I
Ans. (a) : Differential equation is, ( x + 2y3 ) dx dy
=y
dy
x − y = y 2 + 16x 2 dx x + 2y3 x 2y 3
dx = = +
dy y y y
dy
x = y + y 2 + 16x 2 dx x
dx − = 2y 2
(This is linear differential equation)
dy y
dy y + y + 16x
2 2
= dx
….(i) + Px = Q
dx x dy
Let, y = vx 1
Differentiating w.r.t.x, we get - Where, P = − , Q = 2y 2
y
dy dv
= v+x x ( I.F.) = ∫ Q ( I.F.) dy + c -------(i)
dx dx
dy
Then equation (i) becomes- −∫
so, I.F. = e ∫ = e y
pdy

dv vx + v x + 16x
2 2 2
−1
v+x = I.F. = e − log y = elog y
dx x
1
dv I.F. =
v+x = v + v 2 + 16 y
dx
So, from equation (i) :-
dv
x = v 2 + 16 x 1
dx = ∫ 2y 2 , dy + c = 2 ∫ ydy + c
Integration on both side, we get - y y
dv dx x y  2

∫ v2 + 16 = ∫ x y
= 2   + c = y2 + c
2
log v + v 2 + 16 = log x + logc x = y 3 + cy

Differential Equation 1150 YCT


193. Let y = y(x) be a solution curve of the
Ans. (a) : x (1 − x 2 ) + ( 3x 2 y − y − 4x 3 ) = 0
dy
differential (1 − x2y2) dx = ydx + xdy.
dx
If the line x = 1 intersects the curve y = y(x) at y
( ) dx + ( 3x 2 − 1) y = 4x 3
= 2 and the line x = 2 intersects the curve y = x 1 − x 2
y(x) at y = α, then a value of α is
dy

3e 2 3e 2 dy ( 3x − 1)
2
(a) (b) 4x 3
2(3e 2 − 1) 2(3e 2 + 1) + y =
dx ( x − x 3 ) ( x − x3 )
1 − 3e 2 1 + 3e 2
(c) (d) dy
2(3e 2 + 1) 2(3e 2 − 1) + Py = Q
dx
JEE Main-11.04.2023, Shift-I 3x 2 −1
∫ x − x3 dx
IF = e ∫
Pdx
Ans. (d) : (1 − x y ) dx = ydx + x dy, y (1) = 2
2 2
=e
y(2) = α = ? ∫
− dt

d(xy) x − x 3 = t ⇒ IF = e t

dx = 1
1 − (xy) 2 = e − ln t =
d(xy) t
∫ dx =∫ 1 − (xy) 2
∴ IF =
1
x − x3
1 1 + xy
x= ln +C y × IF = ∫ Q × IF dx
2 1 − xy
Put x = 1 and y = 2  1  4x 3 1 4x 3
y 3 
= ∫ x − x3 ( x − x3 )
× dx = ∫ x − x 3 2 dx
1 1+ 2
1 = ln +C
x−x  ( )
2 1− 2 4x
=∫ dx 1 − x2 = K
1
C = 1 − ln 3
2
(1 − x )
2 2

Now put x = 2 and y = α −dK


= 2∫ 2 −2x dx = dK
1 1 + 2α 1 K
2 = ln + 1 − ln 3
2 1 − 2α 2
 1
1 1 1 + 2α = −2  −  + c
1 + ln 3 = ln  K
2 2 1 − 2α
y 2
 1 + 2α  = +c
2 + ln 3 = ln   x − x 3
K
 1 − 2α 
y 2
1 + 2α = +c
= 3e 2
x−x 3
1 − x2
1 − 2α At x = 2, y = −2
1 + 2α −2 2
= 3e , −3e
2 2
= +c
1 − 2α 2 − 8 1− 4
1 + 2α 1 −2
= 3e2 = +c
1 − 2α 3 3
3e − 1
2
∴c=1
α=
2(3e 2 + 1) y 2
1 + 2α = +1
And = −3e 2 x − x 3
1 − x2
1 − 2α Put x = 3
3e 2 + 1 y 2
α= = +1
2(3e − 1)
2
3 − 27 1 − 9
194. Let y = y(x) be the solution of the differential y 1
= − +1
equation x (1 – x 2 ) + ( 3x 2 y – y – 4x 3 ) = 0, x > −24
dy 4
dx y 3
1, with y(2) = −2. Then y(3) is equal to =
−24 4
(a) –18 (b) –12
3
(c) –6 (d) –3 y = (−24) = −18
JEE Main-28.06.2022, Shift-I 4

Differential Equation 1151 YCT


195. Let y = y(x) be the solution curve of the −x 2 2  3
dy y = x (1 + log e x) − ∫ x 2 dx 
differential equation = y2 3 
dx x
−x 2
2 x 
3

y 2 (x) =  x 3 (1 + loge x) −  + C
(1 + xy (1 + loge x ) ) , x > 0, y(1) = 3. Then 9
2
y 2
3 3
Given, y(1) = 3
is equal to:
−1 2  1
x2 = 1(1 + log e 1) −  + C
(a) 9 3  3
5 − 2x 3 ( 2 + log e x 3 ) −1 4
= +C
x2 9 9
(b) −5
3x 3 (1 + log e x 2 ) − 2 C=
9
x2 Solution becomes,
(c)
7 − 3x 3 ( 2 + log e x 2 ) x2 2 
− 2 =  x 3 (1 + loge x) −  −
x3  5
y 3 3 9
x2
(d) 2 3
2x 3 ( 2 + log e x 3 ) − 3
x 2x 5
− 2 =  2 + loge x 3  −
y 9 9
JEE Main-25.01.2023, Shift-I
9 5
dy y = −2x  2 + log e x  + 2
3

Ans. (a) : = (1 + xy 2 (1 + log x e )), x > 0, y(1) = 3 y2 x


dx x
9 5 − 2x  2 + log e x 
3 3
dy y =
− = y 3 (1 + loge x)
dx x y2 x2
1 dy 1 1 y2 x2
− 2 = 1 + log e x =
9 5 − 2x ( 2 + log e x 3 )
3 3
y dx y x
1 196. The solution of the differential
Let, − 2 =t
y dy  x + 3y 
2 2

On differentiating w.r.t. x, we get– equation = – 2 2 


, y (1) = 0 is
2 dy dt
dx  3x + y 
= xy
y 3 dx dx (a) log e x + y – =0
( x + y)
2
1 dy 1 dt
=
y 3 dx 2 dx 2xy
(b) log e x + y – =0
( x + y)
2
Differential equation becomes,
1 dt t xy
+ = 1 + loge x (c) log e x + y + =0
2 dx x
( + y)
2
x
dt 2
+ t = 2(1 + loge x) 2xy
dx x (d) log e x + y + =0
( x + y)
2
On comparing with standard equation of linear
differential equation, JEE Main-30.01.2023, Shift-II
dy Ans. (d) : We have differential equation–
+ Py = Q
dx dy  x + 3y 
2 2

2 = – 2 2 
P = ,Q = 2(1 + log e x) dx  3x + y 
x Let,
Integrating factor, y = vx
1
IF = e ∫ = e ∫x
Pdx 2 dx
dy dv
= v+x
IF = e 2ln x
=e ln x 2
=x 2 dx dx
Then equation,
General solution t ⋅ IF = ∫ Q ⋅ IF dx  x 2 + 3v 2 x 2 
dv
v+x = – 2 2 2 
t ⋅ x 2 = ∫ 2(1 + log e x)x 2 dx dx  3x + v x 
−x 2  x3 1 x3  dv  1 + 3v 2 
v+x = – 2 
3 ∫ x 3 
= 2  (1 + log x) − ⋅ dx
y2 
e dx  3+ v 

Differential Equation 1152 YCT


dv  1 + 3v 2  On integrating both side, we get
x = –v –  2 
tan–1x + tan–1y = c
dx  3+ v  x+y
=c
dv –3v – v3 – 1 – 3v 2 1 − xy
x =
dx 3 + v2 x + y = c(1 – xy)
dv (v + 1)3 198. The general solution of differential equation
x =– dy
dx 3 + v2 2
+ 2xy = 2e − x is
Separating the variable we get– dx
(b) y = ( 2x + c ) e− x
2
3 + v2 dx (a) y = 2xe − x
dv = –
(v + 1) 3
x (c) y = 2xe x (d) y = ( 2x + c ) e x
2

On integrating both side we get– AMU-2017


3 + v2 dx Ans. (b) : The differential equation
∫ (v + 1)3 dv = – ∫ x dy 2
+ 2xy = 2e − x
3 + v2 A B C dx
= + +
(v + 1) 3
(v + 1) (v + 1) (v + 1)3
2
dy
On comparing the linear equation + Py = Q
3 + v = A(v2 + 1 + 2v) + B(v+ 1) + C
2
dx
A=1 P = 2x and Q = 2e − x
2

2A + B = 0, B = – 2
Integrating factor (I.F.) = e ∫
Pdx
A+B+C=3
1 + (–2) + C = 3
= e∫
2xdx
C=4
Then, 2 x2
2
3+ v 2
1 2 4 =e
= – + I.F. = e x
2

(v + 1)3 v + 1 (v + 1)2 (v + 1)3


The equation becomes– General solution
dx 1 2 4 y.I.F. = ∫ I.F.Q dx + c
–∫ = dv – ∫ dv + ∫ dv
x v +1 (v + 1) 2
(v + 1)3
∫ 2e e
2
− x2 ⋅ x2
y ex = dx + c [e0 = 1]
2 4
– logx + logc = log(v+1) + –
= ∫ 2 dx + c
2
v + 1 2(v + 1)2 y ex
Now substituting, v = y/x 2
y e x = 2x + c
y  2 2
log c/x = log  + 1 + – 2 y = ( 2x + c ) e− x
2

  y
x + 1  y + 1
x   199. The integrating factor of the D.E.
x 
 y + x  2x + 2xy – 2x dy
( x logx ) + y = 2 logx is
2 2
log c/x – log  = dx
 x  (x + y)2
(a) log(log x) (b) ex
 c  2xy
log  = (c) log x (d) x
 x + y  (x + y)
2
AMU-2017
x+y 2xy Ans. (c) : Integrating factor of differential equation-
– log =
c (x + y) 2 dy
(x log x) + y = 2 log x
2xy dx
log | x + y | + =0
(x + y) 2 On dividing of x logx in above equation,
197. The general solution of the differential dy y 2 log x
equation (1 + y2) dx + (1 + x2) dy = 0 is + =
dx (x log x) x log x
(a) x – y = c (1 – xy) (b) x – y = c (1 – xy)
(c) x + y = c (1 – xy) (d) x + y = c (1 + xy) dy y 2
+ =
MHT CET-2019 / VITEEE-2012 dx x log x x
SRM JEE-2010 / AMU-2007 It is a linear differential equation of the form-
Ans. (c) : Differential equation
dy
(1 + y2)dx + (1 + x2)dy = 0 + Py = Q
On dividing of (1 + x2)(1 + y2) in above equation dx
dx dy 1 2
+ =0 P= and Q =
1 + x 2 1 + y2 x log x x
Differential Equation 1153 YCT
201. Let the solution curve y = y(x) of the
Now, integrating factor = e ∫
Pdx
differential equation
1
∫ x log x dx  –1 3 
I.F. = e dy 3x tan ( x )
5 –1 3
 x – tan x 
3

dt – y = 2x exp  
∫  (1 + x 6 ) 
3
I.F. = e t
log t
(Q t = logx) dx
(1 + x 6 ) 2  
e = t = log x pass through the origin. Then y(1) is equal to :
200. If y = y(x), x ∈ (0, π/2) be the solution curve of  4+π 4–π
the differential equation (a) exp   (b) exp  
 4 2  4 2
( sin 2x ) dy
2

dx
+ ( 8sin 2x + 2sin4x ) y = 2e ( 2sin2x + cos2x )
2 –4x

1– π  π–4
with, y(ππ/4) = e–ππ, then y(π
π/6) is equal to : (c) exp   (d) exp  
4 2 4 2
2 −2 π / 3 2 2π / 3 JEE Main-30.01.2023, Shift-I
(a) e (b) e
3 3 Ans. (b) : Given,
1 −2 π / 3 1 2π / 3 Differential equation–
(c) e (d) e
dy 3x tan ( x )
3 3 5 –1 3  3 –1 3 
 x – tan x 
JEE Main-28.07.2022, Shift-I – y = 2x exp  
( )
3
dx
( )  
6
6 2 1+ x
Ans. (a) : We have given differential equation– 1+ x 
Now,
+ ( 8sin 2 2x + 2sin 4x ) y = 2e –4x (2sin 2x + cos 2x)
dy
(sin 2 2x) 3x 5 tan –1 ( x 3 )
dx – ∫ dx
(1+ x 6 )
dy  8sin 2 2x + 2sin 4x  I. F = e
–4 x  2sin 2x + cos 2x 
+  y = 2e   Let,
dx  sin 2 2x   sin 2 2x  tan–1(x3) = t
dy  2  1
+ ( 8 + 4cot 2x ) y = 2e –4x  + cot 2x  (3x 2 )dx = dt
dx  sin 2x  1+ x6
tdt.tan t
Which is a linear differential equation – –∫
I.F = e sec t
y. e8x+2log sin2x =
 tan –1 (x 3 ) x3 
(4x + 2logsin 2x )  2 sin 2x + cos 2x  I. F = e ∫
– t sin t dt
= exp  – 
∫ 2e 
 sin 2
2x
dx 
  1+ x
6
1 + x6 
For general solution–
 2sin 2x + cos 2x 
= 2 ∫ e 4x sin 2 2x  2
dx  y . I.F = ∫ Q.(I.F)dx
 sin 2x 
 tan –1 ( x 3 )–x 3 
= 2 ∫ e 4x ( 2sin 2x + cos 2x ) dx
(x 3 – tan –1 ( x )3 ) (tan –1 ( x 3 )–x 3 )
 
 
1+ x 6
= ∫ 2x × e = 2∫ x + C
6
  1+ x 1+ x 6
y.e .e
y. e8x + 2log sin2x = e4x sin2x + C
 tan ( x )–x 
–1 3 3
Given, 
 6


y.e 1+ x  = x 2 + C
π
y   = e– π Which is passes through origin we get–
4 e0 = 0 + C
Then we get– C=0
e–π e8(π/4) + 2log sin2(π/4) = e(4π/4) sin2×π/4 + C  x 3 –tan –1 ( x 3 ) 
 
e–π. e2π + 0 = eπ sinπ/2 + C y = x .e 2 
 1+ x 6

eπ = eπ(1) + C Now,
C=0 Find y(1) then,
Equation,  1– tan –1 (1) 
 
y.e8x + 2log sin2x = e4x sin2x y = 1 .e 2 
 1+ x 6 

Now,  1– π / 4 
 
y(π/6) y=e  2 

y.e8(π/6) + 2log sin(2 × π/6) = e4×π/6 sin2×π/6


4–π
2π / 3
 π  e sin π / 3 y = exp  
y   =  4π π 4 2
6  + 2log sin 
e 3 3
202. Let y = y(x) be the solution of the differential
equation (3y2 – 5x2)y dx + 2x(x2 – y2)dy = 0 such
3
e2 π / 3 that y (1) = 1. Then |(y(2))3 – 12y (2)| is equal to:
2 = 2 e –2 π / 3
3
2 (a) 64 (b) 32 2
4 π / 3.  3
e  2  (c) 32 (d) 16 2
JEE Main-31.01.2023, Shift-II
Differential Equation 1154 YCT
Ans. (b) : We have given differential equation, dv  v–x 
(3y2 – 5x2)ydx + 2x (x2 – y2)dy = 0 = 2  v = kx
2x (x2 – y2)dy = (5x2 – 3y2) ydx
dx  5x + v 
dk  kx – x 
dy (5x 2 – 3y 2 )y k+x = 2 
= dx  5x + kx 
dx 2x(x 2 – y 2 )
Let, y = vx dk (k 2 + 3k + 2)
x =–
dy dv dx k +5
= v+x
dx dx (5 + k) dx
The equation become– ∫ (k + 1)(k + 2) dk = ∫ – x
dv (5x 2 – 3v 2 x 2 )vx  4 3  dx
v+x =
dx 2x(x 2 – v 2 x 2 ) ∫  k + 1 – k + 2 dk = – ∫ x
dv (5 – 3v 2 )v 4 ln. (k + 1) – 3 ln(k + 2) = – ln x + ln c
v+x =
dx 2(1 – v 2 ) (k + 1) 4 c
=
dv 5v – 3v3 (k + 2)3 x
x = –v c(y + 2x)3 = (y2 + x)4
2
dx 2 – 2v 2
dv 5v – 3v3 – 2v + 2v3 204. If the solution curve y = y(x) of the differential
x = equation y2dx + (x2 – xy + y2)dy = 0, which
dx 2 – 2v 2 passes through the point (1, 1) and intersects
dv 3v – v3 α, 3 α ), then
x = the line y = 3 x at the point (α
dx 2 – 2v 2
value of loge ( 3 α) is equal to
2(1 − v 2 ) dx
dv = π π
3v − v 3
x (a) (b)
On integrating both sides, we get– 3 2
1 − v2 dx π π
2∫ dv = ∫ (c) (d)
3v − v 3
x 12 6
JEE Main-25.06.2022, Shift-I
2 3 − 3v 2 dx
3 ∫ 3v − v 3
dv = ∫ Ans. (c) : y2dx – xy dy = – (x2 + y2)dy
x y(y dx – x dy) = – (x2 + y2) dy
2 – y (x dx – y dx) = – (x2 + y2)dy
log 3v − v = log x + log c = log(cx)
3

3 xdy – ydx  y 2  dy
3v – v3 = (cx)3/2 = 1 + 2 
x2  x  y
3y y 3
− = c3/ 2 x 3/ 2 d ( y / x ) dy
x x3 ⇒ =
When, x = 1, y = 1 y2 y
3 – 1 = c3/2⋅(1)3/2 1+ 2
x
c3/2 = 2
−1  y 
3x2y – y3 = 2x9/2 ⇒ tan   = log y + C
Now, put x = 2  x
3(2)2y(2) – y3(2) = 2 × 29/2 π π
|y3(2) – 12y(2)| = 32 2 ( α, 3α ⇒ ) 3
(
= log 3α +) 4
π
203. The general solution of the differential
equation (x – y2)dx + y(5x + y2)dy = 0 is : ∴ log 3α =( )12
(a) (y2 + x)4 = C(y2 + 2x)3
205. Let y = y (x) be the solution of the differential
(b) (y2 + 2x)4 = C(y2 + x)3 equation (x + 1) y' – y = e3x (x + 1)2, with y(0)
(c) (y2 + x)3= C(2y2 + x)4
1 4
(d) (y2 + 2x)3= C(2y2 + x)4 = . Then, the point x = – for the curve y =
JEE Main-25.07.2022, Shift-I 3 3
Ans. (a) : (x – y2)dx + y(5x + y2)dy = 0 y(x) is:
(a) not a critical point
dy y2 – x (b) a point of local minima
=
dx y(5x + y 2 ) (c) a point of local maxima
Let, y2 = v (d) a point of inflection
dy dv JEE Main-25.06.2022, Shift-I
2y =
dx dx Ans. (b) : (x + 1) dy – y dx = e3x(x + 1)2
dy  y2 – x  ( x +1) dy – ydx = e3x
2y = 2 2 
( x +1)
2
dx  5x + y 
Differential Equation 1155 YCT
 y  3x 1 1
d =e . = 2 – 2log 3 + C
 x +1  3 3
y e3x 1
= +C C = 2 log 3 – 2 +
x +1 3 3
C = 2log3 – 5/3
 1
 0,  ⇒ C = 0 Now become general solution,
 3 x –1
( x + 1) e3x y = x – 2log(x + 1) + 2log 3 – 5 / 3
y= x +1
3 Now find 7 y(8) =
33x
= ( ( x + 1) 3e3x + e3x ) =
dy 1
( 3x + 4 ) y
8 –1
= 8 – 2log9 + 2log 3 – 5 / 3
dx 3 3
8 +1
7
y = 8 – 2log3 – 5/ 3
3
–4
Clearly, x =is point of local minima 7 y(8) = 24 – 6log 3 – 5 = 19 – 6log3
3 207. The differential equation of the family of circle
206. Let y = y(x) be the solution curve of the passing through the points (0, 2) and (0, –2) is
1

(a) 2xy + ( x 2 − y 2 + 4 ) = 0
dy 1  x – 1 2 dy
differential equation + 2 y=  x> dx
dx x – 1  x +1
(b) 2xy + ( x 2 + y 2 − 4 ) = 0
dy
 1
1 passing through the point  2,  . Then dx
3 
 (c) 2xy + ( y 2 − x 2 + 4 ) = 0
dy
7 y ( 8 ) is equal to dx
(d) 2xy − ( x 2 − y 2 + 4 ) = 0
(a) 11 + 6loge3 (b) 19 dy
(c) 12 – 2loge3 (d) 19 – 6loge3 dx
JEE Main-28.07.2022, Shift-II JEE Main-28.07.2022, Shift-II
Ans. (d) : Given, Ans. (a) : The equation of family of circle
Curve of differential equation Passing, (0,2) and (0, –2)
1/ 2
(x–x1) (x - x2) + (y –y1)(y –y2) + λx = 0
dy 1  x –1 (x – 0) (x – 0) + (y –2) (y + 2) + λ(x) = 0
+ y= 
dx x 2 – 1  x +1  x2 + y2 – 4 + λx = 0 ….(i)
Which is a linear differential equation. differentiating w.r.t x we get–
dy
I.F = e ∫ 2x + 2y + λ = 0
Pdx

1
dx
∫ x 2 –1dx  dy 
= e λ = –  2x + 2y 
1  x –1   dx 
log x –1 
 x +1  Now putting value of λ in equation (i)
= e2 ⇒
x +1  dy 
For general solution of differential equation x2 + y2 – 4 –  2x + 2y  x = 0
 dx 
x –1 x –1 x –1
y. =∫ . . dx x2 + y2 – 4 – 2x2 –2xy
dy
=0
x +1 x +1 x +1 dx
x –1 x –1 dy
y. =∫ . dx + C –x2 + y2 – 4 – 2xy =0
x +1 x +1 dx
x –1  x +1 – 1 – 1  2xy
dy
( )
+ x 2 – y2 + 4 = 0
x +1 ∫  x +1 
y =  dx dx
208. Let the tangent at any point P on a curve
x –1  2   1 
y = ∫ 1 – dx passing through the point (1, 1) and  ,100  ,
x +1  x +1   10 
= x – 2log (x + 1) + C intersect positive x-axis and y-axis at the points
Given that curve passing through 2, 1/ 3 ( ) A and B respectively. If PA : PB = 1 : k and y =
y(x) is the solution of the differential equation
dy
At, x=2 k
e dx = kx + y(0) = k, then 4y(1) – 5loge 3 is
1 2 –1 2
. = 2 – 2 log 3 + C equal to _______
3 2 +1 JEE Main-10.04.2023, Shift-II
Differential Equation 1156 YCT
Ans. (6) : Let a point p(α, β) tangent on a curve. Then equation,
The point p interest positive x-axis and y axis in the dv 4v3 x 3 + 2vx 3
ratio 1:k v+x =
dx 3x 3 v 2 + x 3
Then point p(α, β)
dv 4v3 + 2v
kx + 1.0 k0 + 1.y  v+x =
( α, β ) =  ,  dx 3v 2 + 1
 k +1 k +1 
dv 4v + 2v 3

 kx  y  x = –v
(α, β) =    dx 3v 2 + 1
 k + 1  k + 1 
dv 4v3 + 2v – 3v3 – v
Now equation of tangent at point p (α, β) x =
dy dx 3v 2 + 1
y – y1 = ( x – x1 ) dv v + v 3
dx x =
y=0 dx 3v 2 + 1
ydx dx 3v 2 + 1
x=– + x1 = dv
dy x v(v 2 + 1)
dy On integrating both side we get–
x + ky = 0
dx 3v 2 + 1 dx
dy k ∫ v3 + v dv = ∫ x
+ y=0
dx x Let, v3+ v = t
k (3v2 + 1)dv = dt
Now, I.F = e ∫ x = ek log x = x k
dx
1 dx
Solution of equation, ∫ t dt =∫ x
y.xk = ∫ 0 × x k dx log t = logx + logc
y.xk = c log(v3 + v) = logxc
c=1 v3 + v = xc
k Putting v = y/x, we get–
1
100.   = 1 y
3

 10    + y / x = xc
k 2 x
  1
1
    = Given,
 10   10  y(1) = 1
k=2 (1)3 + 1 = c
dy C=2
= log(2x + 1) 3
dx y
(2x + 1)   + y/ x = x×2
y= {log(2x + 1) – 1} + c x
2 y3 + yx2 = 2x4
At, x=0 Now, at x = 2
1 y3 + 4y – 32 = 0
2 = (0 – 1) + c
2 y(2) = 8 + 8 – 32 < 0
5 y(3) = 27 + 12 – 32 > 0
c= Hence, n = 3
2
3 5 3 One root lies in (2, 3).
y(1) = (log3 – 1) + = log 3 + 1 dy 1
2 2 2 210. The solution of + y = 1 is
4y(1) = 6log3 + 4 dx 3
x/3
4y(1) – 5log3 = 4 + log3 = 4 + 1.09 = 5.09  6 (a) y = 3 + ce
209. Let y = y(x) be the solution of the differential (b) y = 3 + ce–x/3
(c) 3y = c + ex/3
dy 4y 3 + 2yx 2
equation = ,y (1) = 1 . If for some n (d) y2 + x + x2 + 2 = ce2x
dx 3xy 2 + x 3 AP EAMCET-2002
∈ N y(2) ∈ [n – 1, n), then n is equal to ______ Ans. (b) : Given,
JEE Main-25.07.2022, Shift-II dy 1
Ans. (3) : We have differential equation, + y =1
dx 3
dy 4y3 + 2yx 2 1
= Hence, P = and Q = 1
dx 3xy 2 + x 3 3
I.F = e ∫
Put, y = vx pdx

dy dv
= v+x 1
∫ 3 dx
dx dx I.F = e
Differential Equation 1157 YCT
1
x d
∫ dx (x sec x)dx + c
2
I.F = e 3 y. secx =
Solution of general equation, y secx = x secx + c 2

y.ex/3 = ∫ e x / 3dx y = x2 + c cosx


213. The equation of a curve passing through the
ex / 3
y.ex/3 = +c point (0,1), given that the slope of the tangent
1/ 3 to the curve at any point (x, y) is equal to the
x/3 x/3
y.e = 3e + c sum of the x-coordinate and the product of x
y = 3 + c e–x/3 and y coordinates at that point is _____.
2 2

211. y + x2 =
dy
has the solution (a) y = 1 – 2e(x / 2) (b) y = –1 + 2e( x / 2)
dx 2 2
(c) y = –1 – 2e( x / 2) (d) y = 1 + 2e(x / 2)
(a) y + x + 2x + 2 = ce x
2

AP EAPCET-24.08.2021, Shift-II
(b) y + x + 2x 2 + 2 = ce x Ans. (b): Given,
(c) y 2 + x + x 2 + 2 = ce2x Curve → (0,1)
(d) y + x + x 2 + 2 = ce 2x dy
Slope of tangent, = x + xy
AP EAMCET-2002 dx
Ans. (a) : We have– dy
y + x2 = dy/dx – xy = x
dx
It can be written as
dy/dx – y = x2 dy
+ P(x)y = Q(x)
Which is a linear differential equation. dx
Where, P = –1, Q = x2 P = –x, Q = x
I.F = e ∫ = e ∫
2
pdx (–1)dx
If = e ∫ Pdx = e ∫ − xdx = e − ∫ xdx = e – x / 2
I.F = e–x y × IF = ∫ IF ⋅ Q(x)dx
Now for general solution–
− x2 − x2
y × IF = ∫ IF × Qdx + c y⋅e 2
= ∫e 2
⋅ xdx

∫ x .e
2
y.e–x = 2 –x
dx + c x
Let, U = −
2
–x
y.e = –x e 2 –x
+ ∫ 2x.e dx + c
–x
1
dU = − ⋅ 2x ⋅ dx = − xdx
y.e–x = –x2 e–x + 2 [–xe–x – (e–x)] + c 2
y.e–x = –x2 e–x – 2xe–x – 2e–x + c −x
2
dividing by e–x then we get– ⇒ ye 2
= − ∫ e U dU
y = –x2 – 2x – 2 + cex 2
−x
y + x2 + 2x + 2 = cex =e 2
+C
212. The general solution of −x
2
−x
2
dy ye 2
= −e 2
+C
+ y tanx = 2x + x 2 tanx
dx y 1
(a) y – x2 =c sec x =− +C
x2 x2
(b) ycos x = x2sec x + c e 2
e 2
(c) ysec x = x2 + c cos x x2
(d) y = x2 + c cosx y = – 1 + Ce 2 .......(i)
AP EAMCET-22.04.2018, Shift-II Since curve passes through (0, 1)
Ans. (d) : Given, Putting x = 0, y = 1 in equation (i)
dy x2
+ y tanx = 2x + x 2 tanx y = – 1 + Ce 2
dx
02
Which is a linear differential equation
1 = – 1+ Ce 2
P = tanx and Q = 2x + x2tanx
C=2
= e∫
pdx
I.F Putting value of C in equation (i)

x2 
= e∫
tan x 
 2 

y = – 1 + 2e 
= elog sec x = sec x
dy 1
For solution of equation – 214. The general solution of = is
dx ( x + 4y )2
y.secx = ∫ sec x(2x + x 2 tan x)dx + c
 x + 4y 
y. secx = ∫ (2x sec x + x
2
sec x tan x)dx + c (a) y = sin −1  +c
 2 
Differential Equation 1158 YCT
 x + 4y  ye x = ∫ e – x e x + c
2 2 2

(b) 2y = tan −1  +c


 2 
ye x = ∫ 1dx + c
2

 x + 4y 
(c) y = cos −1  +c
2
ye x = x + c
 2 
(d) x + 4y = 2 tan 2y + c
2
y = e – x (x + c)
AMU-2004 216. The differential equation of the family of
 
dy 1 parabola with focus at the origin and the x-axis
Ans. (b) : General solution,   = as axis, is
 dx  (x + 4y)
2
2
Put, x + 4y = v  dy  dy
(a) y   + 4x = 4y
dy dv  dx  dx
1+ 4 = 2
dx dx  dy  dy
(b) y   + 2x –y=0
dv  1   dx  dx
– 1 = 4 2 
dx v   dy 
2
dy
2 (c) y   + y = 2xy
dv  2  4  4+v 2
  dx  dx
=   +1 = 2 +1 ⇒  2 
dx  v  v  v   dy 
2
dy
(d) y   + 2xy +y=0
v 2 dv  dx  dx
⇒ = dx
4 + v2 AMU-2021
 4  AP EAMCET-22.04.2018, Shift-II
 1 – dv  = dx Ans. (b) : The equation of parabola
 4+v
2
 y2 = 4a(x + a) ….(i)
Integrating, differentiating w.r.t x we get–
4
∫1 – 4 + v2 dv = ∫ dx 2y
dy
dx
= 4a
v y dy
v – 2 tan –1 = x + c a= .
2 2 dx
x + 4y 
( x + 4y ) – 2 tan –1 
Putting in equation (i) we get–
= x+c y dy  y dy 
 2  y2 = 4. .  x + 
 x + 4y  2 dx  2 dx 
4y – 2 tan –1  =c 2
 2  dy  dy 
y2 = 2y. .x + y 2  
 x + 4y  dx  dx 
2y – tan –1  =c
 2  dy  dy 
2

y = 2x + y 
 x + 4y  dx  dx 
2y = tan –1  +c
 2   dy 
2
dy
dy y   + 2x –y=0
215. The general solution of -x 2
+ 2xy = e is  dx  dx
dx y
(a) y = e x + c
2 2
(b) y = e − x + c 217. If cos = A log x + C is the general solution of
x
(c) y = e − x ( x + c )  y  
2 2
(d) y + x 2 y = e − x + c y
 xsin  dy =  ysin – x  dx, then A =
AMU-2004  x   x 
(a) 2 (b) 1
dy 2
Ans. (c) : The general solution of, + 2xy = e – x (c) –1 (d) –2
dx AP EAMCET-06.07.2022, Shift-II
dy Ans. (b) : We have given,
Comparing + py = Q
dx  y  y 
P = 2x, Q = e – x
2
 x sin  dy =  ysin   – x  dx
 x  x 
Integrating factor, y
ysin – x
I.F ⇒ e ∫
p dx
dy x
=
dx x sin y / x
= e∫
2x dx 2
= ex
General solution, Let, y = vx
dy dv
y.(I.F) = ∫ Q(I.F)dx + C = v+x
dx dx
Differential Equation 1159 YCT
Equation become– y = cosx – 2cos2x
vx y = cosx – 2(1–sin2x)
dv
vx sin –x y = cosx – 2 + 2sin2x
v+x = x y = 2sin2x + cosx – 2
dx vx
x sin 219. The solution of the differential equation
x dy
x
dv vsin v – 1
= –v
( 2x − 4y + 3 ) + ( x − 2y + 1) = 0 is
dx
dx sin v (a) log((2x – 4y) + 3) = x – 2y + c
dv vsin v – 1 – vsin v (b) log[2(2x – 4y) + 3] = 2(x – 2y) + c
x =
dx sin v (c) log[2(x – 2y) + 5] = 2(x + y) + c
dv −1 (d) log[4(x – 2y) + 5] = 4(x + 2y) + c
x = (c is an arbitrary constant)
dx sin v
On integrating both side we get– AP EAMCET-2016
dx Ans. (d) : We have differential equation
∫ sin vdx = – ∫ x dy
(2x – 4y + 3) + (x – 2y + 1) = 0
– cosv = – logx + c dx
y y dy (x – 2y + 1)
Put, v = , cos   = log x + c =–
x x dx (2x – 4y + 3)
Comparing it by cos(y/x) = A logx + c dy (x – 2y + 1)
A=1 =–
dx 2(x – 2y) + 3
218. The solution of the equation Let, x – 2y = v
dy dy dv
+ 2ytanx = sinx satisfying y = 0 when 1–2 =
dx dx dx
π 1  dv  dy
x = , is
3 1 –  =
2 2  dx  dx
(a) y = 2sin x + cos x – 2
(b) y = 2sin2x – cos x – 2 Putting in equation we get–
2
(c) y = 2cos x – sin x + 2 1  dv  –(v + 1)
(d) y = 2cosx + sin2x – 1 1 –  =
2  dx  2v + 3
AP EAMCET-21.04.2019, Shift-I –2(v + 1) dv
Ans. (a) : We have equation 1– =
2v + 3 dx
dy 2v + 3 + 2v + 2 dv
+ 2y tan x = sin x =
dx 2v + 3 dx
Which is linear equation comparing –
4v + 5 dv
dy =
+ py = Q 2v + 3 dx
dx 2v + 3
dv = dx
= e∫
pdx
I.F, 4v + 5
Integrating on both side we get–
= e∫
2 tan x

2v + 3
∫ dv = ∫ dx
2log secx
=e
= sec x2 4v +5
For general solution of differential equation, 1 4v + 5 + 1
2 ∫ 4v + 5
dv = ∫ dx
y. sec x = ∫ sin x.sec xdx + c
2 2

1 1 
y. sec x = ∫
2 sin x
.sec xdx + c  1+  dv = ∫ dx
2  4v + 5 
cos x
1
y. sec2x = ∫ tan x.sec x + c v + log(4v + 5) = 2x + c
4
y. sec2x = secx + c 1
log(4v + 5) = 2x – v + c
Given, when x = π/3, then y = 0 4
0 = secπ/3 + c 1
c+2=0 log [ 4(x – 2y) + 5] = 2x – ( x – 2y ) + c
4
c=–2 log4 (x –2y) + 5 = 4(x + 2y) + c
Now equation become– 220. Find the solution of the differential equation
ysec2x = secx – 2 y –x x y
(e )dy = (e – e )dx
1 1
y = secx⋅ – 2× 2
(a) e e = e − e + c
y x 2x x2
sec 2 x sec x
Differential Equation 1160 YCT
x y
(b) e y e x = e x ee − ee + c 1 b2
tan a = ⋅ +c
x x x 2 2
(c) e y e e = e x ee − e e + c
1  ( ln x ) 
y x x  2 2 

(d) ee e x = e x e e − e e + c tan y 2 = +c
AP EAMCET-17.09.2020, Shift-I 2 2 
 
y− x y
)dy = (e x − e )dx
= ( ln x ) + c
Ans. (c) : (e 1 2 2
tan y 2
dy 4
⇒ e y ⋅ = e2 x − e x ⋅ e y 1
= ( 2ln x ) + c
2
dx tan y 2
y dy x y 4
⇒ e . + e .e = e2x
= ( ln x ) + c
2
dx tan y 2
y y dy dz dz
Let e = z⇒e ⋅ = ⇒ + e x ⋅ z = e2 x 222. The differential equation
dx dx dx dy
e x⋅dx x y + x = a where a is a constant, represents?
IF = e∫ = ee dx
(a) a set of circles having centre on the y-axis
∴ Solution,
x x (b) a set of parabolas
z ⋅ ee = ∫ e2 x ⋅ ee ⋅ dx + c (c) a set of circles having centre on the x-axis
(d) a set of straight lines
Let ex = u
SCRA-2014
e x ⋅ dx = du Ans. (c): Given,
x
⇒ z ⋅ ee = ∫ u ⋅ eu du + c = ueu − eu + c dy
y +x=a
y x x dx
⇒ e ⋅ ee = ee (e x − 1) + c
221. The solution of the differential equation ∫ ydy + ∫ = ∫ adx
xdx
y2 x 2

y  2xsec 2 ( y 2 )

dy
dx


4
+ y 3  = ln x 2e y is ( ) 2
+
2
= ax + c

y2 x 2
(a) sec3 y 2 = (1nx ) + c
2
+ − ax = c
2 2
(b) sec3 y 2 = 12 (1nx ) + c y 2 + x 2 − 2ax = 2c
(c) tan y 2 = (1n x ) + c
2
Similar as:-
(d) None of the above x 2 + y 2 + 2gx + 2fy + c = 0
where c is an arbitrary constant. So, the differential equation is a set of circles having
SCRA-2014 centre on the x-axis.
Ans. (c): 223. If I1 is the integrating factor of the differential
dy


dy
dx

y  2x sec 2 ( y 2 ) + y3  = ln x 2 e y

4

( ) equation x
dx
– y = x 2 and I2 is the integrating
factor of the differential equation
2xy sec2 ( y 2 ) + y 4 = ln x 2 + ln e y
dy
dx
4

( ) x
dy
dx
–2
+ y = x , then which one of the following

2xy sec ( y ) 2 dy is not correct?


2
= ln x 2 + y 4 − y 4
dx (a) I1 I2=1 (b) I2 = x2 I1
2
(c) I1 = x I2 (d) I2 > I1 for x > 1
ln x 2
2y sec 2 ( y 2 ) dy = dx SCRA-2015
x
Now, dy
Ans. (c) : x – y = x2
Integrated both side, we get, dx
1 2ln x 2 dy y
∫ 2ysec ( y ) dy = 2 ∫ x dx – = x2
2 2
dx x
Let y 2 = a & Let ln x 2 = b Comparing with
dy
+ p(y) = Q
1 dx
2ydy = 2da ⋅ 2xdx = db 1
x2 P=− , Q=x
So, x
dx
1 – ∫x 1
∫ sec ( a ) da = 2∫
(I.F.)I1 = e = e – log x =
2
bdb
x
Differential Equation 1161 YCT
dy dv
Comparing with + p(y) = Q x = v log v
dx dx
dy 1 dv dx
x +y= 2 =
dx x v log v x
dy y 1 On integrating both side we get–
+ = log(logv) = logx + logc
dx x x 3
log (logv) = logxc
1 1
p = ,θ = 3 logv = xc
x xdx v = exc

(I.F.)I 2 = e = e x log x
=x Putting, v = y/x
So, I1 = x I2 not satisfy
2 y
= e xc
x
dy
224. If the solution of – y loge 0.5 = 0, y(0) = 1, y = xexc
dx 226. The general solution of the differential
and y(x) → k, as x→ ∞ then k =
dy
(a) ∞ (b) –1 equation + yg' (x) = g (x) g' (x) is
(c) 1 (d) 0 dx
AP EAMCET-04.07.2021, Shift-I (a) g(x) + log (1+ y +g(x) = c
Ans. (d) : We have given, (b) g(x) + log (1+ y − g(x) = c
dy (c) g(x) − log (1+ y + g(x) = c
= y log e 0.5
dx
We have linear equation– (d) g(x) − log (1+ y − g(x) = c
dy AP EAMCET-18.09.2020, Shift-II
= y log e 0.5 Ans. (b) : Given differential equation–
dx
dy = log e 0.5dx dy
+ yg '(x) = g(x)g '(x)
Integrating on both side we get– dx
log y = x. loge 1/2 + c Which is a linear differential equation–
Given, P = g'(x) and Q = g(x) g'(x)
y(0) = 1
= e∫
Pdx
log 1 = 0 + c I.F,
c=0
= e∫
g '( x )dx
logy = x loge 1/2 g(x)
We find y(x) → k as x = ∞ I.F, =e
log y = x log e 1/2 For general solution–
logy = (∞) loge1/2 y.e = ∫ e .g(x)g '(x)dx + c ]
g(x) g (x )

1 Let,
logy = log e 1/ 2
0 g(x) = t
log1/2 = 0 = k g'(x)dx = dt
k=0
y. eg(x) = ∫ e t .tdt
dy
225. The solution of x = y (log y − log x + 1) is y. eg(x) = t et – et + c
dx
(a) y = xe cx 2
(b) y = cx 2
y. eg(x) = g(x) eg(x) – eg(x) + c
2
(c) y = cx log (x) (d) log (y) = cx eg(x)(y – g(x) + 1) = c
AP EAMCET-18.09.2020, Shift-II Taking log on both side–
Ans. (a) : We have– g(x) + log(1 + y –g(x)) = c
dy 227. If y = y(x) is the solution of the differential
x = y(log y – log x + 1)
dx  2 + sinx  dy
dy y equation   + cosx = 0 with y ( 0 ) = 1 ,
= (log y – log x + 1)  y + 1  dx
dx x
Let, y = vx π
then y   is equal to
dy dv 2
= v+x
dx dx 1 2
(a) (b)
dv vx
v+x = ( log vx – log x + 1) 3 3
dx x 4
dv (c) 1 (d)
v+x = v ( log v + 1) 3
dx AP EAMCET-2011
Differential Equation 1162 YCT
Ans. (a) : We have differential equation- Ans. (b) : We have given,
 2 + sin x  dy dy
= sin(x + y) tan(x + y) – 1 ….(i)
  + cos x = 0 dx
 y + 1  dx Put,
 2 + sin x  dy x+y=v
  = – cos x
 y + 1  dx 1+
dy dv
=
dy cos x dx dx
=– dx dy dv
y +1 2 + sin x = –1
Integrating on both side– dx dx
dy cos x From equation (i)
∫ y + 1 = ∫ – 2 + sin x dx dv
– 1 = sin v tan v – 1
log(y + 1) = – log(2 + sinx) + logc dx
log(y + 1) = – log(2 + sinx) + logc dv
= sin v tan v
(y + 1) (2 + sinx) = c dx
Given, 1
y(0) = 1 dv = dx
(1 + 1) (2 + 0) = c sin v tan v
cosec v. cot v dv = dx
c=4 On integrating both side we get–
Then equation become–
(y + 1) (2 + sinx) = 4 ∫ cos ec v.cot vdv = ∫ dx
Now, y(π/2) we get– – cosec v = x + c
(y + 1) (2 + sinπ/2) = 4 x + cosec v = c
(y + 1) (2+ 1) = 4 Putting, v = x + y we get–
4 x + cosec (x + y) = c
(y + 1) =
3 dx x
230. The solution of + = x 2 is:
4 1 dy y
y = –1 =
3 3 1 1
228. The solution of (a) = cx – x log x (b) = cy – y log y
y x
dy
tan y = sin(x + y) + sin(x − y)is (c)
1
= cx + x log y (d)
1
= cx – y log x
dx x y
(a) sec y = 2 cos x +c (b) sec y = –2 cos x + c
(c) tan y = –2 cos x + c (d) sec2y= –2 cos x + c AP EAMCET-2006
AP EAMCET-2010 dx x
Ans. (b) : + = x2
dy dy y
Ans. (b) : tan y = sin( x+y) + sin( x–y) 1 dx 1
dx + =1
 dy  sin( x+y) + sin( x–y) x 2 dy xy
  =
 dx  tan y 1
Let, = 1
 
dy 2sin x cos y x
 = − 1 dx dt
 dx  tan y =
2
tan y x dy dy
dy = 2sin dx
cos y −dt t
+ =1
sin y dy y
dy = 2sin dx
cos 2 y dt  −1 
+   t = −1
sin y dy  y 
∫ cos2 y ∫ dy = 2sin dx
1
∫ − dy 1
1 I.F = e y =
= c − 2cos x y
cos y
1 1 1
sec y = −2cos x + c t. = ∫ −1× − ∫
y y y
dy
229. The solution of the differential equation 1
dx = log y + c
sin (x + y) tan (x + y) –1 is xy
(a) cosec (x + y) + tan(x + y) = x + c 1
(b) x + cosec (x + y) = c = − y log y + cy
x
(c) x + tan (x + y) = c 1
(d) x + sec (x + y) = c = cy − y log y
AP EAMCET-2009 x
Differential Equation 1163 YCT
dy dy y (1 + x )
2
231. The solution of + 1 = ex + y is −1
+ =
(a) e–(x + y) + x + c = 0
dx
(b) e–(x + y) – x + c = 0
dx x ( ) + x3
1 + x 2
x
(c) ex + y + x + c = 0 (d) ex + y – x + c = 0 dy y 1
+ =−
AP EAMCET-2007 dx x x + x3
Ans. (a) : Given, dy
+ yP(x) = Q(x)
 dy  x+y dx
  + 1 = e ..…(i)
 dx  Hence, I.F. = e∫P(x)dx
1
Let, x + y = z ∫ dx
e x
dy dz
∴ +1 = …..(ii) elog(x) = x
dx dx 234. The solution of the differential equation
Now, comparing (i) and (ii) we get,
dy
dz – 2y tan2x = e x sec2x is
= ez dx
dx (a) y sin2x = ex + C (b) y cos2x = ex + C
dx = e–z dz x
(c) y = e cos2x + C (d) y cos2x + ex = C
On integrating both sides, we get – AP EAMCET-2013
∫ ∫
−z
dx = e dz Ans. (b) : We have differential equation–
x + c = –e–z dy
– 2y tan 2x = e x sec 2x
x + e–(x + y) + c = 0 dx
232. The differential equation obtained by Which is a linear differential equation,
eliminating the arbitrary constants a and b P = –2tan2x and Q = exsec 2x
 log sec2 x 
from xy = aex + be–x is
= e∫ = e ∫
pdx – 2 tan 2x dx –2  
2
I.F, = e  2 
d y dy –1
= (sec2x) = cos2x
(a) x 2 + 2 − xy = 0
dx dx Now for general solution
d2 y dy y. cos2x = ∫ cos 2x.e x sec 2x dx
(b) 2
+ 2y − xy = 0
dx dx
ycos2x = ∫ e x dx
d2 y dy
(c) x 2 + 2 + xy = 0 y cos2x = ex + C
dx dx 235. If X is a poisson variation with P(X = 0) = 0.8,
d 2 y dy then the variance of X is
(d) + − xy = 0
dx 2 dx (a) loge 20 (b) log10 20
AP EAMCET-2007 (c) loge 5/4 (d) 0
Ans. (a) : Given, AP EAMCET-2004
x
xy = ae + be –x
…..(i) Ans. (c) : We have–
On differentiating w.r.t 'x', we get – P(X = 0) = 0.8
We have that,
dy
x + y = ae x − be − x e– λλ x
dx P(X = 0) =
d 2 y dy dy x!
x 2+ + = ae x + be − x x=0
dx dx dx Then,
d2 y dy e – λ (λ ) 0
x 2 + 2 − xy = 0 = 0.8
dx dx 0!
–λ
233. An integrating facto of the equation e = 0.8
( ) ( )
1 + y + x 2 y dx + x + x 3 dy = 0 is e–λ = 4/5
e–λ = 5/4
(a) ex (b) x2 λ = loge 5/4
1 236. The differential equation of the family of
(c) (d) x
x parabolas with vertex at (0,-1) and having axis
AP EAMCET-2013 along the Y-axis is
Ans. (d) : Dividing the entire equation by 'dx' gives as (a) yy '+ 2xy + 1 = 0 (b) xy '+ y + 1 = 0
(c) xy ' − 2y − 2 = 0 (d) xy '− y − 1 = 0
(1 + y + x 2 y ) + dx ( x + x 3 ) = 0
dy
AP EAMCET-2014
dy y (1 + x )
2 Ans. (c) : We have the family of parabola having axis
−1
+ = along the y axis
dx x + x3 x + x3 (x–h)2 = 4a(y–k)
Differential Equation 1164 YCT
Where (h, k) be the vertex of parabola, ey
Given vertex (h, k)→ (0, –1) 1= + + C1x.e y
The equation become (x–0)2 = 4a(y+1) 2x
2x = ey + 2x2C1ey
x2 = 4a (y + 1) 2x = ey(1 + Cx2) Q c = 2C1
differentiating w.r.t x we get–
dy 238. The solution of the differential equation
2x = 4a dy 2xy − y 2
dx = , is
x dx dx 2xy − x 2
a= (a) xy(x + y) = C (b) xy(x – y) = C
2 dy
(c) x2y(x –y) = C (d) x3y(x – y) = C
Substituting he value of 'a' in equation we get– AMU-2018
x dx
x2 = 4. (y + 1) dy 2xy − y 2
2 dy Ans. (b) : We have, =
dx 2xy − x 2
dy
x = 2(y + 1)  2x 
dx y2  − 1
x
dy
– 2y – 2 = 0
dy
=  y 
dx dx 2  2y 
xy' – 2y – 2 = 0 x  − 1
 x 
dy 1 ey dy dv
237. The solution of + = is Put y = vx ⇒ =v + x
dx x x 2 dx dx
(a) 2x = (1+ Cx ) e 2 y
(b) x = (1+ Cx ) e y
 2x  2 
v2 x 2  − 1  − 1
(c) 2x 2 = (1+ Cx 2 ) e-y (d) x 2 = (1+ Cx 2 ) e-y v+x
dv
=  vx  = v 2  v 
dx
x2 
 2vx 
− 1 ( 2v − 1)
AP EAMCET-2015
Ans. (a) : We have given,  x 
dv 2v − v 2
dy 1 e y x = −v
+ = dx 2v − 1
dx x x 2 dv 2v − v 2 − 2v 2 + v 3v − 3v 2
– y dy e– y 1 x = =
e + = 2 dx 2v − 1 2v − 1
dx x x 2v − 1 dx
Let, –y
e =v dv =
Now equation become– 3( v − v2 ) x
dv v 1 On integrating both side we get.
– + = 2
log ( v − v 2 ) = − log x + log k
dx x x 1
dv v 1 3
– =– 2
k
dx x x ⇒ log ( v − v 2 ) = 3log  
Which is a linear differential equation in v x
1 –1 K
3
P = – and Q = 2 log ( v − v 2 ) = log  
x x
I.F, x
2 3
1 y y K
– ∫ dx ⇒ − =
e ∫ = e x = e – log x =
pdx 1
x x 2 x3
x ⇒ xy (x – y) = C {where, K3 = C}
For general solution we get–
dy
v(I.F) = ∫ (1.F).Q.dx + C 239. The differential equation y +x=a
dx
1 1 1  (a is a constant) represents
v = ∫  – 2  dx (a) a set of circles having centre on the y-axis
x x x 
(b) a set of circles having centre on the x-axis
1 – y x –2 (c) a set of ellipses
e = + C1 (d) none of these.
x –2
–y 2 AMU-2008
e –x
= + C1 dy
x –2 Ans. (b) : We have y + x = a , ydy + xdx = adx
–y
dx
e 1
= 2 + C1 y2 x 2
x 2x Integrating we get + = ax + C
2 2
1 2 2
x + y – 2ax + k = 0
e–y = + + C1x
2x Which represent a set of circles having centre on x-axis.
Differential Equation 1165 YCT
240. Which of the following is a correct solution of (x − y) 2 + 2
 dy  (c) log + 2(y − x) = 0
xcosx   + y ( xsinx + cosx ) = 1 ? 2
 dx 
(x + y) 2 − 2
(a) y x sec y = C+tan x (b) y x cos y = C+tan x (d) (y − x) + log =0
(c) y x sec x = C+tan x (d) None of the above 2
AMU-2014 AP EAMCET-22.04.2019, Shift-II
 dy  Ans. (a) : Given,
Ans. (c) : x cos x   + y(x sin x + cos x) = 1 x + y – 1 . dy x + y + 1
 dx  =
dy (x sin x + cos x) 1 x + y – 2 dx x + y + 2
+y = Let, x+y=v
dx x cos x x cos x
dy  1 1 dy dv
+  tan x +  y = sec x 1+ =
dx  x x dx dx
 1 dy  dv 
∫  tan x + x dx log sec x + logx log (xsec x) =  − 1
If = e  =e =e = x sec x dx  dx 
Required solution v – 1  dv  v + 1
1 Then,  – 1 =
y. IF = ∫ I.F sec xdx + C v – 2  dx  v + 2
x
dv  v + 1  v – 2 
1 =   +1
y x sec x = ∫ x sec x   sec x + C dx  v + 2  v – 1 
x
dv 2v 2 – 4
yx sec x = ∫ sec 2 x dx + C = 2
dx v + v – 2
xy sec x = tan x + C
v2 + v – 2
241. The solution of the differential equation dv = dx
d2y 2v 2 – 4
+ y = 0 is_______ v2 + v – 2
dx 2
(a) y = 3 sin x + 4 cos x (b) y = x2
∫ v2 – 2
dv = ∫ 2dx
(c) y = x + 2 (d) y = log x  v 
AP EAMCET-19.08.2021, Shift-I ∫ 1 + v2 – 2  dv = ∫ 2dx
d2 y 1
Ans. (a): Given, differential equation +y=0 v + log(v 2 – 2) = 2x + c
dx 2 2
A solution is function of x which satisfies given Now, putting v = x + y we get–
differential equation:- 1
Option (a):- x + y + log | (x + y) 2 – 2 |= 2x + c
y = 3sin x + 4cos x 2
1
dy y – x + log | (x + y)2 – 2 |= c .....(i)
= 3cos x − 4sin x 2
dx Given,
d2 y y = 1 when x = 1
= −3sin x − 4 cos x
dx 2 1
1 – 1 + log | 4 – 2 |= c
d2 y 2
= − ( 3sin x + 4cos x )
dx 2 1
log 2 = c
d2 y 2
= −y On putting the value c in equation (i),
dx 2
2 1 1
d y
+y=0 y – x + log | (x + y) 2 – 2 |= log 2
dx 2 2 2
242. The solution of the differential equation 1 (x + y) – 2
2
y – x + log =0
x + y − 1 dy x + y + 1 2 2
= , given that y = 1 when
x + y − 2 dx x + y + 2 (x + y) 2 – 2
x = 1: is 2(y – x) + log =0
2
(x + y) − 2
2
(a) 2(y − x) + log =0 243. If y″″ – 3y′′ + 2y = 0 where y(0) = 1, y′′(0) = 0,
2 then the value of y at x = loge 2 is
(x + y) 2 − 2 (a) 1 (b) –1
(b) log = (x − y) 2 (c) 2 (d) 0
2
WB JEE-2010
Differential Equation 1166 YCT
Ans. (d) : Given, 1
y'' – 3y' + 2y = 0 t. y = ∫ y .dy
d2 y dy t. y = logy + C
– 3 + 2y = 0 ….(i)
dx 2 dx Putting t = logx
Let, ylogx = logy + C
m = dy/dx Given,
Then, m2 – 3m + 2 = 0 y(1) = 1
(m –1) (m – 2) = 0 Then,
m = 1, 2 C=0
General solution of differential equation ylogx = logy
y = Aex + Be2x logxy = logy
y' = Aex + 2Be2x y = xy
Given, d2 y dy
y(0) = 1 245. The solution of 25 – 10 + y = 0,
1=A+B …..(ii) dx 2 dx
And, y'(0) = 0 or x = 0 then y = 0 y ( 0 ) = 1, y (1) = 2e1/5 is
A = 2 and B = – 1 (a) y = e5x + e −5x (b) y = (1 + x ) e5x
Substituting the value of A and B then
y = 2ex – e2x (c) y = (1 + x ) e x / 5 (d) y = (1 + x ) e − x / 5
At, x = loge2 WB JEE-2012
y = 2eloge 2 − e 2 loge 2 Ans. (c) : We have given,
y = 2 × 2 – 22 = 0 d2 y dy
25 2 – 10 + y = 0
244. Let y be the solution of the differential equation dx dx
dy y2 y = emx
x = satisfying y(1) = 1. Then, y dy
dx 1 – ylogx = me mx
satisfies dx
(a) y = x y−1 (b) y = x y d2 y
And, = m 2e mx
(c) y = x y+1 (d) y = x y+ 2 dx 2
WB JEE-2012 d2 y dy
25 2 – 10 + y = 0
Ans. (b) : Given, dx dx
25m2emx – 10memx + emx = 0
dy y2
x = emx (25m2 – 10m + 1) = 0
dx 1 – y log x For auxiliary equation–
dx x(1 – y log x) 25m2 – 10m + 1 = 0
= (5m –1) (5m – 1) = 0
dy y2
dx x –xy log x 1 1
– 2 = m= ,
dy y y 2 5 5
Since roots are real and equation for general solution.
1 dx 1 log x y = (c1 + c2x) ex/5
– =– …..(i)
x dy y 2 y Given,
Let, y(0) = 1
logx = t c1 = 1
1 dx dt and, y(1) = 2e1/5
= (c1 + c2)e1/5 = 2e1/5
x dy dy
Now, c1 + c2 = 2
dt 1 t 1 + c2 = 2
– 2 =– c2 = 1
dy y y Then general solution,
dt t 1 y = (1+1.x)ex/5
+ = 2
dy y y y = (1+x)ex/5
Now it is linear differential equation 246. The integrating factor of the differential
1
∫ y dy logy dy
I.F, = e =e equation 3xlog e x + y = 2log e x is given by
I.F, =y dx
( log e x ) (b) log e ( log e x )
3
For general solution– (a)
1
( log e x )
1/ 3
t.y = ∫ 2 .ydy (c) loge x (d)
y
WB JEE-2012
Differential Equation 1167 YCT
Ans. (d) : We have differential equation,
(a)
( x + y ) = tan y + C , C is an arbitrary constant
dy
3xlogex + y = 2 log e x a a
dx (b) xy = a tan Cx, C is an arbitrary constant
It can be written as– x y
dy 1 2 (c) = tan , C is an arbitrary constant
+ y= a C
dx 3x log e x 3x (d) xy = tan( x + C), C is an arbitrary constant
dy WB JEE-2017
Which is a linear form + Py = Q, where P and Q are Ans. (a) : Given,
dx
function of x and the integrating factor is given by the 2 dy
( x + y) = a2
following formula e ∫ dx
Pdx

1
Let, x+y=v
∫ 3x loge x dy dv
∴ IF = e dx ⇒ 1+ =
Put, logex = t dx dx
1 dy dv
dx = dt ⇒ = −1
x dx dx
dt
∫ 1/ 3
 dv 
= e1/ 3 t = e1/3 logt = elog t = t1/3 = (logex)1/3 ∴ v2  − 1 = a 2
247. A solution of the differential equation  dx 
 dy 
2 dv v 2
+ a2
dy ⇒ =
  –x + y = 0 is : dx v2
 dx  dx
2
(a) y = 2x (b) y = – 2x v
⇒ dv = dx
(c) y = 2x – 4 (d) y = 2x + 4 v2 + a 2
AMU-2013 On integrating both sides, we get
Ans. (c) : Given, v2
 dy 
2

dy
∫ v2 + a 2 dv = ∫ dx
  −x +y=0
a2
 dx  dx
dy
⇒ ∫ 1− 2
v + a2
dv = x + C '
Let, =p
dx a2 v
p2 – px + y = 0 ⇒ v − tan −1 = x + C '
a a
y = xp – p2 x+y
On Differentiating both sides w.r.t x, we get– ⇒ x + y – a tan–1 = x + C'
dy dp dp a
= x ⋅ + p − 2p ⋅ x+y
dx dx dx ⇒ y = a tan–1 + C′
dy dp a
= (x − 2p) + p y − C' x+y
dx dx = tan −1
dy a a
On putting the value in above equation, x+y  y+C
dx ⇒  a  = tan  a  .
dp    
p = (x – 2p) +p
dx
Hence,
( x + y ) = tan y + C , C is an arbitrary constant
dp a a
(x – 2p) =0
dx msin –1 x
249. If y = e then
dp
=0 dy 2
dy
dx (1 – x2) 2
–x – ky = 0 , where k is equal to
On integrating w.r.t. x, we get– 2
dx dx
p=c (a) m (b) 2
dy (c) – 1 (d) – m2
Then, =c WB JEE-2017
dx Ans. (a) : Given,
y = x.c – c2
c=2 d2y dy
(1 – x2) –x – ky = 0 .....(i)
Then, y = 2x – 4 dx 2 dx
−1

dy y = e m sin x
248. Solution of (x + y)2 = a2
dx dy −1 m
⇒ = e m sin x .
(‘a’ being a constant) is dx 1 − x2

Differential Equation 1168 YCT


dy −1 Let v + ev = t ⇒ (1 +ev) dy = dt
⇒ 1− x2 = me m sin x
1 + ev
dx
dy
∴ ∫
v+e v
dv =
dt
t ∫ (
= log t = log v + e v .....(iv))
⇒ 1− x2 = my
dx ⇒ log (v + ev) = – log y + log C [From equation (iii)
d2 y 1 dy dy and (iv)]
⇒ 1− x2 2
+ ( −2x ) = m ⇒ log (v + ev) + log y = logC
dx 2 1− x 2 dx dx ⇒ y(v + ev) = C
2
⇒ (1 − x ) dx
d y
−x
dy
2

dx
=m 2
1− x2
dy
dx
x 
⇒ y  + ex / y  = C
2
y 
⇒ (1 − x ) dx
d y
−x
dy
2
= m. ( my )  x + ye x / y 
⇒ y   = C
2
dx
2  y 
⇒ (1 − x ) dx
d y
−x
dy
2

dx
−m y = 0 2
2
.....(ii) ⇒ x + ye x / y = C
On comparing equation (i) and (ii), we get– dy
251. General solution of ( x + y )
2
∴ k = m2 = a 2 ,a ≠ 0 is
dx
250. The general solution of the differential (C is an arbitrary constant)
 x 
 x
x y
(a) = tan + c (b) tan xy = c
equation  1 + e y  dx +  1 –  e x/y dy = 0is a a
   y 
  y+c x+y
(c is an arbitrary constant) (c) tan ( x + y ) = c (d) tan =
a a
x x
WB JEE-2019
(a) x − ye = c y
(b) y − xe = c y
Ans. (d) : Given,
x x dy
(c) x + ye = c y
(d) y + xe y = c ( x + y )2 = a 2 ,a = 0
dx
WB JEE-2019 Let, x+y=t
Ans. (c) : Given, dy dt
1+ =
 x  dx dx
1 + e y  dx + 1 − x  e x / y dy = 0 dy dt
   y ⇒ = −1
  dx dx
 x  dt 
( )
⇒ 1 + e x / y dx = −e x / y  1 −  dy
 y
Therefore, t2  − 1  = a 2
 dx 
2 dt
dx ex / y  x  ⇒ t − t2 = a2
⇒ =− 1 −  .....(i)
( )
dx
dy 1 + ex / y  y  dt
This is homogeneous differential equation. ⇒ t2 = a2 + t2
dx
So, put x = vy
t2
dx dv dt = dx [Variable separation]
=v+y .....(ii) a2 + t2
dy dy
t2
dv −e (1 − v ) ∫( ∫
v
⇒ dt = dx [int egration]
Therefore, v + y
dy
=
1 + ev
t2 + a2 )
[From equation (i) and (ii)] t2 + a2 − a2
y
dv −e v + ve v
= −v
⇒ ∫ (t 2
+ a2 )
dt = x + C '
dy 1 + ev
[doing a2 adding and subtracting]
v
−e + ve − v 1 + e v
( v
) = −e v v
+ ve − v − ve v
 a2 
=
1+ e v
1 + ev ∫
⇒ 1 − 2
 t +a 
dt = x + C'
2

dv (
− ev + v ) a2
y
dy
=
1+ e v ∫
⇒ dt − 2 ∫
t + a2
dt = x + C '
v
1+ e dy dt

v+e v
dv = −
y
[Variable separation] ⇒ t − a2 2 ∫
t + a2
= x + C'

1 + ev dy 1  t 
⇒ t − a 2  tan −1    = x + C '
⇒ ∫ v+e v
dv = − ∫y .....(iii)
a  a 

Differential Equation 1169 YCT


x+y dr sin 2θ
⇒ ( x + y ) − a tan −1   = –r
 a  dθ cos 2θ
= x + C '[on putting the valueof t] dr
cos 2θ + r sin 2θ = 0
y − C' dθ
x+y
⇒ = tan −1   254. The solution of the differential equation
a  a 
x  x 
x+y  y − C'  y sin   dx =  x sin   − y  dy satisfying
⇒ = tan    
y   
y 
a  a 
x+y π
 y+C y   = 1 is
⇒ = tan   4
a  a 
[where, C = – C' is an arbitrary constant] x 1
(a) cos = log e y +
 y + C x + y y 2
tan  =
 a  a x
(b) sin = log e y +
1
252. The differential equation formed by y 2
eliminating a and b from the equation x 1
(c) sin = log e x −
y = ex (acos x + bsin x)is y 2
d 2 y dy d2 y dy x 1
(a) 2 2 + − 2y = 0 (b) 2
+ 2 − 2y = 0 (d) cos = − log e x −
dx dx dx dx y 2
d 2 y dy d2 y dy WB JEE-2013
(c) 2 2 − + 2y = 0 (d) − 2 + 2y = 0
dx dx dx 2 dx Ans. (a) : Given, differential equation is–
AP EAMCET-21.04.2019, Shift-II x   x  
Ans. (d) : Given that, ysin   dx =  x sin   − y  dy
 y   y  
y = ex(a cos x + b sin x)
On differentiating both sides with respect to x, we get- x
dy x sin   – y
= e x ( –a sin x + b cos x ) + e x ( a cos x + b cos x ) dx
= y x
= –
1
….(i)
dx dy x y x
dy ysin   sin  
– y = e x ( −a sin x + b cos x ) y y
dx x
Again on differentiating both sides w.r.t. x, we get- Now, replace, v = , x = vy
y
d 2 y dy
– = e x ( –a cos x – b sin x ) + e x ( –a sin x + b cos x ) dx dv
dx 2 dx = v.1 + y in equation (i), we get-
dy dy
d 2 y dy dy
– +y= –y dv
v+ y = v–
1
dx 2 dx dx dy sin v
d2 y dy
2
– 2 + 2y = 0 y
dv
=–
1
dx dx dy sin v
253. The differential equation of the family of
dy
curves r2 = a2 cos 2θ θ where 'a' is an arbitrary – ∫ sin v dv = ∫ (On
constant is : y
dr dr integrating)
(a) = r cot 2θ (b) = r tan 2θ cos v = log y + c
dθ dθ
dr dr x
(c) = r sin 2θ (d) cos 2θ ⋅ + r sin 2θ = 0 cos   = log y + c …..(ii)
dθ dθ  y
AP EAMCET-24.04.2018, Shift-II π
Given at x = , y = 1, then form equation (ii)
Ans. (d) : Given, 4
r2 = a2 cos2θ π
On differentiating both sides, w.r.t. 'θ', we get– cos   = log (1) + c
4
dr
2r = −2a 2 sin 2θ  1 
dθ c = 
dr  r2   2
2r = –2   sin 2θ On putting the value of c in equation (ii), we get-
dθ  cos 2θ  x 1
dr cos   = log e y +
2r = –2r 2 tan 2θ  
y 2

Differential Equation 1170 YCT
255. If an edge of a cube measure 2 m with a (b) y = c1 {u ( x ) − v ( x )} + c 2 v ( x ) ,c 2 are arbitrary
possible error of 0.5 cm. Find the
corresponding error in the calculated volume constants
of the cube (c) y = c1u ( x ) v ( x ) + c 2 u ( x ) / v ( x ) ,c1 and c 2 are
(a) 0.6m3 (b) 0.06 m3 arbitrary constants
(c) 0.006 m3 (d) 0.0006 m3 (d) y = y(x) v(x)
Manipal UGET-2012 WB JEE-2014
Ans. (b) : Let x be the length of an edge of a cube and Ans. (a,b) : We know that u(x) and v(x) are two
V be the volume of that cube. independent solutions of the given differential equation
V = x3 then their combination is also the solution of the given
On differentiating w.r.t.x, we get equation.
dV Here, we see that y = 5u(x) + 8v(x) is a linear
= 3x 2 combination and y = c1{u(x) – v(x)} + c2v(x) is also, a
dx
linear combination of two independent solution.
Let δV be error in v and corresponding error δx in x.
258. The solution of the differential equation
dV
∴ δV = δx = 3x 2 δx dy y 1
dx + =
dx x log e x x
0.5
Given that, x = 2 m and δx = 0.5 cm = m under the condition y = 1 when x = e is
100 1 2
2  0.5  12 × 0.5 6 (a) 2y = log e x + (b) y = log e x +
∴ δV = 3 ( 2 )   = = = 0.06m3 log e x log e x
 100  100 100 (c) ylog e x = log e x + 1 (d) y = loge x + e
256. If a curve y = f(x) passes through the point (1, WB JEE-2014
dy Ans. (a) : Given, differential equation,
2) and satisfies x + y = bx 4 , then for what
dx dy y 1
2 + =
65 dx x log e x x
value of b, ∫ f(x)dx = ?
1
5 Form of linear equation,
(a) 5 (b) 10 dy
+ Py = Q
62 31 dx
(c) (d)
5 5 1
Where, P =
JEE Main 24.02.2021, Shift-II x log e x
Ans. (b) : Given, 1
dy and Q=
x + y = bx 4 x
dx
I.F. = e ∫
Pdx
dy y ∴
+ = bx 3
1
dx x ∫ dx
dx = e x loge x = elog(loge x ) = logex
1.F = e ∫ x = x ∴ Solution of differential equation,
5 1
∴ yx = ∫ bx 4dx =
bx
+c y × loge x = ∫ loge xdx
5 x
Above curve passes through (1, 2) (log e x) 2
y × loge x = +c .....(i)
b 2
2 = +c …..(i)
5 When, y = 1 and x = e
2  bx
4
c 62 (log e e) 2
also, ∫  +  dx = 1× log e e = +c
1
 5 x 5 2
1
b
× 32 + c log 2 −
b 62
= 1= +c
25 25 5 2
……(ii) 1
c=
By solving (i) and (ii) 2
c = 0, b = 10 1
On putting c = in equation (i),
257. If u(x) and u(x) are two independent solutions 2
d2 y dy (loge x) 2 1
of the differential equation 2
+ b + cy = 0, y × loge x = +
dx dx 2 2
Then additional solution (s) of the given 1
differential equations is (are) 2y = log e x +
loge x
(a) y = 5u(x) + 8v(x)
Differential Equation 1171 YCT
−x
259. If y = cos −1 x then it satisfies the differential 261. If y = e cos 2x, then which of the following
d2y
differential equation is satisfied?
equation (1 − x 2 ) 2 − x = c, where c is equal to
dy
dx dx d2 y dy
(a) 2
+2 + 5y = 0
(a) 0 (b) 3 dx dx
(c) 1 (d) 2
d2 y dy
WB JEE-2014 (b) 2
+ 5 + 2y = 0
Ans. (d) : Given, dx dx
−1 2
y = cox x (c)
d y dy
− 5 − 2y = 0
dx 2
y = ( cos −1 x )
2 dx
d2 y dy
On differentiating both sides, (d) 2
+ 2 − 5y = 0
dx dx
−1
= 2 ( cos −1 x ) ×
dy
WB JEE-2015
dx 1 − x2 Ans. (a) : Given,
Again, differentiating both sides w.r.t. x, y = e–xcos2x .....(i)
 −1 1 ( − 2x )  dy
 1− x ×
2
− cos −1 x ×  Therefore, = e − x ( − sin 2x ) 2 + cos 2x ⋅ e − x ( −1)
d2 y
= −2  1 − x 2 2 (1 − x ) 
2 1/ 2

dy
dx
dx 2  
( ) = −2sin 2xe − x − y
2
 1− x2  dx
 
dy
 x cos x −1
+ y = −2sin 2xe − x
− 1 + dx
= −2  (1 − x 2 )1/ 2 
  d 2 y dy
 (1 − x 2 )  + = −2 sin 2x ⋅ e − x ( −1) + e − x 2 cos 2x 
dx 2 dx
 2x cos −1 x  = 2sin2x⋅e–x – 4y [from equation (i)]
d2 y  2 − 2 1/ 2 
= (1 − x ) 2
dx 2   d y dy
+
dy
= − − y − 4y
 (1 − x 2 )  dx 2 dx dx
d2 y dy d2 y dy
(1 − x 2 ) 2 = 2 + x + 2 + 5y = 0
dx dx dx 2
dx
2
2 d y dy dy
(1 − x ) 2 − x =2 262. General solution of y + by 2 = a cos x,
dx dx dx
Then, c = 2
0 < x < 1 is
260. The integrating factor of the differential
(a) y 2 = 2a ( 2b sin x + cos x ) + ce−2bx
equation
dy
dx
( )(
−1
+ 3x tan y − x 1 + y = 0 is
2 3
) 2

( )
(b) 4b 2 + 1 y 2 = 2a ( sin x + 2b cos x ) + ce −2bx
2 3
(a) ex (b) ex
(c) e 3x 2
(d) e 3 x3
( )
(c) 4b 2 + 1 y 2 = 2a ( sin x + 2b cos x ) + ce 2bx

WB JEE-2015 (d) y 2 = 2a ( 2b sin x + cos x ) + ce−2bx


Ans. (b) : Given, Here, c is an arbitrary constant.
= − ( 3x 2 tan −1 y − x 3 )(1 + y 2 )
dy WB JEE-2016
dx Ans. (b) : Given general solution,
= x (1 + y ) − 3x ( tan y )(1 + y )
dy 3 2 2 −1 2 dy
y + by 2 = a cos x, 0 < x < 1 .....(i)
dx dx
1 dy Let, y2 = z
= x 3 − 3x 2 tan −1 y
(1 + y ) dx
2
2y
dy dz
=
1 dy −1
dx dx
⋅ + 3x tan y = x 2 3
dy 1 dz
1 + y 2 dx y =
–1 dx 2 dx
Putting, tan y = t
1 dy dt 1 dz
= ∴ + by 2 = a cos x [using equation (i)]
1 + y dx dx
2
2 dx
dt dz
∴ + 3tx 2 = x 3 + 2by 2 = 2a cos x
dx dx
Now, I.F. = e ∫ Now, I.F. = e ∫ = e 2bx
3x 2 dx 3 2b dx
= ex
Differential Equation 1172 YCT
y 2 ⋅ e 2bx = ∫ 2a cos x ⋅ e 2bx dx x3
2a
= ∫ 4x 2 ⋅ dx + c = 4 ⋅
3
+c
y 2 ⋅ e 2bx = (sin x + 2b cos x)e 2bx + c Using B.C. y(0) + c
4b2 + 1
4
–1(1+0) = ( 0 ) + c
2 2 –2bx
(4b + 1)y = 2a(sinx + 2bcosx) + ce
263. If the solution of the differential equation 3
∴ c = –1
dy
x + y = xe x be xy = e x φ ( x ) + C, thenφ ( x ) is 4 3
dx x –1
4
equal to ∴ y(1+x2) = x 3 –1 or y = 3
(a) x +1 (b) x – 1 3 1 + x2
(c) 1 – x (d) x 4
WB JEE-2016 ×1 – 1
1/ 3
Ans. (b) : Given,
∴ y(1) =3 = = 1/6
1+1 2
The D.E 265. The general solution of
dy dy
x + y = x ⋅ ex sin y. = cos y(1 − xcos y)is
dx dx
Or
dy y
+ = ex (a) sec y = x – 1 – cex
dx x (b) sec y = x + 1 + cex
1 (c) sec y = x + ex + c
∴ ∫ ⋅dx
I.F = e x = e e = x log x
(d) sec y = x – ex + c
AP EAMCET-22.04.2019, Shift-I
∴ ∫
y ⋅ I ⋅ F = e x ⋅ I.F ⋅ dx + C
Ans. (b) : Given,
Or ∫
x

y ⋅ x = x ⋅ e x dx + C = x ⋅ e x – 1⋅ e x ⋅ dx + C sin y
dy
dx
= cos y(1 − x cos y)
Or xy = e (x–1) + C dy
Comparing with xy = ex⋅φ (x) + C, we get- sin y − cos y = − x cos 2 y
φ(x) = x – 1 dx
dy
264. Let y(x) be a solution of tan y ⋅ sec y − sec y = − x
dx
(1 + x 2 ) dxdy
+ 2xy – 4x 2 = 0and y ( 0 ) = −1. Then Put, secy = v
dy dv
y(1) is equal to sec y tan y =
1 1 1 dx dx
(a) (b) (c) (d) −1 dv
2 3 6 ∴ − v = −x .....(i)
WB JEE-2018 dx
Ans. (c) : Given the D.E. Here, P = –1

(1 + x2)
dy
+2xy–4x2 = 0 With y (0) = –1 ∫ Pdp = ∫ −1dx = –x
dx I.F = e–x
dy 2x 4x 2 Multiplying (i) by I.F.
Or + ⋅y = dv − x
dx 1 + x 2
1+ x 2
e− x − e v = − xe − x
2x dx
∫1+ x2
⋅ dx Integrating both side,
2x ve − x = ∫ − xe − x dx
∴ I.F = e 1+ x
∫ 2
⋅dx
= xe–x + e–x + c = e–x(x + 1) + c = (1 + x) + cex
2x secy = (1 + x) + cex
Now, Let I = ∫ 1+ x2
⋅ dx secy = x + 1 + cex
Put 1+x =t 2 266. If the general solution of the differential
∴ 2x⋅dx = dt equation cos 2 x
dy
+ y = tanx is
dt dx
∴ I= ∫ t
= log e t (a) e (b) 1
∴ I⋅F = e log e t
( )
= t = 1+ x 2 (c) –1 (d) 1/e
AP EAMCET-05.07.2022, Shift-I
4x 2 Ans. (a) : Given,
∴ y⋅ I⋅F = ∫ 1+ x 2
⋅ I ⋅ F ⋅ dx + c
cos2 x
dy
+ y = tan x
dx
4x 2
Or y⋅(1+x ) = 2
∫ (
1+ x2
) 2
⋅ 1 + x ⋅ dx + c dy
+ y sec 2 x = tan x ⋅ sec2 x
dx
Differential Equation 1173 YCT
Where, P = sec2x 1 t
2∫
or t– sec 2 ⋅ dt = I
∫ Pdp =tanx∫ sec xdx = tanx
2
2
I.F = e 1 t
Multiplying equation (i) by I.F or t – .tan (2) = I
2 2
dy tan x t
e tan x + e y sec 2 x = e tan x ⋅ tan x ⋅ sec 2 x or I = t − tan
dx 2
Integrating both side,
(x + y)
ye tan x = ∫ e tan x tan x ⋅ sec 2 xdx or x + y – tan = x+c
2
On putting tanx = t Using B.C. we get –
sec2x dx = dt y(0) = 0
yet = ∫ te t dt = et(t – 1) + c 0 + 0 tan 0 = 0 + c
⇒ c=0
y = tanx – 1 + ce–tanx
(x + y)
π ∴ y = tan
If, y  = 1 2
4 268. Solve of the differential equation
1 = 1 – 1 + ce–1
1 = ce–1 dy 1 + y2
=
c=e dx (
tan −1 y − x )
267. The solution of the differential equation cos (x tan −1y −1
+ y) dy = dx given that y(0) = 0 is : (a) xe = e-tan y ((tan −1 y ) − 1) + c
x+y x+y (b) xe tan
−1y −1
= e tan y ((tan −1 y ) − 1) + c
(a) y = tan   (b) y = sin  
 2   2  −1y −1
(c) xe tan = e tan y ((tan −1 y ) + 1) + c
y x
(c) y = tan   (d) y = tan   −1y −1

2 2 (d) xe tan = e− tan y ((tan −1 y ) + 1) + c


AP EAMCET-18.09.2020, Shift-I AP EAMCET-21.09.2020, Shift-II
Ans. (a) : Given the P.E. Ans. (b) : Given the D.E.
cos (x + y). dy = dx with y(0) = 0
Put, x+y=t dy 1 + y2
=
dy dt dx tan −1 y − x
1+
dx dx dy dx
=
dy  dt  1 + y 2 tan −1 y − x
= – 1 Put tan–1y = t we get
dx  dx 
1
dy
= sec t =
dt
–1 dy = dt
dx dx 1 + y2
dt dx
= 1 + sec t So, we get dt =
dx t−x
dx
dt ∴ =t−x
∫ 1 + sec t = ∫ dx + c dt
dx
dt or +x=t
∫ 1 + sec t = x + c dt
I.F = e ∫
1.dt
cos t.dt ∴ = et
∫ 1 + sec t = x + c
Now, let
x⋅I. F = ∫ t.I.Fdt + c
cos t.dt or x.e t = ∫ t.e t dt + c = t.e t − ∫ 1.e t dt + c
∫ 1 + sec t = I(cos) = t. et – et + c = et (t –1) +c
cos t + t 1 −1 −1
⇒ ∫ (cos t + 1) dt – ∫ 1 + cos t ⋅ dt = I or x⋅ e tan y = e tan y (tan −1 y − 1) + c
269. The general solution of the differential
1  dy 
or ∫ dt – ∫ t
⋅ dt = I equation log   = ax + by is
1 + 2 cos 2 –1  dx 
2 (a) ae-by + beax = c (b) aeax + be-by = c
1 -by ax
(c) ae – be = c (d) ae2x + be-ax = c
or ∫ dt – ∫ t
⋅ dt = I
AP EAMCET-21.09.2020, Shift-II
1 + 2 cos 2 –1
2 MHT CET-2013 / BCECE-2009
Differential Equation 1174 YCT
Ans. (a) : Given the D.F 1 1
(c) e –x + x 3 (d) e x + x 3
 dy  2 3
log   = ax + by
 dx  WB JEE-2022
dy Ans. (d) : Given the
= e(
ax + by )
= a ax .e by
dx dy
cos y. = e x + sin y + x 2 .esin y
dx
∫ e − by dy = ∫ eax dx + c
= e x .esin y + x 2 esin y = ( e x + x 2 ) .esin y
 1 1
e − by  −  = eax   + c
 b  a ∫ cos y e − sin y dy = ∫ ( e x + x 2 ) dx + c
ax − by
e e Now, L.H.S
or + =c
a b
or b. eax + a. e–by = c
= ∫ cos y.e − sin y dy = I(let)
Put, sin y = t
dy x− y+3 cos y dy = dt
270. On solving = , the solution
dx 2x − 2y + 5
∫e
−t
obtained is x = 2 (x – y) + log (t) + c, find t dt = −e − t
(a) x – y + 2 (b) x + y – 2
(c) x + y + 2 (d) x – y – 2 x3
−e − t = e x + +c
AP EAMCET-21.09.2020, Shift-II 3
Ans. (a) : Given P.E. x3
dy x − y+3 −e − sin y = e x + + C1
= 3
dx 2x − 2y + 5 x 3
e x + e − sin y + + C1 = 0
dy ( x − y) + 3 3
Or , =
dx 2 ( x − y ) + 5 x 3
e x + + e − sin y = −C1 ≡ f ( x ) + e − sin y = c Q c = −C1
Put, x–y =z 3
dy dz x3
∴ 1− = f(x) = ex +
dx dx 3
−1
dy dz 272. If y = e tan x then
∴ = 1−
dx dx (a) (1 + x2)y2 + (2x – 1)y1 = 0
Or, (b) (1 +x2) y2 + 2xy = 0
dz z+3 (c) (1 – x2) y2 – y1 = 0
1− = (d) (1 + x2)y2 + 3xy1 + 4y = 0
dx 2z + 5 WB JEE-2022
dz z + 3 2z + 5 − z − 3 z+2 Ans. (a) : Given,
∴ = 1− = =
dx 2z + 5 2z + 5 2z + 5 y = e tan x
−1

2z + 5
∴ ∫ z + 2 dz = ∫ dx dy −1
= e tan x
1
Or, dx 1+ x2
or (1 + x 2 )
dy −1
 2z + 4 1  = e tan x = y
∫  z + 2 + z + 2  .dz = x + C1 dx
2
Or,
1
(1 + x 2 ) dx 2 + dx
d y dy
( 2x ) =
dy
dx
2 ∫ dz + ∫ . dz = x + C1 or (1 + x2) y2 + 2x.y1 = y1
z+2
Or, or (1+x2)y2 + (2x –1) y1 = 0
2z + ln (z + 2) = x + C1 273. The solution of the differential equation
Or, dx
x = 2(x – y) + log (x – y + 2) + c, Q C = −C1 + 2yx = 2y which passes thorough the point
dy
where is integration constant comparing, we get (2,0) is
t= x–y +2 2 2
(a) (x – 1) = 2e y (b) (x – 1 = 2e − y
dy
= e x+sin y + x 2esin y is f(x)
2 2
271. The solution of cos y (c) (x – 1) = e y (d) (x – 1) = e − y
dx
+ e–siny = C (C is arbitrary real constant) where AP EAMCET-23.04.2018, Shift-II
f(x) is equal to Ans. (d) : Given the D.F
1 1 dx
(a) e x + x 3 (b) e –x + x 3 + 2yx = 2y with y (2) = 0
2 3 dy

Differential Equation 1175 YCT


dx dz dx
or
dy
= 2y(1 − x) or ∫ 1+ z 2
=∫
x
+c
1 = lnx + c

or 2ydy = ∫ x −1
dx
or ln z + 1 + z 2 = ln x + c
y2 = ln k⋅x
− ln ( x − 1) = 2. + c = y 2 + c
2 ∴ z + 1+ z = k⋅ x
2
Using B.C y (2) = 0
2
ln (2 – 1) = 0 + c y  y
c=0 or + 1+   = k ⋅ x
x  x
1
y 2 = ln y + x2 + y2 = k ⋅ x2
( x − 1) or
1
or
1
= ey
2
or x2 = y + x 2 + y2 
x −1 k 
x 2 = c  y + x 2 + y 2  , where c is a constant.
2
or (x – 1) = e − y or
 
dy ∴ Option (a) is correct.
274. The integrating factor of x + 3y = x is
2

dx 276. The differential equation of all non-vertical


3 lines in a plane is:
(a) (b) log x
x d2 y d2x
(c) x 3
(d) x (a) 2
= 0 (b) =0
dx dy 2
AP EAMCET-17.09.2020, Shift-II
dy dx
Ans. (c) : Given the D.F (c) =0 (d) =0
dx dy
dy
x + 3y = x 2 AMU-2013 / JCECE-2006
dx Jamia Millia Islamia-2005
dy 3
or + y=x Ans. (a) : Q The family of all non-vertical line is–
dx x y = mx + c
3
I.F = e ∫ x
dx
r
Where, m = tan
1 2
= e∫x
3 dx
On differentiating w.r.t x,
3
= e3ln x = eln x = x 3 dy
=m
275. Solve the differential equation given below dx
xdy Again, differentiating w.r.t x,
= y + x2 + y 2 d2 y
dx =0
dx 2
(a) x 2 = c  y + y 2 + x 2 
  277. The solution of the differential equation y dx +
2  2 2  (x + x2y) dy = 0 is
(b) y = c x + y -- x
  1 1
(a) − =c (b) − + log y = c
(c) y = c  x + tan
2
 ( -1
)
1+ y2 

xy xy
1
2 
(d) y = c x -- y + x 2 2  (c) + log y = c (d) log y = cx
  xy
AP EAMCET-22.09.2020, Shift-I Jamia Millia Islamia-2007
Ans. (a) : Given the D.E. Ans. (b) : ydx = –(x2y + x)dy
dy ydx + xdy = x2ydy
x⋅ = y + x 2 + y2 ydx + xdy −dy
dx =
2 (xy) 2 y
dy y  y
or = + 1+   d
dx x  x (xy)
dx −dy
y 2
=
Put, = z or y = z⋅x (xy) y
x 1
dy dz − = − log y + k
∴ = z+ x⋅ xy
dx dx 1
dz − + log y = c
∴ z+ x⋅ = z + 1+ z 2
xy
dx
Differential Equation 1176 YCT
278. The particular solution of the differential = −200 ∫ e − t / 2 ⋅ dt + c
equation log (dy/dx) = 3x + 4y at x = 0 and y = 0
is = ( −200 ) ⋅ e − t / 2 ⋅ ( −2 ) + c
(a) 4e3x – 3e–4y = 7 (b) 4e3x + 3e–4y = 0
3x –4y or p ⋅ e− t / 2 = 400 ⋅ e − t / 2 + c
(c) 4e + 3e = 7 (d) 4e3x – 3e–4y = 0
J&K CET-2019 Now, using B⋅C. P (0) = 100
Ans. (c) : Given the D.E. ∴ 100 ⋅ e0 = 400 ⋅ e0 + c
∴ c = – 300
 dy 
log   = 3x + 4y ∴ P = 400 – 300⋅ et/2
 dx  ∴ option (a) is correct.
dy
∴ = e(3x + 4 y ) = e3x . e 4y 281. If(2 + sin x)
dy
+ ( y + 1) cosx = 0 and y(0) = 1,
dx dx
∴ ∫ e dy = ∫ e .dx + c
−4 y 3x
π
then y   is equal to
 −1  1 2
e −4 y .   = e3x   + c
 4  3 (a)
1
(b) –
2
Or, 3 3
−3.e −4y = 4e3x + c 1 4
Or, (c) – (d)
3 3
4.e3x + 3. e–4y = c JEE Main-2017
Using the B.C. Y (0) = 0 Assam CEE-2017
We get,
Ans. (a) : Given the differential equation,
4. e0 + 3. e0 = c
dy
∴ c=4+3=7 (2 + sinx) + ( y + 1) cos x = 0
∴ We get dx
4. e3x + 3. e–4y = 7 dy cos x
Or + ( y + 1) ⋅ =0
279. If f(x) is a differentiable function, them dx 2 + sin x
af(x) – xf(a) dy cos x cos x
lim is equal to Or + ⋅y = –
x→a x–a dx 2 + sin x 2 + sin x
cos x
(a) a f’(a) – f(a) (b) a f(a) – f’(a)
Now,I.F = e 2+sin x
∫ ⋅dx
(c) a f’(a) + f(a) (d) f’(a) + a f(a)
J&K CET-2003 cos x
Ans. (a) : Given the limit
Now, Let ∫
2 + sin x
dx = I

lim a f ( x ) − x f ( a ) 0 Put 2 + sinx = t


x →a Which is from ∴ cosx⋅dx = dt
x −a 0
dt
∴ Using L, Rule we get ∴ I=
t ∫
= l n t = l n ( 2 + sin x )
lim a f ' ( x ) − f ( a )
I⋅F = e (
l n 2+sin x )
x →a ∴ = 2 + sin x
1
– cos x
= af' (a) – f (a)
280. Let the population of rabbits surviving at a
y⋅I⋅F = ∫
2 + sin x
⋅ I ⋅ F ⋅ dx + c

time t be governed by the differential equation cos x


dp(t) 1
= P(t) – 200. If p(0) = 100, then p(t) is
=– ∫
2 + sin x
⋅ ( 2 + sin x ) dx + c

dt 2 = − ∫ cos x ⋅ dx + c
equal to = – sinx + c
(a) 400 – 300 et/2 (b) 300 – 200 e–t/2
(c) 600 – 500 e t/2
(d) 400 – 300 e –t/2 ∴ y (2 + sinx) = – sinx + c
JEE Main-2014 Now, using B.C. y (0) = 1
Ans. (a) : Given, the D⋅E. 1 ⋅ ( 2 + sin 0 ) = − sin 0 + c
dp ( t ) 1 or 2=c
= P(t) − 200 ∴ y (2 + sinx) = – sinx + 2
dt 2
dp 1 π  π π
or − ⋅ P = −200 ∴ y   = y  2 + sin  = − sin + 2
dt 2  2  2  2
1 1 = – 1 + 2 = 1
I ⋅ F⋅ = e ∫ 2 = e 2 = e − t / 2
− ⋅dt − ⋅t

π 1 1
∴ P ⋅ I ⋅ F⋅ = ∫ ( −200 ) ⋅ I ⋅ F ⋅ dt + c ∴ y  = =
 2  2 +1 3
Differential Equation 1177 YCT
282. Consider the differential equation, 283. If y = y(x) is the solution of the differential
 1 dy
+ 2y = x 2 satisfying y(1) = 1, then
y 2 dx +  x –  dy = 0 . If value of y is 1 when x equation, x
 y  dx
= 1, then the value of x for which y = 2, is 1
y   is equal to
5 1 3 1 2
(a) + (b) –
2 e 2 e 13 1
(a) (b)
1 1 3 16 4
(c) + (d) – e 49 7
2 e 2 (c) (d)
JEE Main 12.04.2019, Shift-I 16 64
Ans. (b) : Given, the differential equation JEE Main 09.01.2019, Shift-I
2 Karnataka CET-2015 / Kerala CEE-2006
y dx + (x – 1/y) dy = 0
Ans. (c) : Given the differential equation
1  dy
or y dx =  − x  dy
2
x ⋅ + 2y = x 2 with y(1) = 1
y  dx
dy dx or
or = dy 2
y2  1  + ⋅y= x
 − x dx x
y  2
⋅ dx

∴ I.F = e ∫
x
Put,
1 2
= t or y–1 = t = 2eln x = eln x = x 2
y
∴ y.I.F = ∫ x ⋅ I ⋅ Fdx + c
1
∴ − 2 dy = dt
y x4
y ⋅ x 2 = ∫ x ⋅ x 2 ⋅ dx + c = + c
dy 4
∴ = − dt 2
y 2 x c
Or y= + 2,
dx dx 4 x
∴ −dt = or =x− t Now using B.C. y(1) = 1
( t − x ) dt We get -
or 1
dx 1= + c
−x =− t 4
dt 3
∴ c=
∴ I.F. = e = e ∫ dt −t
4
x2 3
∴ x . IF. = ∫ − t ⋅ I.Fdt + c ∴ y= + 2
4 4x
= ∫ − t ⋅ e − t dt + c
 1  (1/ 2 )
2
3/ 4 1 3 1 49
y   = + = + ×4 =3 + =

= − ∫ t ⋅ e ⋅ dt + c  (1/ 2 )
−t 2
 
2 4 16 4 16 16
 
284. If y(x) is the solution of the differential
= −  − t ⋅ e − t + ∫ 1 ⋅ e − t dt  + c
  dy  2x + 1  –2x
equation +  y = e , x > 0 where y(1)
= −  − t ⋅ e − t − e − t  + c = t ⋅ e − t + e − t + c dx  x 
= x ⋅ e − t = e − t ( t + 1) + c 1
= e –2 , then
2
or x = ( t + 1) + c ⋅ e t
1 
(a) y(x) is decreasing in  ,1
1  2 
or x =  + 1 + c ⋅ e 1/ y
....(i)
 y  (b) y(x) is decreasing in (0, 1)
Put, y (1) = 1 (c) y(log e 2) = log e 4
∴ 1 = (1 + 1) + c. e1 log e 2
(d) y ( log e 2 ) =
∴ c.e = –1 4
∴ c = –1/e JEE Main 11.01.2019, Shift-I
∴ From equation (i), put y = 2 Ans. (a) : Given the differential equation,
1  1 3 1 dy  2x + 1  −2x 1 −2
x =  +1 − ⋅ e1/ 2 = − +  y = e , x > 0 with y(1) = ⋅ e
2  e 2 e dx  x  2
Differential Equation 1178 YCT
2x +1 y(0) = 0
I.F. = e ∫
⋅dx
x
Here, ∴ 0 = (tan0 – 1) + c⋅e–tan0
 1
∫  2 + dx = (0 – 1) + c⋅e–0
= e  x = –1 + c
= e 2x + ln x = e 2x ⋅ e ln x = x ⋅ e 2x ∴ c=1
∴ y ⋅ I.F. = ∫ e −2x
⋅ x ⋅ e dx + c
2x ∴ y = (tanx – 1) + 1⋅e–tanx
 −π 
 −π    −π   − tan  4 
y ⋅ x ⋅ e = ∫ x ⋅ dx + c =
2x x2
+c
∴ y  =  tan   − 1 + e
2  4    4  
= –2 + e = e – 2
x2
y.xe 2x = 286. If the curve y = y(x) represented by the solution
2 of the differential equation (2xy2 – y) dx + xdy
1 −2 1 = 0, passes through the intersection of the lines
∴ y (1)= e ⋅ 1 ⋅ e2 = + c ⇒ c = 0
2 2 2x – 3y = 1 and 3x + 2y = 8, then |y(1)| is equal
x2 to ........ .
x JEE Main 25.02.2021, Shift-II
or y = 2 2x = ⋅ e −2x Ans. (1) : Given the differential equation,
x ⋅e 2
∴ (2xy2 – y)dx + xdy = 0
dy 1  dy y − 2xy 2 y
= e −2x  − x  or = = (1 − 2xy)
dx 2  dx x x
dy 1 1 y
<0⇒ −x < 0⇒ x > Put, = z or y = xz
dx 2 2 x
1  dy dz
∴ y(x) is decreasing in  ,1 . ∴ = z + x⋅
 2  dx dx
285. If y = y(x) is the solution of the differential dz
∴ z + x⋅ = z(1 − 2x ⋅ zx)
dy  π π
equation = ( tanx – y ) sec 2 x, x ∈  – ,  such dx
dx  2 2 = z – 2x2z2
 π dz
y–  = −2xz 2
that y(0) = 0, then  4  is equal to dx
dz
1
(a) – 2 (b)
1
–e ∴ ∫ −2z 2 = ∫ x ⋅ dx + c
e 2
1 1 z −2 +1 x 2
(c) 2 + (d) e – 2 − ⋅ = +c
e 2 −2 + 1 2
JEE Main 10.04.2019, Shift-I 1 1 x2
or ⋅ = +c
Ans. (d) : Given the differential equation, 2 z 2
dy  −π π  1
= (tan x − y)sec 2 x, x ∈  ,  with y(0)=0 or = x2 + c
dx  2 2  z
∴ We get– x
dy or = x2 + c
+ y ⋅ sec x = tan x ⋅ sec x
2 2
y
dx
x
∴ I.F. = e ∫
sec2 x ⋅dx
= e tan x or y= 2 .....(i)
x +c
∴ y ⋅ I.F. = ∫ tan x ⋅ sec2 x ⋅ I.F.dx + c Given the lines,
2x – 3y = 1 and 3x + 2y = 8
= ∫ tan x ⋅ sec 2 x ⋅ e tan x ⋅ dx + c Solving, 2x – 3y = 1
Put, tanx = t or 6x – 9y = 3
∴ sec2x⋅dx = dt 6x + 4y = 16
∴ y ⋅ e tan x = ∫ t⋅ e t ⋅ dt + c – – –
–13y = –13
= t ⋅ e t − ∫ 1 ⋅ e t ⋅ dt + c ∴ y=1
t t and 3x + 2 = 8
= t⋅e – e + c
t
= e (t – 1) + c or 3x =6
tanx
= e (tanx – 1) + c ∴ x = 2
∴ y = [tanx – 1) + c⋅e –tanx ∴ B.C. is y(2) = 1
Differential Equation 1179 YCT
Applying B.C. we get– dx x
= + 2y3
2 dy y
1= or 4 + c = 2
4+c dy x
or c = –2 – = 2y 2
dx y
x Comparing with
∴ y= 2
x −2 dy
1 + Px = Q
∴ y(1) = = −1 dx
1− 2 1
∴ |y(1)| = 1 P = – , Q = 2y2
y
287. Let y = y(x) be the solution of the differential 1
∫ – dy
I.F.= e ∫
equation Pdy 1
= e y = e–l n y =
dy y
= 2(y + 2sinx – 5)x – 2cos x such that, y(0) =

dx x ⋅ I ⋅ F = I ⋅ F ⋅ Qdy + A {A = constant}
7. Then y(π) is equal to
2 2
(a) 2e π + 5 (b) e π + 5 1 1
2
(c) 3e π + 5
2
(d) 7e π + 5
x⋅ =
y y ∫
⋅ 2y 2dy + A

x
Ans. (a) : Given,
JEE Main 27.08.2021, Shift-I
y
= 2ydy + A ∫
dy x
= 2 ( y + 2sin x − 5) x − 2cos x = y2 + A
dx y
and y (0) = 7 x = y3 + Ay
dy 289. Let f be twice differentiable function such that
+ 2cos x = 2 ( y + 2sin x − 5) x …(i) f"(x) = –f(x) and f'(x) = g(x), h(x) = {f(x)}2 +
dx
Let y + 2 sinx – 5 = t {g(x)}2. If h(5) = 11, then h(10) is equal to
dy dt (a) 22 (b) 11
+ 2cos x = (c) 0 (d) 1
dx dx Assam CEE-2022
Then equation (i)
Ans. (b) : We have,
dt dt h(x) = [f(x)]2 + [g(x)]2
= 2tx ⇒ = 2x dx
dx t h'(x) = 2f(x)⋅f'(x) + 2g(x)⋅g'(x)
On Integrating both sides h'(x) = 2f(x)⋅g(x) + 2g (x) g'(x) = 0
2
lnt = x + c Thus, h(x) is a constant function
2
ln (y + 2 sinx – 5) = x + c ….(ii) but h(5) = 11
y (0) = 7 h(x) = 11
ln (7 + 0 – 5) = 0+ c h(10) = 11
c = ln2
290. Let y = y(x) be the solution of the differential
∴ From equation (ii)
( )
2
ln (y + 2 sinx – 5) = x2 + ln2 dy 5 x5 + 1
Now, at x = π, ln [y (π) + 2 sinπ–5] = π + ln22 equation + y = , x > 0 . If
2
dx x x5 + 1 x7 ( )
y(π) – 5 = e π +ln 2 = e π eln 2
2

y(1) = 2, then y(2) is equal to


y ( π ) = 2e π + 5
2
637 679
(a) (b)
dy 128 128
288. Solution of the equation (x + 2y3) – y = 0 (y 693 697
dx (c) (d)
> 0) is 128 128
(a) x = y3 + Ay (b) y(1 – xy) = Ax JEE Main-11.04.2023, Shift-II
(c) x (1 – xy) = Ay (d) x(1 + xy) = Ay Ans. (c) : Given,
Assam CEE-2021
dy 5 (x 5 + 1) 2
Ans. (a) : Given, + y= ,x > 0
dx x(x + 1)
2
x7
( x + 2y3 ) dx
dy
−y=0 And, y(1) = 2
∴ Given differential equation is linear
dy y
= dy 5 (x 5 + 1)2
dx x + 2y3 + y= ,x > 0
dx x(x + 1)5
x7
dx x + 2y3
= 5
∫ x (x5 +1)
5x 4 dx
∫ x5 ( x5 +1)
dy y IF = e dx = e
Differential Equation 1180 YCT
Let, x5 = t, cos x
5x4 dx = dt
dt
R.H.S =
x ∫⋅ dx
∫ t (t +1)
IF = e Put, x = 4 or x = 42
 t 
ln  
 x5 
lin  5  ∴ dx = 24du
 t +1   x +1 
= e =e cos x cos 4
x5
IF = 5
∴ ∫ x
dx = ∫
4
⋅ 24 ⋅ du = 2∫ cos 4du
x +1 = 2. sin 4
 x5  = 2 sin x
Now,  5  y(x)
 x +1  −1 y2
1

∴ e = 2sin x + c
x 5 (x 5 + 1) 2 2
= ∫ x 5 + 1. x 7 dx 1
y2
∴ e = −4sin x + c
x5 + 1  1 
= ∫ 2 dx = ∫  x 3 + 2  dx ∴ Taking by both the sides,
x  x 
 x5  x4 1
1
y2
= log e −4sin x + c ( )
=  5  y(x) = – +C
 x +1 4 x Using B.C y(0) = 1, we get-
Where, x = 1 and y = 2 e = –4 . sin + + c = c = e

Then, C =
7
4

1
y2
= log e e − 4sin x ( )
 x 
5
x4 1 7 f(x) = e − 4sin x
∴  5  y(x) = – + …(i)
 x +1 4 x 4
292. If
dn y
= y n and y = e x
+ e– x
, then 4xy2 + 2y1
On putting x = 2 in equation (i) we get – dxn
 32  21 =
  y(2) = (a) –y (b) y
 33  4
(c) 2y (d) –2y
693
y(2) = AP EAMCET-05.07.2022, Shift-II
128 Ans. (b) : Given,
dx y 3cos x 1 dn y
291. If the solution of = 1/y 2
, y(0) = 1 is 2 = = y n and y = e x
+ e– x
dx xe y dx n
loge(ƒ(x)), then ƒ(x) = 1  –1 
(a) 4 + 4 sin x (b) e sin x y1 = e x
× + e– x
× 
2 x 2 x 
(c) 1 – 4 x (d) e–4 sin x 1 
AP EAMCET-08.07.2022, Shift-II = e x
– e– x 
2 x 
Ans. (d) : Given,
2 xy1 = e x – e – x
dy y3 cox x
= 1/ y2 ⋅ Again differentiate, we get –
dx e x
1
1
2
1
2 2 x y 2 + 2y1 ×
e y dy e y cos x 2 x
Now, L.H.S = ∫ 3 ∫ 3 ⋅ dy = ∫ dx + c
y y x 1  1 
= e x
× – e– x
 
1 2 x 2 x 
Let, = t or y −2 = t
y2
4xy 2 + 2y1 e x
+ e– x

∴ −2 ⋅ y −3 ⋅ dy = dt = =
2 x 2 x
dy −1 4xy2 + 2y1 = y
or 3 = ⋅ dt
y 2 293. The solution of the differential equation y'
dy −1 1
= ⋅ dt = −y
y3 2 e −x
1 1 (a) x = e–y(y + c) (b) y + e–y = x + c
dy −1 t −1 −1 y2
∴ ∫e ⋅ = ∫ e ⋅ dt = e t =
y
(d) x + y = e–y + c
2
y
⋅e (c) x = e (y + c)
y3 2 2 2 TS EAMCET-2016
Differential Equation 1181 YCT
Ans. (a) : Given, 295. The general solution of the differential
Differential equation equation
1 dy
y ' = −y b = 1 + x + y + xy is
e −x dx
dy 1 x2
= −y (a) log(1 + x) = y + +k
dx e − x 2
We can write this equation in the form x2
(b) y = x + +k
dy 2
= e− y − x
dx x3
(c) log (1 + y ) = +k
dy 3
+ x = e− y x2
dx x+
(d) y = ke 2
–1
dy
Compare Equation (i) by + px = Q we get - CGPET-2021
dx
TS EAMCET-07.05.2018, Shift-I
P = 1 Q = e–y
Ans. (d) : Given,
∴ If = e ∫ e∫ = y
pdy 1dy
dy
= 1 + x + y + xy
Solution y given differential equation is dx
x.e y = ∫ e y ⋅ e− y ⋅ dy + c dy
= (1 + x) + (1 + x)
x ⋅ e y = ∫ 1dy + c dx
dy
x ⋅ ey = y + c = (1 + x) (1 + y)
dx
x = e− y ( y + c ) dy
∫ = (1 + x ) dx
294. The general solution of the differential 1+ y ∫
dy xy + x – 2y – 2
equation = is x2
dx xy − 2x + y − 2 log(1+ y) = +x+k
2
x +1 x2
(a) x + y + 3log =c y = ke
x+
−12
y +1
296. The solution of the differential equation
y +1
(b) x + y + 3log =c dy
x +1 = 1 − cos ( y − x ) cot ( y − x ) is
dx
x+1 (a) x tan (y – x ) = c (b) x = tan (y – x ) + c
(c) x – y + 3log =c
y+1 (c) x = sec (y – x) + c (d) x + sec (y – x) = c
y +1 TS EAMCET-04.05.2019, Shift-I
(d) x – y + 3 log =c Ans. (d) : Given,
x +1
Differential equation
TS EAMCET-18.07.2022, Shift-II
dy
Ans. (d) : Given, = 1 − cos ( y − x ) cot ( y − x )
Equation of differential dx
Let, y – x = v
dy xy + x − 2y − 2
= Differential w.r.t x on both sides
dx xy − 2x + y − 2
dy d ( x ) dv
dy (x − 2)(y + 1) − =
= dx dx dx
dx (x + 1)(y − 2)
dy dv
y−2 x−2 −1 =
dx dx
∫ y + 1 dy =∫ x + 1 dx dy dv
= 1+
 3   3  dx dx
∫ 1 − y + 1  dy =∫ 1 − x − 1  dx dv
1 − cos v cot v == 1 +
y – 3ln |y + 1| = x – 3ln |x + 1| + C dx
y +1 cos v dv
x − y + 3ln =C − cos v × =
x +1 sin v dx

Differential Equation 1182 YCT


− cos 2 v dv (b) log (1 − y ) = log (1 + y ) + y + C
=
sin v dx x2
sin v (c) log (1 + y ) = x − +C
∫ dx = ∫ − cos2 v ⋅ dv 2
x2
sin v 1 (d) log (1 + y ) = x + +C
∫ dx = − ∫ cos2 v ⋅ cos v ⋅ dv 2
MHT CET-2022
∫ 1⋅ dx = −∫ tan v ⋅ sec v ⋅ dv Ans. (c) : Given,
dy
x + c = – sec v (v=y–x) =1− x + y − xy
sec v + x = c dx
dy
x + sec ( y –x) = c =1(1 − x ) + y (1 − x )
297. If the solution curve of the differential equation dx
(y– 2loge x)dx + (xloge x2) dy = 0, x > 1 passes dy
= (1 − x ) (1 − x )
 4 dx
through the points  e,  and (e4, α), then α is
 3 1
dy = (1 − x ) dx
equal to____ 1+ y
JEE Main-08.04.2023, Shift-I 1
Ans. (3) : Given, ∫ 1 + y dy = ∫ (1 − x ) dx
differential equation
(y – 2log ex) dx + ( x logex2) dy = 0 x2
log (1 + y ) = x − + C
dy 2
2x log x + y = 2 log e x 299. If y = 3 cos(logx) + 4 sin (logx), then x2y2 + xy1 =
dx
(a) –xy (b) –y
dy y 1
+ = (c) y (d) xy
dx 2x log e x x MHT CET-2022
1
1 log ( log x ) Ans. (b) : Given,
I.F = ∫ dx = e 2 y = 3cos (logx) + 4 sin (logx)
e 2x log e x
1 1
= log x y1 = − 3sin ( log x ) ⋅ + 4cos ( log x ) ⋅
x x
y. log e x = ∫
1
log xdx + c 3sin ( log x ) 4cos ( log x )
x y1 = − + .....(i)
x x
( log x )
3/ 2

y. log e x = +c  1 
3/ 2  x ⋅ cos ( log x ) ⋅ x − sin ( log x ) 
y2 = − 3  
 4  x2 
It passes through  e, 
 3  
4 2  1 
= +c  − x sin ( log x ) ⋅ x − cos ( log x ) 
3 3 +4  
2  x2 
c=  
3
cos ( log x ) − sin ( log x ) 
It also, pusses through (e4 α) y2 = − 3 
16 2 x2
α.2 = + = 6
3 3  − sin ( log x ) − cos ( log x ) 
+4  
16 2  x2 
α.2 = + = 6
3 3 x2 y2 = – 3 cos (logx) + 3sin (logx) – 4sin (logx) –
α=3 4cos(logx)
298. The general solution of the differential x2y2 = – [3cos(logx) + 4sin(logx)] + 3 sin(logx) –
dy 4cos(logx)
equation
dx
= 1 – x + y – xy (where C is a  −3sin ( log x ) 4cos ( log x ) 
x 2 y2 = − y − x  + 
constant of integration.)  x x 
x2 2
x y2 = – y – xy1 {from equation}
(a) log (1 + y ) = y − + C
2 x2y2 + xy1 = – y
Differential Equation 1183 YCT
300. Let y = y(x) be the solution of the d ifferential (a) 2 – 2 (b) 2 + 2
equation, xy' – y = x2 (x cosx + sinx), x > 0. If
1
π π (c) 2–2 (d) –1
π) = π, then y''   + y   is equal to
y(π 2
2 2
JEE Main 05.09.2020, Shift-II
π π2 π Ans. (c) : Given the differential equation,
(a) 2 + + (b) 1+
2 4 2 dy  π
π π2 π cos x ⋅ + 2y sin x = sin 2x, x ∈  0, 
(c) 1+ + (d) 2 + dx  2
2 4 2 dy
JEE Main 04.09.2020, Shift-I or + 2 tan x ⋅ y = 2 sin x
dx
Ans. (d) : Given the differential equation,
I.F. = e ∫
2 tan x ⋅dx
xy′ – y = x2(xcosx + sinx), x > 0 ∴ = e 2ln sec x = sec2 x

or
y
y′ − = x(x cos x + sin x) ∴ y ⋅ sec x = ∫ 2 sin x ⋅ sec 2 x dx + c
2

x
1 = 2 ∫ tan x sec x dx + c = 2secx + c
I.F. = e ∫ x = e − ln x = e ln x =
− ⋅dx −1 1
x ∴ y = 2cosx + c cos2x
∴ y ⋅ I.F. = ∫ Q ⋅ I.F.dx + c π
Now, using boundary condition y   = 0
3
= − cos x + ∫ x cos x dx + c  π π
 
0 = 2 cos   + c ⋅ cos 2  
Now, ∫ x ⋅ cos x ⋅ dx = x ⋅ sin x − ∫ 1 ⋅ sin x ⋅ dx  3   3
π
 
= xsinx – (–cosx) or 0 = 2 + c ⋅ cos  
= xsinx + cosx 3
1 c = –4
∴ y ⋅ = x sin x + cos x − cos x + c ∴ y = 2cosx – 4cos2x
x
= xsinx + c π 1 1
∴ y  = 2⋅ − 4⋅
∴ y = x2sinx + c⋅x 4 2 2
Given boundary condition y(π) = π = 2 −2
∴ π = π2⋅sinπ + c⋅π ∴ Options (c) is correct.
π = 0 + c⋅π
302. If y = y(x) is the solution of the differential
c=1
∴ 2
y = x + x sinx
dy π
equation, + 2y tan x = sinx, y   = 0, then
∴ y′ = 1 + 2x⋅sinx + x ⋅cosx2 dx 3
d d the maximum value of the function y(x) over R
∴ y′′ = 0 + [ 2x ⋅ sin x ] +  x 2 cos x  is equal to
dx dx
= 2⋅sinx + 2x⋅cosx + 2x⋅cosx + x2(–sinx) 1
2
(a) 8 (b)
= sinx (2 – x ) + 4xcosx 2
 π π  π2  π π 15 1
∴ y′′   = sin ⋅  2 −  + 4 ⋅ ⋅ cos (c) – (d)
2 2  4  2 2 4 8
JEE Main 16.03.2021, Shift-I
π2 π2 Ans. (d) : Given differential equation,
= 2− +0 = 2−
4 4 dy π
2 + 2y ⋅ tan x = sin x, boundary condition, y   = 0
π π  π π π π2 dx 3
y   = +   sin = +
2 2 2 2 2 4 2 ∫ tan x ⋅dx x
∴ I.F. = e = e2⋅ln sec = sec 2 x
 π  π π 2 π π2
∴ y′′   + y   = 2 − + + ∴ y ⋅ sec 2 x = ∫ sin x ⋅ sec 2 x dx + c
2 2 4 2 4
π = ∫ tan x sec x dx + c
= 2+
2
∴ Option (d) is correct. = secx + c
301. Let y = y(x) be the solution of the differential ∴ y = cosx + c⋅cos2x .....(i)
dy  π π
equation, cosx + 2y sinx = sin2x, x∈  0,  . If Given, y  3  = 0
dx  2  
π π π π
y   = 0, then y   is equal to 0 = cos   + c ⋅ cos 2  
3 4  3   3
Differential Equation 1184 YCT
π 1 304. Let y = y(x) be the solution of the differential
or 0 = 1 + c ⋅ cos   = 1 + c ⋅ equation
 3 2
dy
= (y + 1) ( y + 1) e x /2 – x  , 0 < x < 2.1, with
2
c  
= −1 dx
2
or c = –2 dy
y(2) = 0. Then the value of at x = 1 is equal
Putting the value of c in equation (i), we get– dx
y = cosx – 2cos2x to
dy –e3/2 2e2
Now, = − sin x − 2 ⋅ 2 cos x ⋅ ( − sin x) (a) (b) –
(1+ e 2 )
2 2 2
dx (e +1)
= –sinx + 4sinxcosx = 0
–1 + 4cosx = 0, sinx = 0 e5/2 5e1/ 2
(c) (d)
1
(1 + e2 ) ( e2 + 1)
2 2
cos x =
4 JEE Main 18.03.2021, Shift-II
1 1 1 1 1
∴ max(y) = − 2 × = − = Ans. (a) : Given,
4 16 4 8 8 dy
= (y + 1) (y + 1)e x / 2 − x 
2

∴ Option (d) is correct.  


.....(i)
dx
303. If y = y(x) is the solution of the differential dy 2

dy π = (y + 1)2 e x / 2 − x(y + 1)
equation + ( tanx ) y = sinx, 0 ≤ x ≤ , with dx
dx 3 dy 2

π + x(y + 1) = (y + 1)2 e x / 2


y(0) = 0, then y   is equal to dx
4 1 dy 1 2
+ ⋅ x = ex / 2 .....(ii)
1  1  (1 + y)2 dx y + 1
(a) log e 2 (b)   log e 2
4 2 2 1
Let, =t
1 y +1
(c) loge 2 (d) log e 2 −1 dy dt
2 =
JEE Main 16.03.2021, Shift-II (y + 1)2 dx dx
Ans. (b) : Given the differential equation, dt 2
− + tx = e x / 2
dy π dx
+ tan x ⋅ y = sin x, 0 ≤ x ≤
dx 3 dt 2
+ ( − x)t = −e x / 2
I.F. = e ∫
tan x ⋅ dx dx
∴ = eln sec x = sec x
Which is linear differential equation,
∴ y ⋅ I.F. = ∫ Q ⋅ I.F. + c
I.F. = e ∫
− xdx 2
= e− x / 2
or y ⋅ sec x = ∫ sin x ⋅ sec x dx + c So, solution of differential equation is–
= − ∫ e− x
2 2 2
t ⋅ e−x /2 /2
⋅ e x / 2dx
= ∫ tan x ⋅ dx + c = lnsecx + c
 1  − x2 / 2

ln sec x
y= + c ⋅ cos x  e = −x + c .....(iii)
sec x  y +1
Apply boundary condition, we get c = Integration constant
y(0) = 0 Given,
ln sec 0 y(2) = 0 i.e. when x = 2 then
Then, 0 = + c ⋅ cos 0 y=0
sec 0 From equation (iii),
=0+c×1 e–2 = –2 + c
c=0 c = 2 + e–2
∴ y = cosx⋅lnsecx 1 −
1

π π π 1 At, x = 1, ⋅ e 2 = −1 + e − 2 + 2
∴ y   = cos ⋅ ln sec = ⋅ ln⋅ 2 y +1
4 4 4 2 −
1

π 1 e 2
y  = ln 2 (y + 1) =
4 2 2 1 + e −2
Now, putting the value of (y + 1) in equation (i),
π 1 −
1
or y  = log e 2  − 12 
4 2 2 e 2 1
y′(1) =  e ⋅ − 
∴ Option (b) is correct. 1 + e −2  e 2
1 
 1 + e −2 
Differential Equation 1185 YCT

1 This is linear differential equation,
e 2  1 − 1 − e −2  1
2 ∫ dy
=
1 + e −2  1 + e −2  I.F. = e y = y 2
5 5 3 Solution of differential equation (i),
− −
−e 2 −e 2 −e 2 xy 2 = ∫ 10y 2 y 2 dy + c
y′(1) = −2 2
= 2 = 2
(1 + e ) (e + 1) 2
(e + 1)2
4
xy 2 = 2y 5 + c .....(ii)
e Solution of differential equation (ii) passes through (0, 1).
305. Let y = y(x) be the solution f the differential 0 × 12 = 2 × 15 + c
equation cosec2 x dy + 2dx = (1+ ycos2x) cosec2 c = –2
π Solution of equation (i) is,
xdy, with y   = 0. Then, the value of
4 xy2 = 2y5 – 2
(y(0)+1)2 is equal to Now, this equation passes through (2, β)
(a) e1/2 (b) e–1/2 ∴ 2β2 = 2⋅β5 – 2
(c) e –1
(d) e β5 – β2 – 1 = 0
JEE Main 22.07.2021, Shift -II β is root of the equation,
Ans. (c) : Cosec2 x dy + 2dx y5 – y2 – 1 = 0
= (1 + y cos 2x) cosec2xdx 307. If y = y(x) is the solution curve of the
On dividing L.H. S and R.H.S by  1
cosec2 x dx differential equation x2dy +  y –  dx = 0; x >0
 x
dy  2 
 = (1 + y cos x )
+ 2  
1
dx  cos ec 2 x  and y(1) = 1, then y   is equal to
2
dy 3 1 1
− y cos 2x = 1 − 2sin 2 x (a) – (b) 3 +
dx 2 e e
 −1 
  sin 2 x
(c) 3 + e (d) 3 – e
1F = e ∫
− cos 2x dx = e 2 
∴ JEE Main 01.09.2021, Shift-II
So, 1F y = ∫ Q.1Fdx Ans. (d) : Given,
−1 1
y = y(x)
(1 − 2 sin x ) dx ]
sin 2x − sin 2 x
ye 2 = ∫e 2 2
 1
x 2 dy +  y −  dx = 0
1
− sin 2x
 x
= ∫e 2
cos 2xdx  1
x 2 dy = −  y −  dx
−1  x
Let, sin 2x = t 1 
2 − y
cos2x dx = –dt dy  x  = 1 − xy
=
1
− sin 2x −
sin 2x
dx x2 x3
ye 2
= −e 2
+c dy 1 y
= −
 π dx x 3 x 2
Now, y  = 0
4 dy y 1
+ = .....(i)

1
1 dx x 2 x 3
0 = −e 2
+c = c = 1
∫ 2 dx −
1
e IF e x = e x
1 −
1

1
1
∴ y (0) ⇒ y . 1 = – 1 + ye x = ∫ e x ⋅ 3 dx
e x
1 1 1
 1
, ( y + 1) = = e −1
2 −
y +1 = ye x = e − x  1 +  + c
e e  x
306. If the solution curve of the differential equation 1.e–1 = e–1(2) + c
(2x –10y3)dy + ydx = 0, passes through the 1
points (0,1) and (2,β β ), then β is a root of the c = −e −1 = −
e
equation 1 1
− −  1 1
(a) y5–2y –2 = 0 (b) 2y5 – 2y – 1 = 0 ye x = e x  1 +  −
5 2
(c) 2y –y –2 = 0 (d) y5 –y2 – 1 = 0  x e
JEE Main 27.08.2021, Shift-II 1 1
Ans. (d) : It is given that, y   = 3 − ⋅ e2
2 e
(2x – 10y3)dy + ydx = 0
1
dx 2x y  = 3− e
+ = 10y 2 .....(i) 2
dy y
Differential Equation 1186 YCT
dy 2
308. If x = y(logy – logx + 1) , then the solution of P = 100x − 12 × x 3/ 2 + C
dx 3
the equation is P = 100x – 8x3/2 + C
x At, x = 0, P = 2000
y
(a) log   = C y (b) log   = C x 2000 = 0 – 0 + C
y x C = 2000
 y x P = 100x – 8x3/2 + 2000
(c) x log   = C y (d) y log   = C x At, x = 25
x  y P = 100(25) – 8(25)3/2 + 2000
AIEEE-2005
P = 2500 – 8 × 25 × 251/2 + 2000
Ans. (b) : Given, P = 4500 – 8 × 25 × 5
dy P = 4500 – 1000
x = y(log y − log x + 1)
dx P = 3500
dy y 310. The population p(t) at time t of a certain mouse
= (log y − log x + 1)
dx x species satisfies the differential eq.
dy y   y   dp(t)
= log   + 1 .....(i) = 0.5(t) – 450 . If p(0) = 850, then the time
dx x   x   dt
y at which the population becomes zero is
Let, =t (a) 2 log 18 (b) log 9
x
y = xt 1
(c) log18 (d) log 18
dy dt 2
=x +t AIEEE-2012
dx dx
From equation (i), Ans. (a) : Given,
dt dp(t)
x + t = t [log t + 1] = 0.5p(t) − 450, p(0) = 850
dx dt
dt dp(t) 1
x = t log t = p(t) − 450
dx dt 2
dt dx dp(t) p(t) − 900
= =
t log t x dt 2
dp(t)
dt dx 2 = p(t) − 900
∫ t log t = ∫ x + C dt
2dp(t)
logt = logx + logC = dt
logt = log(xC) p(t) − 900
logt = Cx dp(t)
2∫
p(t) − 900 ∫
= dt
y  y
log   = Cx Q t = 
x  x 2log|p(t) – 900| = t + c .....(i)
309. At present, a firm is manufacturing 2000 items. At, t=0
it is estimated that the rate of change of 2log|p(0) – 900| = 0 + c
production p w.r.t. additional number of 2log|850 – 900| = c
dp 2log|–50| = c
workers, x is given by = 100 – 12 x . If the c = 2log50
dx
firm employees 25 more workers, then the new From equation (i),
level of production of items is 2log|p(t) – 900| = t + 2log50
(a) 2500 (b) 3000 Taking, p(t) = 0
(c) 3500 (d) 4500 2log|0 – 900| = t + 2log50
JEE Main-2013 2log900 = t + 2log50
2log900 – 2log50 = t
Ans. (c) : Given,
2(log900 – log50) = t
dP
= 100 − 12 x  900 
dx 2 log  =t
 50 
(
dP = 100 − 12 x dx ) 2log18 = t

∫ dP = ∫ (100 − 12 x ) dx
t = 2log18
311. The solution of the differential equation,
x 3/ 2 dy
P = 100x − 12 +C = ( x – y ) , when y(1) = 1, is
2

3/ 2 dx
Differential Equation 1187 YCT
2–y Ans. (b) : Given,
(a) log e = 2(y – 1) f(1) = 2, f′(x) = f(x)
2–x
and h(x) = f(f(x))
1+ x – y f′(x) = f(x)
(b) – log e =x+y–2 f ′(x)
1– x + y =1
f (x)
2–x Integrating both sides,
(c) log e =x–y
2–y f(x) = cex
f(1) = ce
1– x + y 2 = ce
(d) – log e = 2(x – 1)
1+ x – y 2
c=
JEE Main 11.01.2019, Shift-II e
Ans. (d) : Given, f(x) = cex
2
dy f(x) = e x
= (x − y) 2 , y(1) = 1 e
dx
2
Let, x–y=t f(2) = e2
y=x–t e
dy dt Q h(x) = f(f(x))
= 1− h′(x) = f′(f(x)) ⋅ f′(x)
dx dx h′(1) = f′(f(1)) ⋅ f′(1)
dy h′(1) = f′(2) × f(1) {Q f′(x) =f(x)}
∴ = (x − y) 2
dx h′(1) = f(2) × f(1)
dt 2
1− = t2 h′(1) = e 2 × 2
dx e
dt h′(1) = 4e
= 1− t 2

dx dy xy
313. If = 2 ;y(1) = 1; then a value of x
dx 1 dx x + y2
=
dt 1 − t 2
satisfying y(x) = e is
1 1
dx = dt (a) 3e (b) 3e
1− t 2
2
1 e
∫ dx = ∫ 1 − t 2 dt (c) 2e (d)
2
1 1+ t JEE Main 09.01.2020, Shift-II
x + c = log
2 1− t Ans. (b) : Given,
dy xy
1 1+ x − y = , y(1) = 1
x + c = log dx x 2 + y 2
2 1− x + y
xy
Q y(1) = 1
dy 2
1 = 2x 2
log(1) = 1 + c dx x + y
2
c = –1 x2
y
1 1+ x − y dy
log = x −1 = x
2 1− x + y 2
dx y
1+  
1+ x − y x
log = 2(x − 1)
1− x + y Let, t=
y
1− x + y x
− log = 2(x − 1) y = xt
1+ x − y dy dt
=x +t
312. Let f be a differentiable function such that f(1) dx dx
= 2 and f'(x) = f(x) for all x∈ ∈R. If h(x) = f(f(x)), dt t
then h'(1) is equal to ∴ x +t =
dx 1 + t2
(a) 4e2 (b) 4e dt t
(c) 2e (d) 2e2 x = −t
dx 1 + t 2
JEE Main 12.01.2019, Shift

Differential Equation 1188 YCT


dt t − t − t 3 1  1 + x 2 – 1
x = 1 + y2 – 1 – x 2 =
dx 1+ t2 (d) log e   +c
2  1+ x +1
2

dt t3
x =− JEE Main 06.09.2020. Shift-I
dx 1+ t2 Ans. (a) : Given,
1+ t 2
dx dy
3
dt = − 1 + x 2 + y 2 + x 2 y 2 + xy =0
t x dx
1+ t2 dx dy
∫ t 3 dt = − ∫ x 1(1 + x 2 ) + y 2 (1 + x 2 ) + xy
dx
=0
1 dx dy
∫ t dt + ∫ t dt = − ∫ x
−3
(1 + x 2 )(1 + y 2 ) + xy =0
dx
t −2 (1 + x 2 )(1 + y 2 )dx + xydy = 0
+ log t = − log x + c
−2
1 (1 + x 2 ) (1 + y 2 )dx = − xydy
− 2 + log t = − log x + c
2t 1 + x2 y
dx = − dy
1 1 y x 1 + y2
− × 2
+ log   = − log x + c
2 y x
  1 + x2 y
x ∫ x
dx = − ∫
1 + y2
dy
1 x2
− + log y − log x = − log x + c Let, 1 + x2 = t2
2 y2
2tdt
1 x2 2x =
− = − log y + c dx
2 y2
tdt
When, x = 1, y = 1 then dx =
1 x
− = 0+c and 1 + y2 = z2
2 2zdz
1 2y =
c=− dy
2
zdz
1 x2 1 dy =
− = − log y − y
2 y2 2
1x 2
 1 t2 tdt z 2 − 1 zdz

2 y2
= −  log y +  ∴ ∫ t2 − 1 x
= −∫
z2

y
 2
1x 2
2 log y + 1 t⋅t z2 −1 z
2
= ∫ dt = − ∫ ⋅ dz
2y 2 t2 − 1 ⋅ t2 − 1 z z −1
2

x2 = y2(2logy + 1)
t2
At, y(x) = e
x2 = e2(2loge + 1)
∫ t 2 − 1 dt = − ∫ 1dz
x2 = e2(2 + 1) (t 2 − 1) + 1
x2 = 3e2 ∫ (t 2 − 1) dt = −z + c
x = 3e
1
314. The general solution of the differential ∫ 1dt + ∫ t 2 − 1 dt = −z + c
dy
equation 1 + x 2 + y 2 + x 2 y 2 + xy = 0 is 1  t −1
dx t + log   = −z + c
(where c is a constant of integration) 2  t +1
 1 + x 2 + 1 1  1 + x2 − 1 
1 1 + x 2 + log   = − 1 − y2 + c
(a) 1 + y 2 + 1 + x 2 = log e   +c  1 + x2 + 1 
2  
2  1 + x – 1
2

1  1 + x 2 + 1 1  1 + x2 −1 
(b) 1 + y2 – 1 + x 2 = log e   +c 1 + x 2 + 1 + y 2 = − log  +c
2  1+ x2 +1 
2  1 + x – 1  
2

1  1 + x 2 – 1 1  1 + x2 + 1 
(c) 1 + y2 + 1 + x 2 = log e   +c 1 + x 2 + 1 + y 2 = log  +c
2  1 + x2 −1 
2  1 + x + 1
2
 

Differential Equation 1189 YCT


315. If y = y(x) is the solution of the differential eq., Let, loge(y + 3x) = t
 dy  1  dy  dt
ey  – 1  = ex such that y(0) = 1, then y(1) is  + 3 =
 dx  y + 3x  dx  dx
equal to dy dt
(a) 2 + loge 2 (b) 2e + 3 = (y + 3x) .....(i)
(c) 1 + loge 2 (d) loge 2 dx dx
JEE Main 07.01.2020, Shift-I dy y + 3x
Q − +3=0
Ans. (c) : Given, dx loge (y + 3x)
 dy  dy y + 3x
ey  − 1 = ex +3=
 dx  dx log e (y + 3x)
dy ex From equation (i),
−1 = y dt (y + 3x)
dx e (y + 3x) =
dy x−y
dx t
= e +1 dt 1
dx =
Let, x–y=t dx t
dy dt tdt = dx
1− =
dx dx ∫ tdt = ∫ dx
dy dt
= 1− t2
dx dx x= +c
dt 2
∴ 1− = et + 1 t2
dx x− =c
dt 1 2
− = −t Putting the value of t
dx e
–e–tdt = dx 1
x − [log e (y + 3x)] = c
2

− ∫ e − t dt = ∫ dx 2
–t 317. Let C1 be the curve obtained by the solution of
e =x+c
dy
e–(x–y) = x + c differential equation 2xy = y 2 – x 2 , x > 0.
y–x dx
e =x+c
Where, x = 0, y(0) = 0 Let the curve C2 be the solution of
ey(0) – 0 = 0 + c 2xy dy
e0 – 0 = 0 + c = . If both the curves pass through
x 2 – y 2 dx
1=c
c=1 (1, 1), then the area enclosed by the curves C1
y–x
e =x+1 and C2 is equal to
At, x=1 π
(a) π – 1 (b) –1
ey–1 = 1 + 1 2
ey–1 = 2 π
Taking log in both side, (c) π + 1 (d) +1
logeey–1 = log2 4
(y – 1)logee = log2 JEE Main 16.03.2021, Shift-II
y = 1 + log2 Ans. (b) : Given,
316. The solution of the differential equation dy
2xy = y2 − x 2
dy y + 3x dx
– + 3 = 0 is (Where c is constant dy y 2 − x 2
dx log e ( y + 3x ) =
of integration) dx 2xy
(a) x –loge (y + 3x) = c (y 2 − x 2 )
1 dy x2
(b) y + 3x – (log e x) 2 = c =
2 dx 2xy
(c) x – 2loge (y + 3x) = c x2
2
1 y
(d) x – (loge(y + 3x)2 = c −1
2 dy  x 
JEE Main 04.09.2020, Shift-II =
dx y
Ans. (d) : Given, 2 
x
dy y + 3x y
− +3=0 Let, =t
dx loge (y + 3x) x
Differential Equation 1190 YCT
y = tx 318. Let y = y(x) be the solution of the differential
dy dt y
= t+x equation e x 1 – y 2 dx +   dy = 0, y(1) = – 1.
dx dx x
dt t 2 − 1 Then, the value of [y(3)]2 is equal to
∴ t+x = (a) 1 – 4e3 (b) 1 – 4e6
dx 2t 3
(c) 1 + 4e (d) 1 + 4e6
dt t − 12
x = −t JEE Main 20.07.2021, Shift-I
dx 2t Ans. (b) : Given the differential equation–
dt t 2 − 1 − 2t 2 y
x = e x 1 − y 2 dx + dy = 0 with B.C. y(1) = –1
dx 2t x
dt −(t 2 + 1) y ⋅ dy
x = or = − x ⋅ e x dx
dx 2t 1− y 2

2t 1 Integrating both the sides, we get–


dt = − dx
t2 +1 x y ⋅ dy
∫ 1 − y2 = ∫ −x ⋅ e ⋅ dx + C
x
Integrate,
log(t2 + 1) = –log x + C
Let, 1 – y2 = t2
  y 2 
log    + 1 = − log x + C ∴ 0 – 2y⋅dy = 2t⋅dt
 x  
  y⋅dy = –t⋅dt
 y2  t ⋅ dt
∴ ∫ − t = ∫ −x ⋅ e + C
x
log  2 + 1 = − log x + C
x 
Q Curve passing through (1, 1) or t = ∫ x ⋅ e x + C = x ⋅ e x − ∫ 1 ⋅ e x dx + C
1  1 − y 2 = (x − 1) ⋅ e x + C
log  + 1 = − log1 + C
1  ∴ 1 – y2 = [(x – 1)⋅ex + C]2
C = log 2 or y2 = 1 – [(x – 1)⋅ex + C]2
 y2  2
So, log  2 + 1 = − log x + log 2 or y = 1 − (x − 1) ⋅ e x + C 
 x  Now, y(1) = –1
 x 2 + y2  2
log  2  = log   ∴ 1 = 1 − [0 + C]2
 x  x 1 = 1 – C2
x +y
2 2
2 C=0
=
x2 x ∴ y = 1 − [(x − 1) ⋅ e x ]2
x2 + y2 = 2x
x2 + y2 – 2x = 0 (for curve C1) ∴ y(3) = 1 − [2 ⋅ e3 ]2
Similarly, y = 2x − x 2 ∴ [y(3)]2 = 1 – 4⋅e6
∴ Option (b) is correct.
dy 2xy
= 319. Let y = y(x) be the solution of the differential
dx x 2 − y 2   y +1 

 
y equation  (x + 2)e x + 2  + (y +1)  dx
Putting, t =  
x  
= (x + 2)dy, y (1) = 1. If the domain of y = y(x)
is an open interval α, β , then |α + β | is equal to
..... .
JEE Main 22.07.2021, Shift-II
Ans. (4) : Given the differential equation,
y +1
  dx = (x + 2)dy
(x + 2) ⋅ e + (y + 1) 
x +2

y +1
x2 + y2 – 2y = 0 dy (x + 2) ⋅ e x + 2 + (y + 1)
∴ =
1  dx x+2
 area of C1 − Area of ∆AOB  y +1
(y + 1)
4  = e x+2 +
Bounded area (x + 2)
 1  1  π
= 2  × π  − × 1 × 1 = − 1 y +1
 4  2  2 Now, put =Z
x+2
Differential Equation 1191 YCT
∴ y + 1 = z(x + 2) = zx + 2z 1 1
∴ − ⋅ e −4y = ⋅ e3x + C
dy dz dz 4 3
∴ = z+x⋅ +2
dx dx dx At, x = 0, y = 0
dz 1 1
= z + (x + 2) − e0 = e0 + c
dx 4 3
dz 7
∴ We get z + (x + 2) = ez + z c=−
dx 12
dz e −4y e3x
∴ (x + 2) = ez = +c
dx −4 3
dx 4
or e − z dz = e −4 y = − e 3x − 4c
x+2 3
dx 4 3x 7
= − e + 4×
∫ e ,dz = ∫ x + 2 + c
−z
or 3 12
–e–z = ln(x + 2) + c −4 y 7 − 4e3x
e =
1 3
or e − z = ln +c
x+2 e4 y =
3
k 7 − 4e3x
= ln (Q where k is some constant)
x+2 1  3 
y = ln  
  k  4  7 − 4e 3x 
or −z = log e loge  
  x + 2   
 2  1 3
y  − ln 2  =  
 y +1    k   3  4 3 ln 2  
 −2 
or −  = log e  log e  
 x + 2    x + 2   7 − 4e  3  
Given, y(1) = 1 1  3 1
= ln   = − ln 2 = αln2
(1 + 1) k 4 6 4
− = log log   1
1+ 2  3 ∴ α=−
2 k 4
or − = log log   321. The differential equation for the family of
3  3 curves x2 + y2 – 2ay = 0, where a is an arbitrary
k constant, is
or log = e −2 / 3
3 (a) 2 (x2 – y2) y' = xy (b) 2 (x2 + y2) y' = xy
2 2
k −2 / 3 (c) (x – y ) y' = 2xy (d) (x2 + y2) y' = 2xy
= ee AIEEE-2004
3
320. Let y = y(x) be solution of the differential Ans. (c) : Given the curves,
 dy  x2 + y2 – 2ay = 0 .....(i)
equation loge   = 3x + 4y, with y (0) = 0. If 2x + 2y⋅y′ – 2ay′ = 0
 dx 
(x + yy′)
 2  ∴ a=
y  – log e 2  = αlog e 2, then the value of α is y′
 3 
Putting the value of a in equation (i), we get–
equal to
′)y
(a) –
1
(b)
1 ( x 2 + y2 ) = 2(x +yyy ′
4 4
or (x2 + y2)y′ = 2xy + 2y2⋅y′
1
(c) 2 (d) – or (x2 – y2)y′ = 2xy
2
JEE Main 27.07.2021, Shift-I 322. The differential equation of the family of
curves, x2 = 4b(y + b) , b∈R, is
Ans. (a) : Given the differential equation–
(a) xy'' = y' (b) x(y')2 = x + 2yy'
dy 2
(c) x(y') = x – 2yy' (d) x(y')2 = 2yy' – x
log e = 3x + 4y , with y(0) = 0
dx JEE Main 08.01.2020. Shift-II
dy Ans. (b) : Given the curves,
= e(3x + 4 y) = e3x ⋅ e 4 y
dx x2 = 4b(y + b), b ∈ R
∴ e–4y dy = e3x⋅dx 2x = 4by′
Integrating both the sides, we get– x
∴ b=
∫e ⋅ dy = ∫ e3x dx
−4 y
2y′

Differential Equation 1192 YCT


Putting the value of b in the original equation, we get– 2
 dy   dy 
4x  x  (c) y   + 2x   – y = 0
x2 = y+ ′  dx   dx 
2y′  2y   dy   dy 
(d) y   + 2x   – y = 0
2x  x   dx   dx 
or x2 =  y+ 
y′  2y′  JEE Main 18.03.2021, Shift-I
2x (2yy′ + x) Ans. (c) : Given the equation of the parabola as,
or x2 = y2 = 4a(x + a)
y′ 2y′ Differentiating w.r.t. x, we get–
2
or x⋅y′ = 2yy′ + x 2yy′ = 4a(1 + 0)
or x⋅(y′)2 = 2yy′ + x yy′
323. A differential equation representing the family ∴ a=
2
of parabolas with axis parallel to y-axis and Now, putting the value of a in the original equation, we
whose length of latus rectum is the distance of get–
the point (2,-3) from the line 3x + 4y = 5, is
given by 4(yy′)  yy′ 
y2 = x + 
d2 y d2 x 2  2 
(a) 10 2 = 11 (b) 11 2 + 10 2
y = yy′(2x + yy′)
dx dy
or y2 = y2⋅y′2 + 2xy⋅y′
d2 x d2 y 2
(c) 10 = 11 (d) 11 = 10  dy   dy   ′ dy 
dy 2 dx 2 or y = y ⋅   + 2x   Q y = 
 dx   dx   dx 
JEE Main 27.08.2021, Shift -II 2
 dy   dy 
Ans. (d) : Let the equation of parabola be– or y ⋅   + 2x   − y = 0
x2 = 4ay  dx   dx 
∴ Option (c) is correct.
dy y
325. Solve + = x3 .
dx x
x4 c x3
(a) y = + (b) y = + cx
5 x 3
Solving, y = a and x2 = 4ay x3 x3
2 2 (c) y = + c (d) y = +c
We get, x = 4a⋅a = 4a 3 4
∴ x = ±2a J&K CET-2014
∴ Length of latus rectum, Ans. (a) : Given the differential equation,
= 4a dy y 1
Now, distance of the point (2, –3) from the line 3x + 4y + = x 3 ; P = ,Q = x 3
dx x x
– 5 = 0 is ∴ Integrating factor (I.F.),
3 × 2 + 4( −3) − 5 11 1
∫ x ⋅dx = e ln x = x
= = e
32 + 42 5
11 ∴ y ⋅ I.F. = ∫ Q ⋅ I.F.dx + c
∴ 4a =
5 ∴ y ⋅ x = ∫ x 3 ⋅ x dx + c ,
Now, differentiating x2 = 4ay w.r.t. x, we get–
Where c is integration constant,
dy
2x = 4a x5
dx = ∫ x 4 dx + c = + c
Again differentiating, we get– 5
4
x c
d 2 y 11 d 2 y ∴ y= +
2 = 4a ⋅ 2 = ⋅ 2 5 x
dx 5 dx
2
326. Differential equation of those circles which pass
d y through origin and their centre lie on y-axis
∴ 11 ⋅ 2 = 10
dx will be
(a) ( x 2 − y 2 ) + 2xy = 0
324. The differential equation satisfied by the dy
system of parabolas y2 = 4a(x + a) is dx
2
 dy   dy  (b) ( x 2 − y 2 ) − 2xy = 0
dy
(a) y   – 2x   – y = 0
 dx   dx  dx

(c) ( x 2 − y 2 ) − xy = 0
 dy 
2
 dy  dy
(b) y   – 2x   + y = 0 dx
 dx   dx 
Differential Equation 1193 YCT
Integrate both sides
(d) (x 2
− y2 )
dy
dx
+ xy = 0 y
y 1 y dx
CG PET-2021 ∫ e x d  x  + ∫ d  = ∫
 y x x
2
Ans. (b) : Given the equation of the circles which pass 1−  
through origin and their center lie on y-axis. x
Let the center is (0, a) and radius is r. y
 y
∴ The equation becomes– ⇒ e x + sin −1   = ln x + c
(x – 0)2 + (y – a)2 = r2 x
It passes through the origin (0, 0) will satisfy, the It passes through (1, 0)
equation– 1+0=0+c⇒c=1
∴ (0 – 0)2 + (0 – a)2 = r2 It passes through (2α, α)
r=a 1
1
∴ The equation of circles becomes– e 2 + sin −1 = ln 2α + 1
2
x2 + (y – a)2 = a2 .....(i)
Now, differentiating w.r.t x, we get– π
⇒ ln 2α = e + − 1
2x + 2(y – a)⋅y′ = 0 6
 π 
x  e + −1
∴ (y − a) = − ⇒ 2α = e  6 
y′
π 
 x 1  + c −1
and a =  y +  , putting the value of (y – a) and a ⇒ α = e 6
y′  2

in equation (i), we get– 1 π 
⇒ α = exp  + e − 1
2 2 6 
x 2
 x x 2
2xy
x 2 + 2 =  y +  = y2 + 2 + 328. The differential equation of the family of lines
y′  y′  y′ y′ passing through the origin is
2xy dy dy
or (x − y ) =
2 2
(a) x +y=0 (b) x + =0
y′ dx dx
dy  ′ dy  dx dy
or (x 2 − y 2 ) − 2xy = 0 Q y =  (c) x −y =0 (d) =x
dx  dx  dx dx
327. Let the solution curve y = y(x) of the Manipal UGET-2017
differential equation. Ans. (c) : The equation of line passing through the
 x y 
dy  x y  origin is given by
 + ex  x = x+ + ex  y y = mx ……(i)
 x 2 – y 2  dx  x 2 – y 2  on differentiating w.r.t. x, we get
pass through the points (1, 0) and (2α α, α), α > dy dy y
0. Then α is equal to
⇒ =m⇒ = [from equation...(i)]
dx dx x
1 π  1 π  dy
(a) exp  + e − 1 (b) exp  + e − 1 ⇒ x −y =0
2  6  2  3  dx
π  π  329. The general solution of the differential
(c) exp  + e + 1 (d) 2 exp  + e − 1 dy
 6   3  equation = ytan x – y 2sec x is
JEE Main-28.06.2022, Shift-I dx
Ans. (a) : (a) tan x = ( c + sec x ) y
 x y 
dy  x y 
(b) sec y = ( c + tan y ) x
 + ex  x = x+ + ex  y
 x 2 − y2


 dx
 2
 x −y
2 
 (c) sec x = ( c + tan x ) y
y
x (d) tan y = ( c + sec x ) x
⇒ e x (x dy − y dx) + (xdy − ydx) = xdx Manipal UGET-2020
x − y2
2
Ans. (c) : We have,
Dividing both side by x2 dy
= y tan x − y 2 sec x
 xdy − ydx   xdy − ydx  dx
y
1 dx
⇒ ex  +  =
 x2   y 
2 x2  x 1 dy 1
1−   ⇒ − tan x = − sec x
x y 2 dx y
1 1
y
y 1  y  dx Put − = v ⇒ 2 dy = dv
⇒ exd   + d  = y y
x y x x
2
dv
1−   ∴ + v tan x = sec x ...(i)
x dx
Differential Equation 1194 YCT
Here, Ans. (c) : Given,
P = tan x ⇒ ∫ tan x dx = log sec x x(1 + y 2 )1/ 2 dx + y(1 + x 2 )1/ 2 dy = 0
I ⋅ F⋅ = elog sec x = sec x Equation can be rewritten as.
By, equation (i), we have:- x y
dx + dy = 0
dv 1+ x 2
1 + y2
sec x + v sec x = sec 2 x
dx On integrating both side, we get
Integrating both sides:-
1 + x 2 + 1 + y2 = c
vsec x = ∫ sec2 x dx
1 dy
⇒ sec x = tan x + c 332. If = 1 – y and y (0) = 3, then y (loge 8) is
y dx
equal to
sec x = y ( tan x + c ) (a) 5 (b) 5/4
330. The solution of the differential equation (c) 0 (d) –5
Assam CEE-2018
dy
a+x + xy = 0 is Ans. (b) : Given,
dx dy
2
( 2a − x ) x + a
2
(a −x ) x+a = 1 − y with y(0) = 3
(a) y = ce 3
(b) y = ce 3 dx
2
Here, we get–
− ( 2a − x ) x + a
(c) y = ce 3 (d) None of these dy
= dx
Manipal UGET-2020 1 −y
Ans. (a) : We have:- or –ln(1 – y) = x + c
dy  1 
a+x + xy = 0 or ln   = x+c (c is integration constant)
dx  1− y 
dy 1
a+x = − xy or = e( x +c) = e x ⋅ k
dx 1− y
dy −x ∴ 1 – y = e–x⋅k
= dx
y a+x Here k is some constant,
On integrating both sides, we get:- or y = 1 – e–x⋅k
 −x − a a  Using boundary condition y(0) = 3, we get–
log y − log c = ∫  +  dx 3 = 1 – e–0⋅k
 a + x a + x  ∴ k = –2
y 1 ∴ y = 1 + 2⋅e–x
log   = − ∫ x + a dx + a ∫ dx
c a+x Now, y(loge8) = 1 + 2 ⋅ e − loge 8
( x + a ) + a (a + x )
3/ 2 1/ 2 1
y loge 2 1 5
log   = − = 1+ 2 ⋅e 8 = 1+ = 1+ =
c 3/ 2 1/ 2 8 4 4
333. If y = f (x), passes through the point (1 – 1) and
y 2
log   = − ( x + a ) x + a + 2a x + a satisfies the equation y (1 + xy) dx = x dy, then f
c 3
1
 y 2   is equal to
log   = ( 2a − x ) x + a 2
c 3 4 2
y
2
( 2a − x ) x + a (a) − (b)
=e 3 5 5
c −4 2
2
( 2a − x ) x + a (c) (d) −
y = c ⋅ e3 5 5
Assam CEE-2018
331. The general solution of
Ans. (c) : Given the D.E.
x (1 + y 2 ) dx + y (1 + x 2 ) dy = 0is
1/2 1/2
y(1 + xy)dx = x⋅dy
(a) sin −1 x + sin −1 y = c dy  y 
∴ =   (1 + xy)
(b) x + y = (1 + x ) + (1 + y ) + c x
2 2 2 1/ 2 2 1/ 2 dx
y
= z or y = xz
(c) (1 + x 2 ) + (1 + y 2 ) = c
1/ 2 1/ 2 Put,
x
(d) tan −1 x − tan −1 y = c dy dz
∴ = z+x⋅
Manipal UGET-2018 dx dx
Differential Equation 1195 YCT
dz 335. The integrating factor f linear differential
∴ z+x⋅ = z(1 + xy) dy
dx equation + ytanx – secx = 0
= z + z⋅x⋅z⋅x dx
dz (a) cosx (b) secx
∴ x⋅ = z2 ⋅ x2 (c) ecosx (d) esinx
dx
GUJCET-2023
dz
or = dx ⋅ x Jamia Millia Islamia-2012
z2 Ans. (b) : Given the differential equation,
dz
∴ ∫ z 2 = ∫ x ⋅ dx + c dy
+ y ⋅ tan x = sec x
dx
z −2 +1 x 2 Here, we compare the above with the differential
or = +c equation,
−2 + 1 2
1 x2 dy
or − = +c + Py = Q
z 2 dx
∴ Integrating factor (I.F.) = e ∫ = e ∫
Pdx tan x ⋅dx
x x2
− = +c = elog|secx| = |secx|
y 2
336. Eliminating a and b, we get
−x
∴ y= 2 y = a log x + b, we get
x
+c (a) xy2 + y1 = 0 (b) xy – y2 = 0
2 (c) xy1 + y2 = 0 (d) x2y2 + x = 0
Now, given that it passes through (1, –1) AP EAMCET-23.09.2020, Shift-I
−1 Ans. (a) : Given,
∴ −1 = y = a log x + b
1
+c Differentiating w.r.t. x, both sides, we get –
2
1 dy a a
+c =1 ∴ = +0=
2 dx x x
1 Again, differentiating w.r.t. x, we get –
c= d2 y a x dy 1 dy
2
2
= a ⋅ −1 ⋅ x −2 = − 2 = − 2 ⋅ =− ⋅
−2x dx x x dx x dx
∴ y= 2 y1
x +1 ∴ y2 = −
1 x
−2 × or xy2 + y1 = 0
1 2
∴ y  = 337. An integrating factor of the differential
 2  1 +1 equation
4

(1 + x ) ( )
x4
(1 – x ) dy
3
−1 4 2
+ xy = 1 – x2 is
= =− dx 5
5 5
4 x
334. Differential equation of all parabolas having (a) 1 – x2 (b)
their axes coincident with the x-axis is : 1 – x2
(a) yy1 + y12 = 0 (b) yy2 + y12 = 0
x2 1
(c) both (A) and (B) (d) None of these (c) (d)
2
Assam CEE-2017 1– x 1 – x2
Ans. (b) : Let the equation of the parabola is, AP EAMCET-2012
y2 = 4ax Ans. (d) : Given,
dy Differential equation –
Now, 2y = 4a
x4
( )
3
dx dy
(1 − x 2 ) + x.y = 1 − x2
dy 2a
= dx 1+ x 5

dx y dy x x4
or + ⋅y = ⋅ 1 − x2
or y⋅
dy
= 2a , Again differentiating w.r.t. x, we dx 1 − x 2 1 + x5
dx Now, we compare the above with the D.E.
get– dy
2
+ P(x) ⋅ y = Q( x)
d 2 y  dy  dx
or y⋅ +  =0 ∴ We get,
dx 2  dx 
x
or y ⋅ y 2 + y12 = 0 P( x) =
1 − x2
Differential Equation 1196 YCT
Now, x y
or − − ln = ln x + c
I.F. = e ∫
P( x ) ⋅dx
y x
x y
Here, ∫P ( x) ⋅ dx or − = ln + ln x + c
y x
x
=∫ ⋅ dx = lny + c
1 − x2 x
Put, 1–x =t 2
∴ ln y = − + c
y
–2x⋅dx = dt x x
− +c −
1 y=e y
= A⋅e y
or xdx = − ⋅ dt
2
x 1 dt dy y2
∴ ∫ 1 − x 2 dx = − 2 ∫ t 339. If the general solution of =
dx xy – y 2 – x 2
is

1 1 y
= − ln t = − ⋅ ln(1 − x 2 ) tan –1   = f(y) + C , then f (e3) =
2 2 x
= ln(1 – x2)–1/2 (a) 0 (b) 1
2 −1 / 2
∴ I.F. = eloge (1− x ) (c) 2 (d) 3
1 AP EAMCET-07.07.2022, Shift-II
= (1 − x 2 ) −1/ 2 = Ans. (d) : Given,
1− x2 Differential equation –
338. The general solution of xdy – ydx = ydy is 2
(a) y = Ae–x/y (b) y = A ex y
y2  
y x y dy
= = x
(c) = Ae x (d) + =C dx xy − y − x2 2 2
x y x y y
    −1

AP EAMCET-08.07.2022, Shift-II x x
Ans. (a) : Given, y
Differential equation – Put, =z
x
xdy – y⋅dx = y⋅dy dy z2
or (x – y)dy = y⋅dx =
dx z − z 2 − 1
 y y
y = x⋅z
or  1 −  dy = ⋅ dx Now,
 x x dy dz
y ∴ = z+x⋅
Put, =z dx dx
x dz z2
∴ y = xz ∴ z+x⋅ =
dx z − z 2 − 1
dy dz
= z+x⋅ dz z2 z 2 − z(z − z 2 − 1)
dx dx or x⋅ = −z =
We have, dx z − z − 1
2
z − z2 −1
dy y/x z − z2 + z3 + z
2

= =
dx 1 − y / x z − z2 − 1
Putting the values, we get– dz z(z 2 + 1)
x. =
dz z dx (z − z 2 − 1)
z+x⋅ =
dz 1 − z (z − z 2 − 1) dx
or x⋅
dz
=
z
−z =
z − z + z2 ∴ ∫ z(z 2 + 1) ⋅ dz = ∫ x + c = lnx + c
dx 1 − z 1− z 1 z 1
z2 ∴ ∫ 2 dz − ∫ 2 dz − ∫ ⋅ dz = ln x + c
= z +1 z +1 z(z + 1)
2

1− z Now, partial fraction


(1 − z) dx
∴ ⋅ dz = 1 A  Bz + C 
z2 x = + 
z(z 2 + 1) z  z 2 + 1 
 1 1 dx
∫  z 2 − z  dz = ∫ x + c 1 = A(z + 1) + (Bz + C)⋅z
2
or or
or 1 = Az2 + A + Bz2 + Cz
z −2 +1 = z2(A + B) + Cz + A
− ln z = ln x + c ∴ A=1
−2 + 1
A+B=0
1
or − − ln z = ln x + c B = –1
z C=0
Differential Equation 1197 YCT

You might also like